Med-Surg 555 Exam 1

¡Supera tus tareas y exámenes ahora con Quizwiz!

Critical thinking characteristics include a. Considering what is important in a given situation. b. Accepting one, established way to provide patient care. c. Making decisions based on intuition. d. Being able to read and follow physician's orders.

a. Considering what is important in a given situation.

Be tolerant of the patient's views and beliefs a. open-mindedness b. analyticity c. truth seeking d. systematicity

a. open-mindedness

Mr. Johnson's BP measures 135/87 mmHg. His mean arterial pressure (MAP) measures: a. 48 mmHg. b. 103 mmHg. c. 119 mmHg. d. 111 mmHg.

b. 103 mmHg.

Which diagnosis below is NANDA-I approved? a. Sleep disorder b. Acute pain c. Sore throat d. High blood pressure

b. Acute pain

Of the following disorders, which is caused by a virus? a. Corns b. Plantar warts c. Athletes feet d. Callus

b. Plantar warts

Normally, Mr. Johnson's lowest body temperature would be around: a. 7 PM b. 2 PM c. 3 AM d. 12 PM

c. 3 AM

A nurse and patient take action to meet health-related goals. The nurse is in which phase of the helping relationship? a. Pre-interaction b. Orientation c. Working d. Termination

c. Working

In which step of the nursing process does the nurse determine if the patient's condition has improved and whether the patient has met expected outcomes? a. Assessment b. Planning c. Implementation d. Evaluation

d. Evaluation

Repositioned patient on right side. Encouraged patient to use patient-controlled analgesia (PCA) device. a. subjective b. objective c. assessment d. plan

d. plan

Physical care technique ________ a. assisting patient with oral care b. protecting a violent patient from injury c. discussing a patient's options in choosing palliative care d. using safe patient handling during positioning of a patient

d. using safe patient handling during positioning of a patient

Category/Stage I pressure ulcer a. nonblanchable redness of intact skin. Discoloration, warmth, edema, or pain may also be present. b. partial thickness skin loss or intact blister with serosanginous fluid c. full thickness tissue loss, muscle and bone visible. may include undermining d. full thickness skin loss, subcutaneous fat may be visible. may include undermining

a. nonblanchable redness of intact skin. Discoloration, warmth, edema, or pain may also be present.

An elderly patient comes to the hospital with a complaint of severe weakness and diarrhea for several days. Of the following problems, which is the most important to assess initially? a. Malnutrition b. Dehydration c. Skin breakdown d. Incontinence

b. Dehydration Dehydration caused by fluid loss from the intestinal tract is an immediate and possibly dangerous consequence of diarrhea.

A patient who is hospitalized has just been diagnosed with diabetes. He is going to need to learn how to give himself injections. Which teaching method does the nurse use? a. Simulation b. Demonstration c. Group instruction d. One-on-one discussion

b. Demonstration Demonstration is used to help patients learn psychomotor skills.

One purpose of using standard formal nursing diagnoses in practice is to a. Form a language that can be encoded only by nurses. b. Distinguish the nurse's role from the physician's role. c. Allow for the communication of patient needs to assistive personnel. d. Help nurses focus on the scope of medical practice.

b. Distinguish the nurse's role from the physician's role.

The nurse delegates needed hygiene care for an elderly stroke patient. Which intervention would be appropriate for the nurse assistant to accomplish during the bath? a. Checking distal pulses b. Providing range-of-motion (ROM) exercises to extremities c. Determining type of treatment for stage 1 pressure ulcer d. Changing the dressing over an intravenous site

b. Providing range-of-motion (ROM) exercises to extremities ROM may be delegated to assistive personnel. The other activities should be performed by the nurse.

The critical thinking skill of EVALUATION in nursing practice can be best described as a. Examining the meaning of data. b. Reviewing the effectiveness of nursing actions. c. Supporting findings and conclusions. d. Searching for links between data and the nurse's assumptions.

b. Reviewing the effectiveness of nursing actions.

The nurse is caring for a patient who has an order to change a dressing twice a day, at 0600 and 1800. At 1400, the nurse notices that the dressing is saturated with blood. What is the nurse's next action? a. Wait and change the dressing at 1800 as ordered. b. Revise the plan of care and change the dressing now. c. Reassess the dressing and the wound in 1 hour. d. Discontinue the plan of care.

b. Revise the plan of care and change the dressing now.

What does the Braden Scale evaluate? a. Skin integrity at bony prominences, including any wounds b. Risk factors that place the patient at risk for skin breakdown c. The amount of repositioning that the patient can tolerate d. The factors that place the patient at risk for poor healing

b. Risk factors that place the patient at risk for skin breakdown The Braden Scale measures factors in six subscales that can predict the risk of pressure ulcer development. It does not assess skin or wounds.

A new nurse asks the preceptor why a change-of-shift report is important since care is documented in the chart. What is the preceptor's best response? a. "A hand-off report provides an opportunity to share essential information to ensure patient safety and continuity of care." b. "A change-of-shift report provides the oncoming nurse with data to help set priorities and establish reimbursement costs." c. "A hand-off report provides an opportunity for the oncoming nurse to ask questions and determine research priorities." d. "A change-of-shift report provides important information to caregivers and develops relationships within the health care team."

a. "A hand-off report provides an opportunity to share essential information to ensure patient safety and continuity of care."

A group of nurses is discussing the advantages of using computerized provider order entry (CPOE). Which of the following statements indicates that the nurses understand the major advantage of using CPOE? a. "CPOE reduces transcription errors." b. "CPOE reduces the time needed for health care providers to write orders." c. "CPOE eliminates verbal and telephone orders from health care providers." d. "CPOE reduces the time nurses use to communicate with health care providers."

a. "CPOE reduces transcription errors." CPOE allows the provider to enter the order directly, thus eliminating the need to transcribe orders. There is no evidence that CPOE reduces the time needed for providers to write orders for their patients or the time nurses must spend communicating with providers. Nurses use CPOE systems under certain circumstances to enter orders given by a provider in person or over the phone.

The nurse is caring for a 23-year-old male client who is in the ICU with second and third degree burns over 40 percent of his body. One of the first symptoms that the client is having organ failure is that the urine output is less than: a. 30 mL/hour b. 40 mL/hour c. 50 mL/hour d. 60 mL/hour

a. 30 mL/hour An output of less than 30 mL/hr indicates possible renal alterations.

A nurse is teaching a community group about ways to minimize the risk of developing osteoporosis. Which of the following statements reflect understanding of what was taught? a. "I usually go swimming with my family at the YMCA 3 times a week." b. "I need to ask my doctor if I should have a bone mineral density check this year." c. "If I don't drink milk at dinner, I'll eat broccoli or cabbage to get the calcium that I need in my diet." d. "I'll check the label of my multivitamin. If it has calcium, I can save money by not taking another pill." e. "My lactose intolerance should not be a concern when considering my calcium intake."

a. "I usually go swimming with my family at the YMCA 3 times a week." Patients at risk for or diagnosed with osteoporosis have special health promotion needs. Encourage patients at risk to be screened for osteoporosis and assess their diets for calcium and vitamin D intake. Multivitamins do not always have the needed amount of calcium for every individual. A patient needs to know his or her requirement and make a decision based on that. b. "I need to ask my doctor if I should have a bone mineral density check this year." Patients at risk for or diagnosed with osteoporosis have special health promotion needs. Encourage patients at risk to be screened for osteoporosis and assess their diets for calcium and vitamin D intake. Multivitamins do not always have the needed amount of calcium for every individual. A patient needs to know his or her requirement and make a decision based on that. c. "If I don't drink milk at dinner, I'll eat broccoli or cabbage to get the calcium that I need in my diet." Patients at risk for or diagnosed with osteoporosis have special health promotion needs. Encourage patients at risk to be screened for osteoporosis and assess their diets for calcium and vitamin D intake. Multivitamins do not always have the needed amount of calcium for every individual. A patient needs to know his or her requirement and make a decision based on that.

What is the correct order of steps for removal of protective barriers after leaving an isolation room? 1. Remove gloves. 2. Perform hand hygiene. 3. Remove eyewear or goggles. 4. Untie top and then bottom mask strings and remove from face. 5. Untie waist and neck strings of gown. Remove gown, rolling it onto itself without touching the contaminated side. a. 1, 3, 5, 4, 2 b. 1, 5, 3, 4, 2 c. 1, 3, 4, 5, 2 d. 3, 1, 5, 4, 2

a. 1, 3, 5, 4, 2 This sequence ensures that the risk of contamination to other surfaces or health care personnel is minimized.

The nurse is working the evening shift at a hospital that uses military time for documentation. The nurse administered morphine 2 mg intravenously (IV) for pain at 3:45 PM, changed the dressing over the patient's abdominal incision at 5:34 PM, and administered Ancef 1 g IV at 8:00 PM. Using correct military time, which of the following is the correct label term-532of documentation for each task with the time that it was completed? a. 15 45, 17 34, 20 00 b. 3 45, 17 34, 20 00 c. 15 45, 5 34, 8 00 d. 3 45, 5 34, 8 00

a. 15 45, 17 34, 20 00 Military time is essentially the same as civilian time for the hours between 1 am and 12 noon, with the exception that you add a leading "0" to times before 10:00 AM (e.g., 08:00 instead of 8:00 AM). To convert military time to civilian time: For a military time that is 13:00 or larger, simply subtract 12:00 to get the standard time (e.g., if someone says "Meet me in room 202 at 15:45, subtract 12:00 from 15:45 to get 3:45 PM). To convert standard time to military time: add 12:00 to any time from 1:00 PM to 11:00 PM (e.g., if you want to say 6:30 PM in military time, add 12:00 to 6:30 to get 18:30).

A nurse is assigned to a new patient admitted to the nursing unit following admission through the emergency department. The nurse collects a nursing history and interviews the patient. What are the steps for making a nursing diagnosis in the correct order, beginning with the first step?1. Considers context of patient's health problem and selects a related factor 2. Reviews assessment data, noting objective and subjective clinical information 3. Clusters clinical cues that form a pattern 4. Chooses diagnostic label a. 2, 3, 4, 1 b. 3, 2, 4, 1 c. 2, 3, 1, 4 d. 1, 4, 3, 2

a. 2, 3, 4, 1 This is the correct steps for making a nursing diagnosis.

A patient with a malignant brain tumor requires oral care. The patient's level of consciousness has declined, with the patient only being able to respond to voice commands. What is the correct order for administration of oral care?1. If patient is uncooperative or having difficulty keeping mouth open, insert an oral airway. 2. Raise bed, lower side rail, and position patient close to side of bed with head of bed raised up to 30 degrees. 3. Using a brush moistened with chlorhexidine paste, clean chewing and inner tooth surfaces first. 4. For patients without teeth, use a toothette moistened in chlorhexidine rinse to clean oral cavity. 5. Remove partial plate or dentures if present. 6. Gently brush tongue but avoid stimulating gag reflex. a. 2, 5, 1, 3, 6, 4 b. 5, 1, 2, 3, 6, 4 c. 2, 5, 3, 1, 6, 4 d. 2, 1, 5, 3, 4, 6

a. 2, 5, 1, 3, 6, 4 This is the correct order for administration of oral care.

A patient has been newly admitted to a medicine unit with a history of diabetes and advanced heart failure. The nurse is assessing the patient's fall risks. Which of the following is the proper order of steps for the "Timed Get-up and Go Test" (TGUGT)?1. Have patient rise from straight-back chair without using arms for support. 2. Begin timing. 3. Tell patient to walk 10 feet as quickly and safely as possible to a line you marked on the floor, turn around, walk back, and sit down. 4. Check time elapsed. 5. Look for unsteadiness in patient's gait. 6. Have patient return to chair and sit down without using arms for support. a. 3, 1, 2, 5, 6, 4 b. 2, 1, 3, 5, 6, 4 c. 1, 2, 3, 6, 5, 4 d. 1, 2, 3, 5, 6, 4

a. 3, 1, 2, 5, 6, 4 These are the correct steps for performing the TGUGT.

A registered nurse performs the following four steps in delegating a task to a nursing assistant. Place the steps in the order of appropriate delegation. 1. Do you have any questions about walking Mr. Malone? 2. Before you take him for his walk to the end of the hallway and back, please take and record his pulse rate. 3. In the next 30 minutes please assist Mr. Malone in room 418 with her afternoon walk. 4. I will make sure that I check with you in about 40 minutes to see how the patient did. a. 3, 2, 4, 1 b. 2, 3, 4, 1 c. 3, 4, 2, 1 d. 3, 2, 1, 4

a. 3, 2, 4, 1 This is the sequence for effective delegation. The nurse delegated the task of walking a patient to the nursing assistant, which is in the scope of the nursing assistant's duties and responsibilities and matched to his skill level. The nurse provided clear directions by describing the task (the walk, taking and recording the pulse), the desired outcome (walk to the end of the hallway and back), and the time period (within the next 30 minutes). The nurse then told the nursing assistant that she would follow up with him to check how the patient did. By asking the nursing assistant if he had any questions, the nurse provided him an opportunity to ask questions for clarification.

Place the steps of the scientific method in their correct order with number 1 being the first step of the process. 1. Formulate a question or hypothesis. 2. Evaluate results of the study. 3. Collect data. 4. Identify the problem. 5. Test the question or hypothesis. a. 4, 3, 1, 5, 2 b. 3, 4, 1, 2, 5 c. 4, 3, 2, 1, 5 d. 3, 4, 1, 5, 2

a. 4, 3, 1, 5, 2 The correct order of the steps of the scientific method are: Identify the problem, collect data, formulate a question or hypothesis, test the question or hypothesis, and evaluate results of the test or study.

What is the correct sequence of steps when performing a wound irrigation? 1. Use slow continuous pressure to irrigate wound. 2. Attach angio catheter to syringe 3. Fill syringe with irrigation fluid 4. Place water proof bag near bed 5. Position angio catheter over wound a. 4, 3, 2, 5, 1 b. 3, 4, 2, 1, 5 c. 4, 2, 3, 5, 1 d. 2, 3, 4, 5, 1

a. 4, 3, 2, 5, 1 Organized steps ensure a safe effective irrigation of the wound.

The licensed practice nurse (LPN) provides you with the change-of-shift vital signs on four of your patients. Which patient do you need to assess first? a. 84-year-old man recently admitted with pneumonia, RR 28, SpO2 89% b. 54-year-old woman admitted after surgery for fractured arm, BP 160/86 mm Hg, HR 72 c. 63-year-old man with venous ulcers from diabetes, temperature 37.3°C (99.1°F), HR 84 d. 77-year-old woman with left mastectomy 2 days ago, RR 22, BP 148/62

a. 84-year-old man recently admitted with pneumonia, RR 28, SpO2 89% Remember ABC's when determining priorities for patient care. A patient with pneumonia and an oxygen saturation of 89 shows respiratory compromise and needs to be assessed first.

The licensed practice nurse (LPN) provides you with the change-of-shift vital signs on four of your patients. Which patient do you need to assess first? a. 84-year-old man recently admitted with pneumonia, RR 28, SpO2 89% b. 54-year-old woman admitted after surgery for fractured arm, BP 160/86 mm Hg, HR 72 c. 63-year-old man with venous ulcers from diabetes, temperature 37.3° C (99.1° F), HR 84 d. 77-year-old woman with left mastectomy 2 days ago, RR 22, BP 148/62

a. 84-year-old man recently admitted with pneumonia, RR 28, SpO2 89% SpO2 89% is a critical value and requires immediate attention. Other values require attention but are not life threatening.

Which of the following assessment findings is most representative of a stage II pressure ulcer? a. A blister b. Undermining c. Nonblanchable redness d. Visible subcutaneous fat

a. A blister Stage II ulcers have partial-thickness skin loss involving the epidermis, dermis, or both. The ulcer is superficial and presents clinically as an abrasion, blister, or shallow crater. The remaining options describe elements of stage I and stage III ulcers.

Which of the following preoperative assessment findings would most likely delay a planned procedure requiring general anesthetic? a. A cough and low-grade fever b. The pulse oximetry reading of 97% on room air c. A blood pressure that is 5 systolic points higher than baseline d. The client's report of "being so nervous about this procedure"

a. A cough and low-grade fever Preoperative assessment occasionally reveals an abnormality that delays or cancels surgery. A client who presents with a cough and low-grade fever on admission would require the nurse to notify the surgeon immediately. The other options do not necessarily warrant delay or cancellation of a procedure.

Which of the following is the best description of a hydrocolloid dressing? a. A dressing that forms a gel which interacts with the wound surface. b. A dressing containing a débriding enzyme that is used to remove necrotic tissue. c. Premoistened gauze placed over a granulating wound. d. A dressing containing a seaweed derivative that is highly absorptive.

a. A dressing that forms a gel which interacts with the wound surface. The wound contact layer of a hydrocolloid dressing forms a gel as fluid is absorbed and maintains a moist healing environment. It does not contain a débriding enzyme, a seaweed derivative, or premoistened gauze.

Subjective data include a. A patient's feelings, perceptions, and reported symptoms. b. A description of the patient's behavior. c. Observations of a patient's health status. d. Measurements of a patient's health status.

a. A patient's feelings, perceptions, and reported symptoms.

The staff on the nursing unit are discussing implementing interprofessional rounding. Which of the following statements correctly describe interprofessional rounding? a. Allows team members to share information about patients to improve care b. Provides an opportunity for early patient discharge planning c. Improves communication among health care team members d. Allows each of the health care team members to identify separate patient goals e. Allows each health care provider an opportunity to delegate a task.

a. Allows team members to share information about patients to improve care b. Provides an opportunity for early patient discharge planning c. Improves communication among health care team members Allowing team members to share information about patients to improve care, providing an opportunity for early patient discharge planning, and improving communication among team members all focus on the benefits of interprofessional rounding. This type of rounding has been found to decrease medication errors and improve quality of patient care. During interprofessional rounding all team members focus on the same patient goals.

A patient has not had a bowel movement for 4 days. Now she has nausea and severe cramping throughout her abdomen. On the basis of these findings, what should the nurse suspect? a. An intestinal obstruction b. Irritation of the intestinal mucosa c. Gastroenteritis d. A fecal impaction

a. An intestinal obstruction Absence of bowel movement, nausea, cramping, and possibly vomiting are characteristic of an intestinal obstruction.

When a nurse conducts an assessment, data about a patient often comes from which of the following sources? a. An observation of how a patient turns and moves in bed b. The unit policy and procedure manual c. The care recommendations of a physical therapist d. The results of a diagnostic x-ray film e. Your experiences in caring for other patients with similar problems

a. An observation of how a patient turns and moves in bed There are many sources of data for an assessment, including the patient through interview, observations, and physical examination; family members or significant others, health care team members such as a physical therapist, the medical record (which includes x-ray film results, and the scientific and medical literature. c. The care recommendations of a physical therapist There are many sources of data for an assessment, including the patient through interview, observations, and physical examination; family members or significant others, health care team members such as a physical therapist, the medical record (which includes x-ray film results, and the scientific and medical literature. d. The results of a diagnostic x-ray film There are many sources of data for an assessment, including the patient through interview, observations, and physical examination; family members or significant others, health care team members such as a physical therapist, the medical record (which includes x-ray film results, and the scientific and medical literature.

The nurse is assessing a client admitted with complaints related to chronic kidney dysfunction. The nurse recognizes that this client is most likely to present with which of the resulting symptoms? a. Anemia b. Hypotension c. Diabetes mellitus d. Clinical depression

a. Anemia Clients with chronic alterations in kidney function cannot produce sufficient quantities of the hormone erythropoietin; therefore they are prone to anemia. Diabetes mellitus may be a cause of the renal dysfunction, and the client may or may not be depressed. Hypertension, not hypotension, is a typical outcome of kidney dysfunction.

The client will have an incision in the lower left abdomen. Which of the following measures by the nurse will help decrease discomfort in the incisional area when the client coughs postoperatively? a. Applying a splint directly over the lower abdomen b. Keeping the client flat with her feet flexed c. Turning the client onto the right side d. Applying pressure above and below the incision

a. Applying a splint directly over the lower abdomen Deep-breathing and coughing exercises place additional stress on the suture line and cause discomfort. Splinting the incision with hands or a pillow provides firm support and reduces incisional pulling. Keeping the client flat will not decrease discomfort in the incisional area when the client coughs. Having the knees bent slightly will aid in relaxing the abdominal muscles, causing less discomfort. Turning the client onto the right side will not decrease discomfort in the incisional area when the client coughs. The client should turn from side to side at least every 2 hours and may splint the incision to decrease discomfort when doing so. Splinting should be done directly over the incision to provide firm support and reduce incisional pulling as the client coughs postoperatively.

A patient is scheduled to have an intravenous pyelogram (IVP) the next morning. Which nursing measures should be implemented prior to the test? a. Ask the patient about any allergies and reactions. b. Instruct the patient that a full bladder is required for the test. c. Instruct the patient to save all urine in a special container. d. Ensure that informed consent has been obtained. e. Explain that the test includes instrumentation of the urinary tract.

a. Ask the patient about any allergies and reactions. An intravenous pyelogram (IVP) involves intravenous injection of an iodine based contrast media. Patients that have had a previous hypersensitivity reaction to contrast media in the past are at high risk for another reaction. Informed consent is required. There is no need for a full bladder such as with a pelvic ultrasound or to save any urine for testing. There is no instrumentation of the urinary tract such as with a cystoscopy. d. Ensure that informed consent has been obtained. An intravenous pyelogram (IVP) involves intravenous injection of an iodine based contrast media. Patients that have had a previous hypersensitivity reaction to contrast media in the past are at high risk for another reaction. Informed consent is required. There is no need for a full bladder such as with a pelvic ultrasound or to save any urine for testing. There is no instrumentation of the urinary tract such as with a cystoscopy.

A 62-year-old patient had a portion of the large colon removed and a colostomy created for drainage of stool. The nurse has had repeated problems with the patient's colostomy bag not adhering to the skin and thus leaking. The nurse wants to consult with the wound care nurse specialist. Which of the following should the nurse do? a. Assess condition of skin before making the call b. Rely on the nurse specialist to know the type of surgery the patient likely had c. Explain the patient's response emotionally to the repeated leaking of stool d. Describe the type of bag being used and how long it lasts before leaking e. Order extra colostomy bags currently being used

a. Assess condition of skin before making the call The nurse should have as much information as possible available before making the call to the nurse specialist. It is also important for the nurse to interpret and explain the problem. In addition, it is important to explain the patient's perspective. Assuming that the nurse specialist knows the extent of the surgery is not appropriate. Ordering extra supplies is not a guaranteed solution that the existing bag is beneficial to the patient. c. Explain the patient's response emotionally to the repeated leaking of stool The nurse should have as much information as possible available before making the call to the nurse specialist. It is also important for the nurse to interpret and explain the problem. In addition, it is important to explain the patient's perspective. Assuming that the nurse specialist knows the extent of the surgery is not appropriate. Ordering extra supplies is not a guaranteed solution that the existing bag is beneficial to the patient. d. Describe the type of bag being used and how long it lasts before leaking The nurse should have as much information as possible available before making the call to the nurse specialist. It is also important for the nurse to interpret and explain the problem. In addition, it is important to explain the patient's perspective. Assuming that the nurse specialist knows the extent of the surgery is not appropriate. Ordering extra supplies is not a guaranteed solution that the existing bag is beneficial to the patient.

Vital signs for a patient reveal a high blood pressure of 187/100. Orders state to notify the health care provider for diastolic blood pressure greater than 90. What is the nurse's first action? a. Assess the patient for other symptoms or problems, and then notify the health care provider. b. Review the most recent lab results for the patient's potassium level. c. Follow the clinical protocol for a stroke. d. Administer an antihypertensive medication from the stock supply, and then notify the health care provider.

a. Assess the patient for other symptoms or problems, and then notify the health care provider.

The nurse is caring for a patient who requires a complex dressing change. While in the patient's room, the nurse decides to change the dressing. What does the nurse do just before changing the dressing? a. Assesses the patient's readiness for the procedure b. Gathers and organizes needed supplies c. Decides on goals and outcomes for the patient d. Calls for assistance from another nursing staff member

a. Assesses the patient's readiness for the procedure

A nurse administers an antihypertensive medication to a patient at the scheduled time of 0900. The nursing student then reports to the nurse that the patient's blood pressure was low when it was taken at 0830. The nursing nurse states she was busy and had not had a chance to tell the nurse yet. The patient begins to complain of feeling dizzy and light-headed. The blood pressure is re-checked and it has dropped even lower. The nurse first made an error in what phase of the nursing process? a. Assessment b. Diagnosis c. Planning d. Evaluation

a. Assessment

The nurse is preparing to provide a patient with instructions for how to perform incentive spirometry. The patient will likely have incisional pain after returning from an elective colon resection. Which of the following steps for incentive spirometry is the patient likely to have the most difficulty performing? a. Assuming semi-Fowler's or high-Fowler's position b. Setting the incentive spirometer device scale at the volume level to be attained c. Placing the mouthpiece of the incentive spirometer so lips completely cover the mouthpiece d. Inhaling slowly while maintaining constant flow through unit until it reaches goal volume e. Breathing normally for a short period between each of the 10 breaths on incentive spirometry f. Ending with two coughs after the end of 10 incentive spirometry breaths hourly

a. Assuming semi-Fowler's or high-Fowler's position The patient will likely have pain after surgery, making it difficult to change positions and sit upright, take a full deep breath, and be able to cough. Splinting and administration of analgesics before the use of spirometry can increase the patient's ability to perform the exercise. d. Inhaling slowly while maintaining constant flow through unit until it reaches goal volume The patient will likely have pain after surgery, making it difficult to change positions and sit upright, take a full deep breath, and be able to cough. Splinting and administration of analgesics before the use of spirometry can increase the patient's ability to perform the exercise. f. Ending with two coughs after the end of 10 incentive spirometry breaths hourly The patient will likely have pain after surgery, making it difficult to change positions and sit upright, take a full deep breath, and be able to cough. Splinting and administration of analgesics before the use of spirometry can increase the patient's ability to perform the exercise.

What best describes measurement of post-void residual (PVR)? a. Bladder scan the patient immediately after voiding. b. Catheterize the patient 30 minutes after voiding. c. Bladder scan the patient when they report a strong urge to void. d. Catheterize the patient with a 16 Fr/10 mL catheter

a. Bladder scan the patient immediately after voiding. A PVR or post void residual is the measurement of urine in the bladder within 15 minutes of normal voiding. It would not be a true measurement of PVR if the bladder was full, or if after 30 minutes of voiding. A 16 Fr/10 mL catheter and would not be appropriate to use when catheterizing for PVR.

The nurse wears a gown when: a. Blood or body fluids may get on the nurse's clothing from a task that he or she plans to perform. b. The nurse is assisting with medication administration. c. The patient has acquired immunodeficiency syndrome (AIDS) or hepatitis. d. The patient's hygiene is poor.

a. Blood or body fluids may get on the nurse's clothing from a task that he or she plans to perform.

A nurse is educating parents to look for clues in teenagers for possible substance abuse. Which environmental and psychosocial clues should the nurse include? a. Blood spots on clothing b. Long-sleeved shirts in warm weather c. Changes in relationships d. Wearing dark glasses indoors e. Increased computer use

a. Blood spots on clothing b. Long-sleeved shirts in warm weather c. Changes in relationships d. Wearing dark glasses indoors Environmental clues include the presence of drug-oriented magazines, beer and liquor bottles, drug paraphernalia and blood spots on clothing, and the continual wearing of long-sleeved shirts in hot weather and dark glasses indoors. Psychosocial clues include failing grades, change in dress, increased absenteeism from school, isolation, increased aggressiveness, and changes in interpersonal relationships.

A patient is receiving 5000 units of heparin subcutaneously every 12 hours while on prolonged bed rest to prevent thrombophlebitis. Because bleeding is a potential side effect of this medication, the nurse should continually assess the patient for the following signs of bleeding: a. Bruising b. Pale yellow urine c. Bleeding gums d. Coffee ground-like vomitus e. Light brown stool

a. Bruising Because bleeding is a potential side effect of these medications, continually assess the patient for signs of bleeding such as hematuria, bruising, coffee ground-like vomitus or gastrointestinal aspirate, guaiac-positive stools, and bleeding gums. c. Bleeding gums Because bleeding is a potential side effect of these medications, continually assess the patient for signs of bleeding such as hematuria, bruising, coffee ground-like vomitus or gastrointestinal aspirate, guaiac-positive stools, and bleeding gums. d. Coffee ground-like vomitus Because bleeding is a potential side effect of these medications, continually assess the patient for signs of bleeding such as hematuria, bruising, coffee ground-like vomitus or gastrointestinal aspirate, guaiac-positive stools, and bleeding gums.

The posterior hypothalamus helps control temperature by a. Causing vasoconstriction. b. Shunting blood to the skin and extremities. c. Increasing sweat production. d. Causing vasodilation.

a. Causing vasoconstriction

A nurse is charting on a patient's record. Which action is most accurate legally? a. Charts legibly b. States the patient is belligerent c. Uses correction fluid to correct error d. Writes entry for another nurse

a. Charts legibly

When working with an older adult who is hearing-impaired, the use of which techniques would improve communication? (Select all that apply) a. Check for needed adaptive equipment. b. Exaggerate lip movements to help the patient lip read. c. Give the patient time to respond to questions. d. Keep communication short and to the point. e. Communicate only through written information.

a. Check for needed adaptive equipment. c. Give the patient time to respond to questions. d. Keep communication short and to the point. Communication techniques such as assessing the need for adaptive equipment, keeping communication short and direct, and giving the patient time to respond help the nurse provide clear effective communication. Patients may have difficulty with rapid or lengthy explanations. Exaggerated lip reading may be difficult or demeaning to individuals with hearing deficits.

A nurse is evaluating a patient who is in soft wrist restraints. Which of the following activities does the nurse perform? a. Check the patient's peripheral pulse in the restrained extremity b. Evaluate the patient's need for toileting c. Offer the patient fluids if appropriate d. Release both limbs at the same time to perform range of motion (ROM) e. Inspect the skin under each restraint

a. Check the patient's peripheral pulse in the restrained extremity b. Evaluate the patient's need for toileting c. Offer the patient fluids if appropriate e. Inspect the skin under each restraint The nurse should evaluate patient for signs of injury every 15 minutes e.g., circulation, vital signs, ROM, physical and psychological status, and readiness for discontinuation. The nurse should evaluate patient's need for toileting, nutrition and fluids, hygiene, and elimination and release restraint at least every 2 hours but should do it one limb at a time.

A nurse checks an intravenous (IV) solution container for clarity of the solution, noting that it is infusing into the patient's left arm. The IV solution of 9% NS is infusing freely at 100 mL/hr as ordered. The nurse reviews the nurses' notes from the previous shift to determine if the dressing over the site was changed as scheduled per standard of care. While in the room the nurse inspects the condition of the dressing and notes the date on the dressing label. In which ways did the nurse evaluate the condition of the IV site? a. Checked the IV infusion rate b. Checked the type of IV solution c. Confirmed from nurses' notes the time of dressing change d. Inspected the condition of the IV dressing at the site e. Checked clarity of IV solution

a. Checked the IV infusion rate d. Inspected the condition of the IV dressing at the site The condition or status of the IV site is determined by checking the IV infusion rate and the condition of the IV site dressing. Checking the type of solution is important to ensure that correct therapy is being administered but is not a measure of the IV site condition. Confirming a dressing change or the appearance of the IV solution is not an indicator of the IV site status.

Which measures does a nurse follow when being asked to perform an unfamiliar procedure? a. Checks scientific literature or policy and procedure b. Reassesses the patient's condition c. Collects all necessary equipment d. Delegates the procedure to a more experienced nurse e. Considers all possible consequences of the procedure

a. Checks scientific literature or policy and procedure The nurse does not delegate a procedure to a more experienced nurse. Instead the nurse has another nurse (e.g., staff nurse, faculty, nurse specialist) who has completed the procedure correctly and safely provide assistance and guidance. b. Reassesses the patient's condition The nurse does not delegate a procedure to a more experienced nurse. Instead the nurse has another nurse (e.g., staff nurse, faculty, nurse specialist) who has completed the procedure correctly and safely provide assistance and guidance. c. Collects all necessary equipment The nurse does not delegate a procedure to a more experienced nurse. Instead the nurse has another nurse (e.g., staff nurse, faculty, nurse specialist) who has completed the procedure correctly and safely provide assistance and guidance. e. Considers all possible consequences of the procedure The nurse does not delegate a procedure to a more experienced nurse. Instead the nurse has another nurse (e.g., staff nurse, faculty, nurse specialist) who has completed the procedure correctly and safely provide assistance and guidance.

The nurse is to collect a specimen for culture after assessing the client's wound drainage. The best technique for obtaining the culture is to: a. Cleanse the wound first. b. Send the soiled dressing to the laboratory. c. Swab from the outside skin edge inward. d. Collect the specimen from accumulated drainage.

a. Cleanse the wound first. Cleansing the wound first and swabbing the granulation tissue will provide a culture specimen that will show a more accurate picture of any causative organisms of wound infection. Sending a soiled dressing and collecting a specimen from accumulated drainage are not appropriate, because old and new drainage are mingled, and the drainage is possibly growing organisms of its own and may not provide a true reflection of the wound flora. Swabbing from the outer edge of the skin inward may introduce organisms into the wound and contaminate the culture specimen.

The primary reason that family members should be included when a nurse teaches the patient preoperative exercises is so they can: a. Coach and encourage the patient after surgery. b. Demonstrate to the patient at home. c. Relieve the nurse by getting the patient to do the exercises every 2 hours. d. Practice with the patient while he or she is waiting to be taken to the operating room.

a. Coach and encourage the patient after surgery. Patients may need support from family to be motivated to return to their previous state of health. The family may also have better retention of preoperative teaching and will be with the patient and able to help him or her in recovery.

Before consulting with a physician about a female patient's need for urinary catheterization, the nurse considers the fact that the patient has urinary retention and has been unable to void on her own. The nurse knows that evidence for alternative measures to promote voiding exists, but none has been effective, and that before surgery the patient was voiding normally. This scenario is an example of which implementation skill? a. Cognitive b. Interpersonal c. Psychomotor d. Consultative

a. Cognitive This is an example of a cognitive skill, being used before consultation. It involves critical thinking and decision making so the nurse is able to deliver a relevant nursing intervention.

Which strategies should a nurse use to facilitate a safe transition of care during a patient's transfer from the hospital to a skilled nursing facility? a. Collaboration between staff members from sending and receiving departments b. Requiring that the patient visit the facility before a transfer is arranged c. Using a standardized transfer policy and transfer tool d. Arranging all patient transfers during the same time each day e. Relying on family members to share information with the new facility

a. Collaboration between staff members from sending and receiving departments c. Using a standardized transfer policy and transfer tool Providing a standardized process, policy, and tool can assist in a predictable, safe transfer of important patient information between health care facilities. Communication and collaboration between the sender and receiver of information enable the staff to validate that information was received and understood. Requiring a patient visit is not always necessary, and relying on family members to share information does not release staff from their responsibilities.

Which of the following examples are steps of nursing assessment? a. Collection of information from patient's family members b. Recognition that further observations are needed to clarify information c. Comparison of data with another source to determine data accuracy d. Complete documentation of observational information e. Determining which medications to administer based on a patient's assessment data

a. Collection of information from patient's family members Assessment includes collection of data from secondary sources such as the patient's family. Recognizing that more observation is needed is an example of validation of data. Comparing data to determine accuracy is a feature of interpretation. Although complete documentation is an important step in communicating assessment data, it is not an assessment step. b. Recognition that further observations are needed to clarify information Assessment includes collection of data from secondary sources such as the patient's family. Recognizing that more observation is needed is an example of validation of data. Comparing data to determine accuracy is a feature of interpretation. Although complete documentation is an important step in communicating assessment data, it is not an assessment step. c. Comparison of data with another source to determine data accuracy Assessment includes collection of data from secondary sources such as the patient's family. Recognizing that more observation is needed is an example of validation of data. Comparing data to determine accuracy is a feature of interpretation. Although complete documentation is an important step in communicating assessment data, it is not an assessment step.

The nurse has received a report from the emergency department that a patient with tuberculosis will be coming to the unit. What items will the nurse need to care for this patient? (Select all that apply.) a. Communication signs for Airborne Precautions b. Negative-pressure airflow in room c. Surgical mask, gown, gloves, eyewear d. N95 respirator, gown, gloves, eyewear e. Communication signs for Droplet Precautions f. Private room

a. Communication signs for Airborne Precautions b. Negative-pressure airflow in room d. N95 respirator, gown, gloves, eyewear f. Private room

A nurse has been caring for a patient over the last 10 hours. The patient's plan of care includes the nursing diagnosis of Nausea related to effects of postoperative anesthesia. The nurse has been asking the patient to rate his nausea over the last several hours after administering antiemetics and using comfort measures such as oral hygiene. The nurse reviews the patient's responses over the past 10 hours and notes how the patient's self-report of nausea has changed. This review an example of: a. Comparing outcome criteria with actual response. b. Gathering outcome criteria. c. Evaluating the patient's actual response. d. Reprioritizing interventions.

a. Comparing outcome criteria with actual response. The key to this question is observation for change. The nurse compares the patient's actual self-report rating of nausea with the expected outcome of a reduction in nausea. Gathering outcome criteria simply involves having the patient rate nausea. Evaluating the behavior or self-report is the determination of the patient's actual response.

During the assessment phase of the nursing process ensures that the nurse a. Completes a comprehensive database. b. Identifies pertinent nursing diagnoses. c. Intervenes based on patient goals and priorities of care. d. Determines whether outcomes have been achieved.

a. Completes a comprehensive database.

Which of the following does a nurse perform when discontinuing a plan of care for a patient? a. Confirms with the patient that expected outcomes and goals have been met b. Talks with the patient about reprioritizing interventions in the plan of care c. Changes the frequency of interventions provided d. Reassesses how goals were met

a. Confirms with the patient that expected outcomes and goals have been met When you discontinue a plan of care, you determine that expected outcomes and goals have been met, and you confirm this evaluation with the patient when possible. If you and the patient agree, you discontinue that part of the care plan. Reassessing how goals were met is not necessary if you confirm that discontinuation of a plan is appropriate. Talking with the patient about reprioritizing interventions in the plan of care and changing the frequency of the interventions provided are not appropriate when a plan is discontinued.

A nurse on a busy medicine unit is assigned to four patients. It is 10 am. Two patients have medications due and one of those has a specimen of urine to be collected. One patient is having complications from surgery and is being prepared to return to the operating room. The fourth patient requires instructions about activity restrictions before going home this afternoon. Which of the following should the nurse use in making clinical decisions appropriate for the patient group? a. Consider availability of assistive personnel to obtain the specimen b. Combine activities to resolve more than one patient problem c. Analyze the diagnoses/problems and decide which are most urgent based on patients' needs d. Plan a family conference for tomorrow to make decisions about resources the patient will need to go home e. Identify the nursing diagnoses for the patient going home

a. Consider availability of assistive personnel to obtain the specimen Analyzing urgency of problems helps to prioritize as does considering the resources that are available (such as assistive personnel) to complete patient care activities. Deciding how to combine activities is good time management. Holding a family conference is a good idea but in this case would be too late to be beneficial to the patient. To determine priorities the nurse must identify nursing diagnoses for all patients. b. Combine activities to resolve more than one patient problem Analyzing urgency of problems helps to prioritize as does considering the resources that are available (such as assistive personnel) to complete patient care activities. Deciding how to combine activities is good time management. Holding a family conference is a good idea but in this case would be too late to be beneficial to the patient. To determine priorities the nurse must identify nursing diagnoses for all patients. c. Analyze the diagnoses/problems and decide which are most urgent based on patients' needs Analyzing urgency of problems helps to prioritize as does considering the resources that are available (such as assistive personnel) to complete patient care activities. Deciding how to combine activities is good time management. Holding a family conference is a good idea but in this case would be too late to be beneficial to the patient. To determine priorities the nurse must identify nursing diagnoses for all patients.

The nurse is caring for a patient whose calcium intake must increase because of high risk factors for osteoporosis. Which of the following menus should the nurse recommend? a. Cream of broccoli soup with whole wheat crackers, cheese, and tapioca for dessert b. Hot dog on whole wheat bun with a side salad and an apple for dessert c. Low-fat turkey chili with sour cream with a side salad and fresh pears for dessert d. Turkey salad on toast with tomato and lettuce and honey bun for dessert

a. Cream of broccoli soup with whole wheat crackers, cheese, and tapioca for dessert Teach patient and/or caregiver the current recommended dietary allowances for calcium and review foods high in calcium (e.g., milk fortified with vitamin D, leafy green vegetables, yogurt, and cheese).

The nurse is caring for a patient who has been diagnosed with a stroke. As part of her ongoing care, the nurse should a. Encourage the patient to perform as many self-care activities as possible. b. Provide a complete bed bath to promote patient comfort. c. Place the patient on bed rest to prevent fatigue. d. Understand that the patient will not eat owing to a decreased energy need.

a. Encourage the patient to perform as many self-care activities as possible.

A nurse reviews all possible consequences before helping a patient ambulate such as how the patient ambulated last time; how mobile the patient was before admission to the health care facility; or any current clinical factors affecting the patient's ability to stand, remain balanced, or walk. Which of the following is an example of a nurse's review of this situation? a. Critical thinking b. Managing an adverse event c. Exercising self-discipline d. Time management

a. Critical thinking The process of reviewing consequences for a patient is an example of critical thinking and clinical decision making. Managing an adverse event occurs after consequences have occurred. Exercising self-discipline is a critical thinking attitude that guides you in reviewing, modifying, and implementing interventions, which occurs after reviewing consequences. This is not an example of time management.

A healthy adult patient tells the nurse that he obtained his blood pressure in "one of those quick machines in the mall" and was alarmed that it was 152/72 when his normal value ranges from 114/72 to 118/78. The nurse obtains a blood pressure of 116/76. What would account for the blood pressure of 152/92? a. Cuff too small b. Arm positioned above heart level c. Slow inflation of the cuff by the machine d. Patient did not remove his long-sleeved shirt e. Insufficient time between measurements

a. Cuff too small Using too small of a cuff and not allowing for insufficient time between measurements will result in false-high readings. Arm above heart level and slow inflation result in false low readings. e. Insufficient time between measurements Using too small of a cuff and not allowing for insufficient time between measurements will result in false-high readings. Arm above heart level and slow inflation result in false low readings.

A nurse is caring for patients with a variety of wounds. Which would will most likely heal by primary intention? a. Cut in the skin from a kitchen knife b. Excoriated perineal area c. Abrasion of the skin d. Pressure ulcer

a. Cut in the skin from a kitchen knife A cut in the skin from a kitchen knife; A cut in the skin by a sharp instrument with minimal tissue loss can heal by primary intention when the wound edges are lightly pulled together (approximated).Excoriations, abrasions, and pressure ulcers heal by secondary, not primary. Secondary intention healing occurs when wound edges are not approximated because of full-thickness tissue loss; the wound is left open until it fills with new tissue. Abrasions and excoriations are injuries to the surface of the skin.

Scaling of the scalp accompanied by itching is known as a. Dandruff b. Pedicilosis c. Alopecia d. Ticks

a. Dandruff

A nurse reviews data gathered regarding a patient's ability to cope with loss. The nurse compares the defining characteristics for Ineffective Coping with those for Readiness for Enhanced Coping and selects Ineffective Coping as the correct diagnosis. This is an example of the nurse avoiding an error in: a. Data collection. b. Data clustering. c. Data interpretation. d. Making a diagnostic statement. e. Goal setting.

a. Data collection. This is an example of an error in interpretation and data collection. When making a diagnosis, the nurse must interpret data that he or she has collected by identifying and organizing relevant assessment patterns to support the presence of patient problems. In the case of the two diagnoses in this question, there can be conflicting cues. The nurse must obtain more information and recognize the cues that point to the correct diagnosis. c. Data interpretation. This is an example of an error in interpretation and data collection. When making a diagnosis, the nurse must interpret data that he or she has collected by identifying and organizing relevant assessment patterns to support the presence of patient problems. In the case of the two diagnoses in this question, there can be conflicting cues. The nurse must obtain more information and recognize the cues that point to the correct diagnosis.

What is the removal of devitalized tissue from a wound called? a. Debridement b. Pressure reduction c. Negative pressure wound therapy d. Sanitization

a. Debridement Debridement is the removal of nonliving tissue, cleaning the wound to move toward healing.

A patient has been on bed rest for over 4 days. On assessment, the nurse identifies the following as a sign associated with immobility: a. Decreased peristalsis b. Decreased heart rate c. Increased blood pressure d. Increased urinary output

a. Decreased peristalsis Immobility disrupts normal metabolic functioning: decreasing the metabolic rate; altering the metabolism of carbohydrates, fats, and proteins; causing fluid, electrolyte, and calcium imbalances; and causing gastrointestinal disturbances such as decreased appetite and slowing of peristalsis.

The use of standard formal nursing diagnostic statements serves several purposes in nursing practice, including which of the following? (Select all that apply) a. Defines a patient's problem, giving members of the health care team a common language for understanding the patient's needs b. Allows physicians and allied health staff to communicate with nurses how they provide care among themselves c. Helps nurses focus on the scope of nursing practice d. Creates practice guidelines for collaborative health care activities e. Builds and expands nursing knowledge

a. Defines a patient's problem, giving members of the health care team a common language for understanding the patient's needs The use of nursing diagnosis creates a common language for nurses to communicate patient care needs, allows nurses to focus on the realm and scope of nursing practice, and helps to develop nursing knowledge. It is not a language for physicians and allied health staff because they do not rely on providing nursing interventions. Terminology in nursing diagnosis may be familiar to other health care providers but not in a way for directing nursing interventions. Nursing diagnosis has the purpose of creating practice guidelines for nursing. c. Helps nurses focus on the scope of nursing practice The use of nursing diagnosis creates a common language for nurses to communicate patient care needs, allows nurses to focus on the realm and scope of nursing practice, and helps to develop nursing knowledge. It is not a language for physicians and allied health staff because they do not rely on providing nursing interventions. Terminology in nursing diagnosis may be familiar to other health care providers but not in a way for directing nursing interventions. Nursing diagnosis has the purpose of creating practice guidelines for nursing. e. Builds and expands nursing knowledge The use of nursing diagnosis creates a common language for nurses to communicate patient care needs, allows nurses to focus on the realm and scope of nursing practice, and helps to develop nursing knowledge. It is not a language for physicians and allied health staff because they do not rely on providing nursing interventions. Terminology in nursing diagnosis may be familiar to other health care providers but not in a way for directing nursing interventions. Nursing diagnosis has the purpose of creating practice guidelines for nursing.

A nurse is caring for an older adult who has had a fractured hip repaired. In the first few postoperative days, which of the following nursing measures will best facilitate the resumption of activities of daily living for this patient? a. Encouraging use of an overhead trapeze for positioning and transfer. b. Frequent family visits c. Assisting the patient to a wheelchair once per day d. Ensuring that there is an order for physical therapy

a. Encouraging use of an overhead trapeze for positioning and transfer. The trapeze bar allows the patient to pull with the upper extremities to raise the trunk off the bed, aid in transfer from bed to wheelchair, or perform upper-arm exercises. It increases independence and maintains upper body strength to help in performing activities of daily living.

A nurse has provided care to a patient. Which entry should the nurse document in the patient's record? a. "Patient seems to be in pain and states, 'I feel uncomfortable.'" b. Status unchanged, doing well c. Left abdominal incision 1 inch in length without redness, drainage, or edema d. Patient is hard to care for and refuses all treatments and medications. Family present.

c. Left abdominal incision 1 inch in length without redness, drainage, or edema

An 88-year-old patient comes to the medical clinic regularly. During a recent visit the nurse noticed that the patient had lost 10 lbs in 6 weeks without being on a special diet. The patient tells the nurse that he has had trouble chewing his food. Which of the following factors are normal aging changes that can affect an older adult's oral health? a. Dentures do not always fit properly. b. Most older adults have an increase in saliva secretions. c. With aging the periodontal membrane becomes tighter and painful. d. Many older adults are edentulous, and remaining teeth are often decayed. e. The teeth of elderly patients are more sensitive to hot and cold.

a. Dentures do not always fit properly. Dentures or partial plates do not always fit properly, causing pain and discomfort. Many older adults are edentulous (without teeth), and the teeth that are present are often diseased or decayed. An age-related decline in saliva secretion is common. The periodontal membrane weakens with aging, making the area prone to infection. Normally aging does not affect temperature sensitivity. d. Many older adults are edentulous, and remaining teeth are often decayed. Dentures or partial plates do not always fit properly, causing pain and discomfort. Many older adults are edentulous (without teeth), and the teeth that are present are often diseased or decayed. An age-related decline in saliva secretion is common. The periodontal membrane weakens with aging, making the area prone to infection. Normally aging does not affect temperature sensitivity.

Before implementing any intervention, the nurse uses critical thinking to a. Determine whether an intervention is correct and appropriate for the given situation. b. Evaluate the effectiveness of interventions. c. Establish goals for a particular patient without the need for reassessment. d. Read over the steps and perform a procedure despite lack of clinical competency.

a. Determine whether an intervention is correct and appropriate for the given situation.

Nurses who make the best communicators a. Develop critical thinking skills. b. Like different kinds of people. c. Learn effective psychomotor skills. d. Maintain perceptual biases.

a. Develop critical thinking skills.

During a patient's routine annual physical, she tells you that she has noted that her heart feels like it's " racing, " usually in the later morning, early afternoon, or just before she goes to bed. Her radial pulse rate is 68 beats/min and regular; her blood pressure is 134/82 mm Hg. What additional information is helpful in evaluating the patient's racing heart? (Select all that apply.) a. Dietary habits b. Medication list c. Exercise regimen d. Age, weight, and height

a. Dietary habits b. Medication list Dietary habits and medications can affect a persons heart rate. eg. caffeine and albuterol.

Which type of personal protective equipment are staff required to wear when caring for a pediatric patient who is placed into airborne precautions for confirmed chicken pox/herpes zoster? a. Disposable gown b. N 95 respirator mask c. Face shield or goggles d. Surgical mask e. Gloves

a. Disposable gown The organism is dispersed into the air and is light enough to stay afloat for long periods of time. Wearing protective covering and the filtration mask or hood prevents the staff from breathing in the infected air particles. b. N 95 respirator mask The organism is dispersed into the air and is light enough to stay afloat for long periods of time. Wearing protective covering and the filtration mask or hood prevents the staff from breathing in the infected air particles. e. Gloves The organism is dispersed into the air and is light enough to stay afloat for long periods of time. Wearing protective covering and the filtration mask or hood prevents the staff from breathing in the infected air particles.

The nurse is supervising a beginning nursing student and allowing the student to complete documentation of care under direct observation. Which of the following actions are not appropriate and would require intervention? The nursing student: a. Documents a medication given by another nursing student. b. Includes the date and time of the entry into the medical record. c. Enters assessment data into the electronic medical record using the computer mounted on the wall in the patient's room. d. Leaves a slip of paper with her user name and password in the patient's room. e. Starts to enter "Docusate sodium 100 mg ordered at 08:00 held. Patient declined to take dose stating, "I had several loose stools yesterday, and I'm afraid if I take this dose the problem will get worse," as a narrative comment.

a. Documents a medication given by another nursing student. d. Leaves a slip of paper with her user name and password in the patient's room. Nurses only document the care they provide; entries in the chart need to be dated, timed, and signed. Personal passwords used to access an electronic medical record need to be kept secure to provide for safety and confidentiality of patient information. All of the other actions are appropriate for documentation.

You are conducting an education class at a local senior center on safe-driving tips for seniors. Which of the following should you include? a. Drive shorter distances b. Drive only during daylight hours c. Use the side and rearview mirrors carefully d. Keep a window rolled down while driving if has trouble hearing e. Look behind toward the blind spot f. Stop driving at age 75

a. Drive shorter distances b. Drive only during daylight hours c. Use the side and rearview mirrors carefully d. Keep a window rolled down while driving if has trouble hearing e. Look behind toward the blind spot Educate patients regarding safe driving tips (e.g., driving shorter distances or only in daylight, using side and rearview mirrors carefully, and looking behind them toward their "blind spot" before changing lanes). If hearing is a problem, encourage the patient to keep a window rolled down while driving or reduce the volume of the radio or CD player. Counseling is often necessary to help older patients make the decision of when to stop driving.

Which of the following statements should the nurse use to instruct the nursing assistant caring for a client with an indwelling urinary catheter? a. Empty the drainage bag at least every 8 hours. b. Clean up the length of the catheter to the perineum. c. Use clean technique to obtain a specimen for culture and sensitivity. d. Place the drainage bag on the client's lap while transporting the client to testing.

a. Empty the drainage bag at least every 8 hours. The urinary drainage bag should be emptied at least every 8 hours. If large outputs are noted, more frequent emptying will be required. The perineum should be cleansed and then down the catheter for a length of approximately 10 cm (4 inches). Only use sterile technique to collect specimens from a closed drainage system. Avoid raising the drainage bag above the level of the bladder. If it becomes necessary to raise the bag during transfer of the client to a bed or stretcher, clamp the tubing or empty the tubing contents to the drainage bag first. The drainage bag can be attached to the wheelchair below the level of the client's bladder for transport. It should not be placed on the client's lap.

A nursing student knows that all patients should be ambulated regularly. The patient to which she is assigned has had reduced activity tolerance. She followed orders to ambulate the patient twice during the shift of care. In what way can the nursing student make the goal of improving the patient's activity tolerance a patient-centered effort? a. Engage the patient in setting mutual outcomes for distance he is able to walk b. Confirm with the patient's health care provider about ambulation goals c. Have physical therapy assist with ambulation d. Refer to medical record regarding nature of patient's physical problem

a. Engage the patient in setting mutual outcomes for distance he is able to walk All goals and outcomes of care should be patient centered whenever possible. An approach for ensuring patient centered goals is having the patient involved so goals can be mutually set and realistic to the patient. Confirming with the physician and checking the medical record help the nurse understand the extent of exercise in which a patient can participate. But these approaches are not examples of mutual patient-centered goal setting. Having physical therapy assistance would not make a goal patient centered.

Which of the following statements correctly describes the evaluation process? a. Evaluation is an ongoing process. b. Evaluation usually reveals obvious changes in patients. c. Evaluation involves making clinical decisions. d. Evaluation requires the use of assessment skills. e. Evaluation is only done when a patient's condition changes.

a. Evaluation is an ongoing process. c. Evaluation involves making clinical decisions. d. Evaluation requires the use of assessment skills. Evaluation is ongoing throughout the nursing process once nursing diagnoses or patient health problems have been identified. It is a process that involves clinical decision making and use of assessment skills as evaluative measures. Evaluation may reveal changes in patients that often are not obvious. It occurs after any intervention and not only when a patient's condition changes.

A patient has been febrile and coughing thick secretions; adventitious lung sounds indicate rales in the left lower lobe of the lungs. The nurse decides to perform nasotracheal suction because the patient is not coughing. The nurse inspects the mucus that is suctioned, which is minimal. The nurse again auscultates for lung sounds. Auscultation and mucus inspection are examples of: a. Evaluative measures. b. Expected outcomes. c. Reassessments. d. Reflection.

a. Evaluative measures. Auscultation of lung sounds and inspection of mucus after the intervention of suctioning are examples of evaluative measures. An outcome would be clear secretions or clear lung sounds. It is not a reassessment because the nurse has not yet compared findings with expected outcomes. Suctioning is a standard of care.

A nurse has been caring for a patient over 2 consecutive days. During that time the patient has had an intravenous (IV) catheter in the right forearm. At the end of shift on the second day the nurse inspects the catheter site, observes for redness, and asks if the patient feels tenderness when the site is palpated. This is an example of which indicator reflecting the nurse's ability to perform evaluation: a. Examining results of clinical data b. Comparing achieved effects with outcomes c. Recognizing error d. Self-reflection

a. Examining results of clinical data Examination of the IV site is an example of examining results of clinical data. The nurse will next take the results of the examination and compare them to the norms for a normal IV site to decide if the outcome of maintaining a site free of infection is achieved. No errors were identified in this example, and the nurse is not self-reflecting.

A male nurse is caring for a 32-year-old female Muslim patient who has an indwelling Foley catheter. After introducing himself to the patient, the nurse learns that the patient does not want him to help her with personal hygiene care. Which of the following is(are) appropriate actions? (Select all that apply.) a. Finding a female nurse to help the patient b. Convincing the patient that he will work quickly and provide as much privacy as possible c. Skipping hygiene care for the day except for the parts that the patient can complete independently d. Asking the patient if she prefers a family member assist with the care

a. Finding a female nurse to help the patient d. Asking the patient if she prefers a family member assist with the care

Which skin care measures are used to manage a patient who is experiencing fecal and/or urinary incontinence? a. Frequent position changes. b. Keeping the buttocks exposed to air at all times c. Using a large absorbent diaper, changing when saturated d. Using an incontinence cleaner e. Frequent cleaning, applying an ointment, and covering the areas with a thick absorbent towel f. Applying a moisture barrier ointment

a. Frequent position changes. Skin that is in contact with stool and urine can become moist and soft, allowing it to become damaged. The stool contains bacteria and in some cases enzymes that can harm the skin if in contact for a prolonged period of time. The use of an incontinence cleaner provides a gentle removal of stool and urine, and the use of the moisture-barrier ointment provides a protective layer between the skin and the next incontinence episode. However, skin care and moisture barriers must also be used with frequent position changes to assist in reducing the risk for pressure ulcers. d. Using an incontinence cleaner Skin that is in contact with stool and urine can become moist and soft, allowing it to become damaged. The stool contains bacteria and in some cases enzymes that can harm the skin if in contact for a prolonged period of time. The use of an incontinence cleaner provides a gentle removal of stool and urine, and the use of the moisture-barrier ointment provides a protective layer between the skin and the next incontinence episode. However, skin care and moisture barriers must also be used with frequent position changes to assist in reducing the risk for pressure ulcers. f. Applying a moisture barrier ointment Skin that is in contact with stool and urine can become moist and soft, allowing it to become damaged. The stool contains bacteria and in some cases enzymes that can harm the skin if in contact for a prolonged period of time. The use of an incontinence cleaner provides a gentle removal of stool and urine, and the use of the moisture-barrier ointment provides a protective layer between the skin and the next incontinence episode. However, skin care and moisture barriers must also be used with frequent position changes to assist in reducing the risk for pressure ulcers.

When assessing a patient's feet, the nurse notices that the toenails are thick and separated from the nail bed. The nurse is aware that this condition is caused by a. Fungi b. Nail polish c. Friction d. Nail polish remover

a. Fungi

Motivational interviewing (MI) is a technique that applies understanding a patient's values and goals in helping the patient make behavior changes. What are other benefits of using MI techniques? a. Gaining an understanding of patient's motivations b. Focusing on opportunities to avoid poor health choices c. Recognizing patient's strengths and supporting their efforts d. Providing assessment data that can be shared with families to promote change e. Identifying differences in patient's health goals and current behaviors

a. Gaining an understanding of patient's motivations c. Recognizing patient's strengths and supporting their efforts e. Identifying differences in patient's health goals and current behaviors MI is a technique used to promote an understanding of patients' motivation, health goals, and current behaviors in a nonjudgmental environment while focusing on the patient's strengths and efforts. The nurse provides a supportive approach to help the patient establish and promote positive health care changes.

Based on Mr. Johnson's vital signs(Heart rate = 72 beats per minute Respiration rate = 28 breaths per minute Blood pressure = 135/87 mmHg. Temperature = 37.2° C (99° F) Oral), you expect the nurse to: a. administer oxygen via nasal cannula. b. administer the ordered Tylenol (Acetaminophen). c. hold Mr. Johnson's next BP medication dose. d. document the vital signs as normal.

a. administer oxygen via nasal cannula Tylenol is not indicated since there is no fever.Holding BP medication is not indicated since BP is not low.Documentation is not enough, an action is indicated due to high RR.

What is your role as a nurse during a fire? a. Help to evacuate patients b. Shut off medical gases c. Use a fire extinguisher d. Single carry patients out e. Direct ambulatory patients

a. Help to evacuate patients b. Shut off medical gases c. Use a fire extinguisher e. Direct ambulatory patients Direct all ambulatory patients to walk by themselves to a safe area. If you have to carry a patient, do so correctly (e.g., two-man carry). After a fire is reported and patients are out of danger, nurses and other personnel take measures to contain or extinguish it such as closing doors and windows, placing wet towels along the base of doors, turning off sources of oxygen and electrical equipment, and using a fire extinguisher.

A registered nurse (RN) is providing care to a patient who had abdominal surgery 2 days ago. Which task is appropriate to delegate to the nursing assistant? a. Helping the patient ambulate in the hall b. Changing surgical wound dressing c. Irrigating the nasogastric tube d. Providing brochures to the patient on health diet

a. Helping the patient ambulate in the hall Helping the patient with activity is within the scope of nursing assistive duties. The other activities require the skill and knowledge of the RN.

The nurse is caring for an unresponsive patient who has a nasogastric tube in place for continuous tube feedings. The nurse assesses the patient's oral hygiene because good oral hygiene a. Helps prevent gingivitis. b. May cause glossitis. c. May lead to halitosis. d. Causes tongue coating.

a. Helps prevent gingivitis.

The patient is admitted with shortness of breath and chest discomfort. Which of the following laboratory values could account for the patient's symptoms? a. Hemoglobin level of 8.0 b. Hematocrit level of 45% c. Red blood cell count of 5.0 million/mm3 d. Pulse oximetry of 90%

a. Hemoglobin level of 8.0 decrease hemoglobin levels can limit the oxygen carrying capacity of RBC's and cause shortness of breath and chest discomfort related to hypoxemia.

A client who is scheduled for surgery is found to have thrombocytopenia. A specific postoperative concern for the nurse for this client is: a. Hemorrhage b. Wound infection c. Fluid imbalance d. Respiratory depression

a. Hemorrhage A client with thrombocytopenia is at risk for hemorrhaging during and after surgery. Clients with immunological disorders or diabetes mellitus have an increased risk for wound infection after surgery. A client who has a fever is at risk for fluid imbalance. A client who has chronic respiratory disease may be at increased risk for respiratory depression, not the client with thrombocytopenia.

The nurse is changing linens for a postoperative patient and feels a stick in her hand. A nonactivated safe needle is noted in the linens and has punctured the nurse's skin. This scenario would indicate that the nurse may be at risk for a. Hepatitis B. b. Diphtheria. c. Clostridium difficile. d. Methicillin-resistant Staphylococcus aureus.

a. Hepatitis B. A needle stick injury puts the healthcare provider at risk for a blood infection from a contaminated needle. Hepatitis B is the only blood transmitted infection.

A nurse is getting ready to assess a patient in a neighborhood community clinic. He was newly diagnosed with diabetes just a month ago. He has other health problems and a history of not being able to manage his health. Which of the following questions reflects the nurse's cultural competence in making an accurate diagnosis? a. How is your diabetic diet affecting you and your family? b. You seem to not want to follow health guidelines. Can you explain why? c. What worries you the most about having diabetes? d. What do you expect from us when you do not take your insulin as instructed? e. What do you believe will help you control your blood sugar?

a. How is your diabetic diet affecting you and your family? Asking "How is your diabetic diet affecting you and your family?" "What worries you the most about having diabetes?" and "What do you believe will help you control your blood sugar?" are open-ended and allow the patient to share his values and health practices. The statements "You seem to not want to follow health guidelines. Can you explain why?" and "What do you expect from us when you do not take your insulin as instructed?" both show the nurse's bias. c. What worries you the most about having diabetes? Asking "How is your diabetic diet affecting you and your family?" "What worries you the most about having diabetes?" and "What do you believe will help you control your blood sugar?" are open-ended and allow the patient to share his values and health practices. The statements "You seem to not want to follow health guidelines. Can you explain why?" and "What do you expect from us when you do not take your insulin as instructed?" both show the nurse's bias. e. What do you believe will help you control your blood sugar? Asking "How is your diabetic diet affecting you and your family?" "What worries you the most about having diabetes?" and "What do you believe will help you control your blood sugar?" are open-ended and allow the patient to share his values and health practices. The statements "You seem to not want to follow health guidelines. Can you explain why?" and "What do you expect from us when you do not take your insulin as instructed?" both show the nurse's bias.

Which skills must a patient with a new colostomy be taught before discharge from the hospital? a. How to change the pouch b. How to empty the pouch c. How to open and close the pouch d. How to irrigate the colostomy e. How to determine if the ostomy is healing appropriately

a. How to change the pouch The patient must be able to do these tasks to successfully manage his or her colostomy when going home. b. How to empty the pouch The patient must be able to do these tasks to successfully manage his or her colostomy when going home. c. How to open and close the pouch The patient must be able to do these tasks to successfully manage his or her colostomy when going home. e. How to determine if the ostomy is healing appropriately The patient must be able to do these tasks to successfully manage his or her colostomy when going home.

A nurse needs to teach a young woman newly diagnosed with asthma how to manage her disease. Which of the following topics does the nurse teach first? a. How to use an inhaler during an asthma attack b. The need to avoid people who smoke to prevent asthma attacks c. Where to purchase a medical alert bracelet that says she has asthma d. The importance of maintaining a healthy diet and exercising regularly

a. How to use an inhaler during an asthma attack It is important to start with essential lifesaving information when teaching people because they usually remember what you tell them first.

A nurse has taught the staff about informatics. Which statement indicates that the staff needs more education? a. If a nurse has computer competency, the nurse is competent in informatics. b. To be proficient in informatics, a nurse should be able to discover, retrieve, and use information in practice. c. A nurse needs to know how to acquire, critique, and apply scientific evidence from literature databases. d. Nursing informatics integrates nursing science, computer science, and information science to manage and communicate information in nursing practice.

a. If a nurse has computer competency, the nurse is competent in informatics.

The nurse is caring for a patient who has cultured positive for Clostridium difficile. Which of the following nursing actions would be appropriate given this organism? a. Instruct assistive personnel to use soap and water rather than sanitizer to clean hands. b. Wear an N95 respirator when entering the patient room. c. Teach the patient cough etiquette. d. Place the patient on Droplet Precautions.

a. Instruct assistive personnel to use soap and water rather than sanitizer to clean hands. Clostridium difficile is a Gram-positive spore-forming bacteria that is best known for causing antibiotic-associated diarrhea. This gastrointestinal organism is spread by a patients feces. Contact precautions and proper hand hygiene are utilized to prevent transmission.

Review the following problem-focused nursing diagnoses and identify the diagnoses that are stated correctly. a. Impaired Skin Integrity related to physical immobility b. Fatigue related to heart disease c. Nausea related to gastric distention d. Need for improved Oral Mucosa Integrity related to inflamed mucosa e. Risk for Infection related to surgery

a. Impaired Skin Integrity related to physical immobility The related factors in diagnoses "Fatigue related to heart disease" and "Need for improved oral mucosa integrity related to inflamed mucosa" are incorrect. The related factor of a medical diagnosis (in Fatigue related to heart disease) cannot be corrected through nursing intervention. In "Need for improved oral mucosa integrity related to inflamed mucosa" there is no diagnosis, but instead a goal of care. "Risk for infection related to surgery" is incorrect; risk nursing diagnoses do not have defining characteristics or related factors because they have not occurred yet. c. Nausea related to gastric distention The related factors in diagnoses "Fatigue related to heart disease" and "Need for improved oral mucosa integrity related to inflamed mucosa" are incorrect. The related factor of a medical diagnosis (in Fatigue related to heart disease) cannot be corrected through nursing intervention. In "Need for improved oral mucosa integrity related to inflamed mucosa" there is no diagnosis, but instead a goal of care. "Risk for infection related to surgery" is incorrect; risk nursing diagnoses do not have defining characteristics or related factors because they have not occurred yet.

A couple who is caring for their aging parents are concerned about factors that put them at risk for falls. Which factors are most likely to contribute to an increase in falls in the elderly? a. Inadequate lighting b. Throw rugs c. Multiple medications d. Doorway thresholds e. Cords covered by carpets f. Staircases with handrails

a. Inadequate lighting b. Throw rugs c. Multiple medications d. Doorway thresholds e. Cords covered by carpets Falls most often occur while transferring from beds, chairs, and toilets; getting into or out of bathtubs; tripping over items such as cords covered by rugs or carpets, carpet edges, or doorway thresholds; slipping on wet surfaces; and descending stairs. Multiple medications also contribute to fall risk.

Given a rationale for preoperative and postoperative procedures, the client is better prepared to participate in care. For which of the following should the nurse provide instruction and rationale? a. Incentive spirometry b. Specific details regarding the progression of diet c. Working the call button for the nurse d. Using the patient-controlled analgesia (PCA) pump

a. Incentive spirometry Given a rationale for preoperative and postoperative procedures, the client is better prepared to participate in care. The diet progression should be discussed with the client by the unit nurse as the postsurgical diet progresses. The call light may be specific to the unit the client is on and is best taught to the client once he or she is on the unit so that the client can demonstrate to the nurse that he or she understands how to use it. The PCA pump is best taught to the client once he or she is on the unit so that the client can demonstrate to the nurse that he or she understands how to use it.

A new nurse complains to her preceptor that she has no time for therapeutic communication with her patients. Which of the following is the best strategy to help the nurse find more time for this communication? a. Include communication while performing tasks such as changing dressings and checking vital signs. b. Ask the patient if you can talk during the last few minutes of visiting hours. c. Ask Pastoral care to come back a little later in the day. d. Remind the nurse to complete all her tasks and then set up remaining time for communication.

a. Include communication while performing tasks such as changing dressings and checking vital signs. It is important for the nurse to take the opportunity to provide communication opportunities while providing routine patient care.

Which are key points that the nurse should include in patient education for a person with complaints of chronic constipation? a. Increase fiber and fluids in the diet b. Use a low-volume enema daily c. Avoid gluten in the diet d. Take laxatives twice a day e. Exercise for 30 minutes every day f. Schedule time to use the toilet at the same time every day g. Take probiotics 5 times a week

a. Increase fiber and fluids in the diet These steps are the initial ones to take to resolve chronic problems with constipation before considering regular laxative or enema use. e. Exercise for 30 minutes every day These steps are the initial ones to take to resolve chronic problems with constipation before considering regular laxative or enema use. f. Schedule time to use the toilet at the same time every day These steps are the initial ones to take to resolve chronic problems with constipation before considering regular laxative or enema use.

Which of the following describes a nurse's application of a specific knowledge base during critical thinking? (Select all that apply) a. Initiative in reading current evidence from the literature b. Application of nursing theory c. Reviewing policy and procedure manual d. Considering holistic view of patient needs e. Previous time caring for a specific group of patients

a. Initiative in reading current evidence from the literature A nurse's specific knowledge base will vary but includes basic nursing education, continuing education courses, and additional college degrees. In addition, it includes the knowledge gained from a nurse reading the nursing literature and acquiring information and theory from the basic sciences, humanities, behavioral sciences, and nursing. Nurse's knowledge base also involves a different way of thinking holistically about patient problems. b. Application of nursing theory A nurse's specific knowledge base will vary but includes basic nursing education, continuing education courses, and additional college degrees. In addition, it includes the knowledge gained from a nurse reading the nursing literature and acquiring information and theory from the basic sciences, humanities, behavioral sciences, and nursing. Nurse's knowledge base also involves a different way of thinking holistically about patient problems. d. Considering holistic view of patient needs A nurse's specific knowledge base will vary but includes basic nursing education, continuing education courses, and additional college degrees. In addition, it includes the knowledge gained from a nurse reading the nursing literature and acquiring information and theory from the basic sciences, humanities, behavioral sciences, and nursing. Nurse's knowledge base also involves a different way of thinking holistically about patient problems.

The nurse administers a tube feeding via a patient's nasogastric tube. This is an example of which of the following? a. Physical care technique b. Activity of daily living c. Indirect care measure d. Lifesaving measure

a. Physical care technique Administering a tube feeding is an example of a physical care, a direct care technique.

A patient is admitted through the emergency department following a motorcycle crash with multiple orthopedic injuries. He goes to surgery for repair of fractures. He is postoperative day 3 from an open-reduction internal fixation of bilateral femur fractures and external fixator to his unstable pelvic fracture. Interventions that are necessary for prevention of venous thromboembolism in this high-risk postsurgical patient include: a. Intermittent pneumatic compression stockings. b. Vitamin K therapy. c. Passive range-of-motion exercises every 4 hours. d. Subcutaneous heparin or enoxaparin (Lovenox). e. Continuous heparin drip with a goal of an international normalized ratio (INR) 5 times higher than baseline.

a. Intermittent pneumatic compression stockings. Combination therapy with mechanical and pharmacological prophylaxis is recommended for high-risk patients. Vitamin K therapy creates a higher risk for clotting, and the goal INR should not be 5 times higher than baseline. d. Subcutaneous heparin or enoxaparin (Lovenox). Combination therapy with mechanical and pharmacological prophylaxis is recommended for high-risk patients. Vitamin K therapy creates a higher risk for clotting, and the goal INR should not be 5 times higher than baseline.

A patient has an indwelling urinary catheter. Why does an indwelling urinary catheter present a risk for urinary tract infection? a. It allows migration of organisms into the bladder. b. The insertion procedure is not done under sterile conditions. c. It obstructs the normal flushing action of urine flow. d. It keeps an incontinent patient's skin dry. e. The outer surface of the catheter is not considered sterile.

a. It allows migration of organisms into the bladder. The presence of a catheter in the urethra breaches the natural defenses of the body. Reflux of microorganisms up the catheter lumen from the drainage bag or backflow of urine in the tubing increases the risk for infection. The indwelling catheter should always be placed under sterile conditions. c. It obstructs the normal flushing action of urine flow. The presence of a catheter in the urethra breaches the natural defenses of the body. Reflux of microorganisms up the catheter lumen from the drainage bag or backflow of urine in the tubing increases the risk for infection. The indwelling catheter should always be placed under sterile conditions.

A nurse uses long firm, strokes distal to proximal while bathing a patient's legs because: a. It promotes venous circulation. b. It covers a larger area of the leg. c. It completes care in a timely fashion. d. It prevents blood clots in legs.

a. It promotes venous circulation. Bathing a patient with long, firm strokes distal to proximal promotes circulation and increases venous return.

What should the nurse teach a young woman with a history of urinary tract infections about UTI prevention? a. Keep the bowels regular. b. Limit water intake to 1-2 glasses a day c. Wear cotton underwear d. Cleanse the perineum from front to back. e. Practice pelvic muscle exercise (Kegel) daily.

a. Keep the bowels regular. All are interventions that lead to healthy bladder habits. Adequate hydration will ensure that the bladder is regularly flushed out and will help prevent a UTI. Pelvic muscle exercises promote pelvic health but not necessarily prevent UTI. c. Wear cotton underwear All are interventions that lead to healthy bladder habits. Adequate hydration will ensure that the bladder is regularly flushed out and will help prevent a UTI. Pelvic muscle exercises promote pelvic health but not necessarily prevent UTI. d. Cleanse the perineum from front to back. All are interventions that lead to healthy bladder habits. Adequate hydration will ensure that the bladder is regularly flushed out and will help prevent a UTI. Pelvic muscle exercises promote pelvic health but not necessarily prevent UTI.

The nurse enters the room of an 82-year-old patient for whom she has not cared previously. The nurse notices that the patient wears a hearing aid. The patient looks up as the nurse approaches the bedside. Which of the following approaches are likely to be effective with an older adult? a. Listen attentively to the patient's story. b. Use gestures that reinforce your questions or comments. c. Stand back away from the bedside. d. Maintain direct eye contact. Ask questions quickly to reduce the patient's fatigue.

a. Listen attentively to the patient's story. Approaches for collecting an older-adult assessment include listening patiently, using nonverbal communication when a patient has a hearing deficit, and maintaining patientdirected eye gaze. Leaning forward, not backward, shows interest in what the patient has to say. b. Use gestures that reinforce your questions or comments. Approaches for collecting an older-adult assessment include listening patiently, using nonverbal communication when a patient has a hearing deficit, and maintaining patientdirected eye gaze. Leaning forward, not backward, shows interest in what the patient has to say. d. Maintain direct eye contact. Ask questions quickly to reduce the patient's fatigue. Approaches for collecting an older-adult assessment include listening patiently, using nonverbal communication when a patient has a hearing deficit, and maintaining patientdirected eye gaze. Leaning forward, not backward, shows interest in what the patient has to say.

A nurse is conferring with another nurse about the care of a patient with a stage II pressure ulcer. The two decide to review the clinical practice guideline of the hospital for pressure ulcer management. The use of a standardized guideline achieves which of the following? a. Makes it quicker and easier for nurses to intervene b. Sets a level of clinical excellence for practice c. Eliminates need to create an individualized care plan for the patient d. Delivers evidence-based interventions for stage II pressure ulcer e. Summarizes the various approaches used for the practice concern or problem

a. Makes it quicker and easier for nurses to intervene Even though a standardized clinical practice guideline offers evidence-based solutions for clinical excellence that nurses can quickly and easily apply in practice, a nurse remains accountable for individualizing even standardized interventions when necessary. A guideline is not a summary of various approaches used by clinicians for a practice issue; it is a summary of the most relevant evidence-based information. b. Sets a level of clinical excellence for practice Even though a standardized clinical practice guideline offers evidence-based solutions for clinical excellence that nurses can quickly and easily apply in practice, a nurse remains accountable for individualizing even standardized interventions when necessary. A guideline is not a summary of various approaches used by clinicians for a practice issue; it is a summary of the most relevant evidence-based information. d. Delivers evidence-based interventions for stage II pressure ulcer Even though a standardized clinical practice guideline offers evidence-based solutions for clinical excellence that nurses can quickly and easily apply in practice, a nurse remains accountable for individualizing even standardized interventions when necessary. A guideline is not a summary of various approaches used by clinicians for a practice issue; it is a summary of the most relevant evidence-based information.

Activity of daily living _____ a. assisting patient with oral care b. protecting a violent patient from injury c. discussing a patient's options in choosing palliative care d. using safe patient handling during positioning of a patient

a. assisting patient with oral care

After surgery the patient with a closed abdominal wound reports a sudden "pop" after coughing. When the nurse examines the surgical wound site, the sutures are open, and pieces of small bowel are noted at the bottom of the now-opened wound. Which are the priority nursing interventions? a. Notify the surgeon b. Allow the area to be exposed to air until all drainage has stopped c. Place several cold packs over the area, protecting the skin around the wound d. Cover the area with sterile, saline-soaked towels and immediately. e. Cover the area with sterile gauze and apply an abdominal binder

a. Notify the surgeon If a patient has an opening in the surgical incision and a portion of the small bowel is noted, the small bowel must be protected until an emergency surgical repair can be done. The small bowel and abdominal cavity should be maintained in a sterile environment; thus sterile towels that are moistened with sterile saline should be used over the exposed bowel for protection and to keep the bowel moist. d. Cover the area with sterile, saline-soaked towels and immediately. If a patient has an opening in the surgical incision and a portion of the small bowel is noted, the small bowel must be protected until an emergency surgical repair can be done. The small bowel and abdominal cavity should be maintained in a sterile environment; thus sterile towels that are moistened with sterile saline should be used over the exposed bowel for protection and to keep the bowel moist.

You are caring for a patient after surgery who underwent a liver resection. His prothrombin time (PT) or an activated partial thromboplastin time (APTT) is greater than normal. He has low blood pressure; tachycardia; thready pulse; and cool, clammy, pale skin, and he is restless. You assess his surgical wound, and the dressing is saturated with blood. Which immediate interventions should you perform? a. Notify the surgeon. b. Maintain intravenous (IV) fluid infusion and prepare to give volume replacement. c. Monitor the patient's vital signs every 15 minutes or more frequently until his condition stabilizes. d. Wean oxygen therapy. e. Provide comfort through bathing.

a. Notify the surgeon. A common early complication of surgery is bleeding. It is important to continue oxygen therapy and notify the surgeon. Signs of bleeding include hypotension; tachycardia; and cool, clammy, pale skin. Signs of bleeding may be visible, or the bleeding may be internal. Be prepared to administer fluid or blood as needed and frequently monitor vital signs to assess the patient's status. b. Maintain intravenous (IV) fluid infusion and prepare to give volume replacement. A common early complication of surgery is bleeding. It is important to continue oxygen therapy and notify the surgeon. Signs of bleeding include hypotension; tachycardia; and cool, clammy, pale skin. Signs of bleeding may be visible, or the bleeding may be internal. Be prepared to administer fluid or blood as needed and frequently monitor vital signs to assess the patient's status. c. Monitor the patient's vital signs every 15 minutes or more frequently until his condition stabilizes. A common early complication of surgery is bleeding. It is important to continue oxygen therapy and notify the surgeon. Signs of bleeding include hypotension; tachycardia; and cool, clammy, pale skin. Signs of bleeding may be visible, or the bleeding may be internal. Be prepared to administer fluid or blood as needed and frequently monitor vital signs to assess the patient's status.

Which of the following are examples of subjective data? (Select all that apply.) a. Patient describing excitement about discharge. b. Patient's wound appearance. c. Patient's expression of fear regarding upcoming surgery. d. Patient pacing the floor while awaiting test results. e. Patient's temperature.

a. Patient describing excitement about discharge. c. Patient's expression of fear regarding upcoming surgery.

Which of the following patients are at most risk for tachypnea? a. Patient just admitted with four rib fractures b. Woman who is 9 months' pregnant c. Adult who has consumed alcoholic beverages d. Adolescent waking from sleep e. Three-pack-per-day smoker with pneumonia

a. Patient just admitted with four rib fractures Patient with rib fractures is unlikely to breathe deeply and a large fetus restricts diaphragmatic movement, leading to decreased ventilatory volume. Pneumonia decreases gas exchange surface area. Tachypnea occurs to increase minute ventilation. Alcohol is a respiratory depressant. b. Woman who is 9 months' pregnant Patient with rib fractures is unlikely to breathe deeply and a large fetus restricts diaphragmatic movement, leading to decreased ventilatory volume. Pneumonia decreases gas exchange surface area. Tachypnea occurs to increase minute ventilation. Alcohol is a respiratory depressant. e. Three-pack-per-day smoker with pneumonia Patient with rib fractures is unlikely to breathe deeply and a large fetus restricts diaphragmatic movement, leading to decreased ventilatory volume. Pneumonia decreases gas exchange surface area. Tachypnea occurs to increase minute ventilation. Alcohol is a respiratory depressant.

Which of the following patients is at most risk for tachypnea? (Select all that apply.) a. Patient just admitted with four rib fractures b. Woman who is 9 months' pregnant c. Adult who has consumed alcoholic beverages d. Adolescent awaking from sleep

a. Patient just admitted with four rib fractures b. Woman who is 9 months' pregnant Rib fractures can be painful and decrease the patients willingness to take deep breaths leading to tachypnea. Pregnant women have decreased lung expansion related to compartmental changes of the abdomen which also can lead to tachypnea.

Which of these outcomes would be most appropriate for a patient with a nursing diagnosis of Constipation related to slowed gastrointestinal motility secondary to pain medications? a. Patient will have one soft, formed bowel movement by end of shift. b. Patient will not take any pain medications this shift. c. Patient will walk unassisted to bathroom by the end of shift. d. Patient will not take laxatives or stool softeners this shift.

a. Patient will have one soft, formed bowel movement by end of shift.

A nurse is checking a patient's intravenous line and, while doing so, notices how the patient bathes himself and then sits on the side of the bed independently to put on a new gown. This observation is an example of assessing: a. Patient's level of function. b. Patient's willingness to perform self-care. c. Patient's level of consciousness. d. Patient's health management values.

a. Patient's level of function. Observing a patient perform activities physical, socially, psychologically, and developmentally assesses his or her level of function. In the case of this question the nurse assesses physical functional level. Observation does not measure willingness to perform self-care but the ability to do so. Observing physical performance of self-hygiene is not a measure of level of consciousness nor does it reveal a patient's values.

Before transferring a patient from the bed to a stretcher, which assessment data does the nurse need to gather? (Select all that apply.) a. Patient's weight b. Patient's level of cooperation c. Patient's ability to assist d. Presence of medical equipment e. 24-hour calorie intake

a. Patient's weight b. Patient's level of cooperation c. Patient's ability to assist d. Presence of medical equipment

A nurse knows that the people most at risk for accidental hypothermia are: (Select all that apply) a. People who are homeless. b. People with respiratory conditions. c. People with cardiovascular conditions. d. The very old. e. People with kidney disorders.

a. People who are homeless. c. People with cardiovascular conditions. d. The very old. Exposure to severe cold for prolonged periods causes frostbite and accidental hypothermia. Older adults, the young, patients with cardiovascular conditions, patients who have ingested drugs or alcohol in excess, and people who are homeless are at high risk for hypothermia.

The nursing assessment of an 80-year-old patient who demonstrates some confusion but no anxiety reveals that the patient is a fall risk because she continues to get out of bed without help despite frequent reminders. The initial nursing intervention to prevent falls for this patient is to: a. Place a bed alarm device on the bed. b. Place the patient in a belt restraint. c. Provide one-on-one observation of the patient. d. Apply wrist restraints.

a. Place a bed alarm device on the bed. Consider and implement alternatives as appropriate before the use of a restraint. A bed alarm is an alternative that the nurse implements independently.

A nurse begins the night shift being assigned to five patients. She learns that the floor will be a registered nurse (RN) short as a result of a call in. A patient care technician from another area is coming to the nursing unit to assist. The nurse is required to do hourly rounds on all patients, so she begins rounds on the patient who has recently asked for a pain medication. As the nurse begins to approach the patient's room, a nurse stops her in the hallway to ask about another patient. Which factors in this nurse's unit environment will affect her ability to set priorities? a. Policy for conducting hourly rounds b. Staffing level c. Interruption by staff nurse colleague d. RN's years of experience e. Competency of patient care technician

a. Policy for conducting hourly rounds Many factors within the health care environment affect your ability to set priorities, including model for delivering care, the workflow routine and staffing levels of a nursing unit, and interruptions from other care providers. Available resources (e.g., policies and procedures) also affect priority setting. The nurse's years of experience and the competency of the patient care technician are not part of the environment. b. Staffing level Many factors within the health care environment affect your ability to set priorities, including model for delivering care, the workflow routine and staffing levels of a nursing unit, and interruptions from other care providers. Available resources (e.g., policies and procedures) also affect priority setting. The nurse's years of experience and the competency of the patient care technician are not part of the environment. c. Interruption by staff nurse colleague Many factors within the health care environment affect your ability to set priorities, including model for delivering care, the workflow routine and staffing levels of a nursing unit, and interruptions from other care providers. Available resources (e.g., policies and procedures) also affect priority setting. The nurse's years of experience and the competency of the patient care technician are not part of the environment.

When changing the soiled linen on the bed of a client who is comatose, the nurse notices a reddened, blanchable area approximately 2 cm in diameter on her left buttock. The nurse's initial skin breakdown intervention is to: a. Position the client on her right side b. Finish providing fresh, dry linen to the client's bed c. Include a 2-hour turning schedule in the client's care plan d. Measure the area in order to describe it in the nurses' notes

a. Position the client on her right side Pressure is the major cause in pressure ulcer formation, and changing the client's position to minimize the time spent in a particular position will be the best intervention to relieve the pressure. The remaining options are appropriate, but none has priority over proper positioning of the client.

Before meeting the patient, a nurse talks to other caregivers about the patient. The nurse is in which phase of the helping relationship? a. Pre-interaction b. Orientation c. Working d. Termination

a. Pre-interaction

At 12 noon the emergency department nurse hears that an explosion has occurred in a local manufacturing plant. Which action does the nurse take first? a. Prepare for an influx of patients b. Contact the American Red Cross c. Determine how to resume normal operations d. Evacuate patients per the disaster plan

a. Prepare for an influx of patients The emergency department nurse needs to prepare for the potential influx of patients first. Staff need to be aware of the disaster plan. Patients may need to be evaluated but not initially. The American Red Cross is not contacted initially. Determination of how to resume normal operations is part of the disaster plan and is determined before an actual event.

What is the priority concern when providing oral hygiene for a patient who is unconscious? a. Preventing aspiration b. Controlling mouth odor c. Applying local antiseptic such as chlorhexidine d. Thoroughly brushing all tooth and oral surfaces

a. Preventing aspiration

Which of these statements are true regarding disinfection and cleaning? a. Proper cleaning requires mechanical removal of all soil from an object or area. b. General environmental cleaning is an example of medical asepsis. c. When cleaning a wound, wipe around the wound edge first and then clean inward toward the center of the wound. d. Cleaning in a direction from the least to the most contaminated area helps reduce infections. e. Disinfecting and sterilizing medical devices and equipment involve the same procedures.

a. Proper cleaning requires mechanical removal of all soil from an object or area. Environmental surfaces (e.g., bedside table) potentially can contribute to cross-transmission by contamination of health care personnel from hand contact with contaminated surfaces, medical equipment, or patients. Cleaning from the least to the most contaminated area of a wound prevents recontamination of the cleaned area. b. General environmental cleaning is an example of medical asepsis. Environmental surfaces (e.g., bedside table) potentially can contribute to cross-transmission by contamination of health care personnel from hand contact with contaminated surfaces, medical equipment, or patients. Cleaning from the least to the most contaminated area of a wound prevents recontamination of the cleaned area. d. Cleaning in a direction from the least to the most contaminated area helps reduce infections. Environmental surfaces (e.g., bedside table) potentially can contribute to cross-transmission by contamination of health care personnel from hand contact with contaminated surfaces, medical equipment, or patients. Cleaning from the least to the most contaminated area of a wound prevents recontamination of the cleaned area.

The nurse is caring for a patient with the diagnosis of Impaired physical mobility. The nurse needs to be alert for which of the following potential complications? (Select all that apply.) a. Pulmonary emboli b. Pneumonia c. Impaired skin integrity d. Somnolence e. Increased socialization

a. Pulmonary emboli b. Pneumonia c. Impaired skin integrity

A nurse in a community health clinic has been caring for a young teenager with asthma for several months. The nurse's goal of care for this patient is to achieve self-management of asthma medications. Identify appropriate evaluative indicators for self-management for this patient. a. Quality of life b. Patient satisfaction c. Use of clinic services d. Adherence to use of inhaler e. Description of side effects of medications

a. Quality of life c. Use of clinic services d. Adherence to use of inhaler Relevant and appropriate evaluative indicators of self-management include self-efficacy, health behavior or attitude, health status, health service use, quality of life, and psychological indicators. In this case the patient's quality of life, use of clinic services, and adherence (behavior) to use of an inhaler are all appropriate. Patient satisfaction is a perception and not an indicator of self-management. Ability to describe medication side effects is a measure of knowledge but does not necessarily equate with successful self-management.

A 74-year-old is accompanied by his daughter to the ambulatory surgery department for the surgical removal of a suspicious skin lesion. The client has experienced dysphasia since a cerebral vascular accident 3 years ago. The most effective way for the nurse to secure the necessary preoperative interview information is to: a. Question the client's daughter b. Review the client's past medical records c. Present the questions in a simple format d. Counting respirations per minute

a. Question the client's daughter If a client is unable to relate all of the necessary information, rely on family members as resources. The remaining options are not reliable, effective methods of securing information regarding this client.

When repositioning an immobile client, the nurse notices redness over a bony prominence. When the area is assessed, the red spot blanches with fingertip touch, indicating: a. Reactive hyperemia, a reaction that causes the blood vessels to dilate in the injured area b. Sensitive skin that calls for the use of special bed linen c. A stage III pressure ulcer needing the appropriate dressing d. A local skin infection requiring antibiotics

a. Reactive hyperemia, a reaction that causes the blood vessels to dilate in the injured area This observation is indicative of reactive hyperemia. This is not a local skin infection or a stage III pressure ulcer. Not enough information is given to determine whether the client has sensitive skin.

A patient signals the nurse by turning on the call light. The nurse enters the room and finds the patient's drainage tube disconnected, 100 mL of fluid remaining in the intravenous (IV) line, and the patient asking questions about whether his doctor is coming. Which of the following does the nurse perform first? a. Reconnect the drainage tubing b. Inspect the condition of the IV dressing c. Obtain the next IV fluid bag from the medication room d. Explain when the health care provider is likely to visit

a. Reconnect the drainage tubing The nurse must reconnect the drainage tube for the priority of patient safety. There is no reason to suspect a problem with the IV dressing unless the fluid is not infusing on time. The nurse must prepare the next bottle of solution after reconnecting the drainage tube. At that time the nurse can check the condition of the IV dressing. As the nurse performs her care, she can inform the patient about when the physician will round, unless she is uncertain and needs to contact the physician.

A nurse just started working at a well-baby clinic. One of her recent experiences was to help a mother learn the steps of breastfeeding. During the first clinic visit the mother had difficulty positioning the baby during feeding. After the visit the nurse considers what affected the inability of the mother to breastfeed, including the mother's obesity and inexperience. The nurse's review of the situation is called: a. Reflection. b. Perseverance. c. Intuition. d. Problem solving.

a. Reflection. The mother had difficulty breastfeeding first time. The nurse relied on reflection to consider her previous actions and review what was successful and the opportunities for improvement. The nurse has not yet problem solved but might do so after reflection in anticipation of the patient's next clinic visit.

After a surgical patient has been given preoperative sedatives, which safety precaution should a nurse take? a. Reinforce to patient to remain in bed or on the stretcher b. Raise the side rails and keep the bed or stretcher in the high position c. Determine if patient has any allergies to latex d. Obtain informed consent immediately after sedative administration

a. Reinforce to patient to remain in bed or on the stretcher It is important for patient safety to inform patients who have been given sedatives of the importance of remaining in bed after preoperative sedatives are administered. It is inappropriate to have a bed or stretcher in the high position because of the increased fall risk and potential for injury. Obtain informed consent and assess allergies before sedative administration.

During the skin assessment of an older adult client who had a stroke, the nurse noted a reddened area over the coccyx. The next actions of the nurse for this client should include: a. Repositioning the client off the coccygeal area and reassessing the area in 1 hour b. Inserting a urinary catheter to prevent accumulation of moisture from urinary incontinence c. Placing the client in Fowler's position and returning in 2 hours d. Massaging the reddened area and repositioning the client

a. Repositioning the client off the coccygeal area and reassessing the area in 1 hour Repositioning the client and reassessing the area in 1 hour is the most appropriate action for the nurse. When pressure is relieved from an area, the blood flow returns and the redness will disappear if no damage has occurred. This is the appropriate assessment. Placement in Fowler's position would only increase pressure on the coccyx. Massaging of a reddened area is not recommended because it could cause further injury if the tissue is already compromised. Insertion of a urinary catheter will not relieve pressure on the coccyx.

A nurse is preparing medications for a patient. The nurse checks the name of the medication on the label with the name of the medication on the doctor's order. At the bedside the nurse checks the patient's name against the medication order as well. The nurse is following which critical thinking attitude: a. Responsible b. Complete c. Accurate d. Broad

a. Responsible The nurse is demonstrating responsibility for correct medication and patient identification. The other three choices are critical thinking intellectual standards.

A 55-year-old female patient was in a motor vehicle accident and is admitted to a surgical unit after repair of a fractured left arm and left leg. She also has a laceration on her forehead. An intravenous (IV) line is infusing in the right antecubital fossa, and pneumatic compression stockings are on the right lower leg. She is receiving oxygen via a simple face mask. Which sites do you instruct the nursing assistant to use for obtaining the patient's blood pressure and temperature? a. Right antecubital and tympanic membrane b. Right popliteal and rectal c. Left antecubital and oral d. Left popliteal and temporal artery

a. Right antecubital and tympanic membrane IV in right arm can be turned off while blood pressure is obtained. Blood pressure should not be measured on fractured extremities that have compromised circulation. Sequential stocking should remain on all the time while the patient is in bed to promote blood flow in lower right extremity. Tympanic membrane temperature is not affected by oxygen; the oxygen would need to be removed to take an oral temperature. Forehead laceration excludes temporal measurement. Rectal temperature is more invasive.

To assess Mr. Johnson's apical pulse, you should place the stethoscope on his point of maximum impulse (PMI), which is located: a. at 5th intercostal space (ICS) and midclavicular line. b. at 2nd intercostal space (ICS) and left sternal edge. c. at 2nd intercostal space (ICS) and right sternal edge. d. at 4th intercostal space (ICS) and midaxillary line.

a. at 5th intercostal space (ICS) and midclavicular line.

A nurse asks a nursing assistive personnel (NAP) to help the patient in room 418 walk to the bathroom right now. The nurse tells the NAP that the patient needs the assistance of one person and the use of a walker. The nurse also tells the NAP that the patient's oxygen can be removed while he goes to the bathroom but to make sure that it is put back on at 2 L. The nurse also instructs the NAP to make sure the side rails are up and the bed alarm is reset after the patient gets back in bed. Which of the following components of the "Five Rights of Delegation" were used by the nurse? a. Right task b. Right circumstances c. Right person d. Right direction/communication e. Right supervision/evaluation

a. Right task b. Right circumstances c. Right person d. Right direction/communication The nurse provided four of the five components but did not provide the right supervision/evaluation. The nurse delegated the task of taking a patient to the bathroom to the NAP, which is in the scope of an NAP's duties and responsibilities and matched to the NAP skill level. The nurse provided clear directions by describing the task and the time period to complete the task. The nurse did not use "please" and "thank you" in the request. The nurse did not ask the if there were any questions, which would provide the NAP an opportunity to get clarification if there were questions. The nurse did not ask the NAP to follow up to check on how the patient did or if there were any problems. The nurse did not provide appropriate monitoring, evaluation, intervention as needed, or feedback.

The operating room (OR) and postanesthesia care unit (PACU) are high-risk environments for patients with a latex allergy. Which safety measures to prevent a latex reaction should the nurse implement? a. Screening patients about food allergies known to have cross-reactivity to latex b. Having a latex allergy cart available at all times c. Communicating with the operating room (OR) team as soon as 24 to 48 hours in advance of the surgery when a latex-sensitive patient is identified d. Scheduling the latex-sensitive patient for the last operative case of the day e. Planning for the patient to be admitted to a private room after surgery

a. Screening patients about food allergies known to have cross-reactivity to latex Identifying patients with potential cross-reactivity is important since they may be unaware of their latex sensitivity. Having all necessary equipment easily accessible to staff ensures that all items are available when needed. It is important for the operative team to be aware of the case so they can plan appropriate safeguards; scheduling the latex-sensitive patient for the first case means that latex dust from the previous day was removed overnight before the latex-sensitive patient's operation. b. Having a latex allergy cart available at all times Identifying patients with potential cross-reactivity is important since they may be unaware of their latex sensitivity. Having all necessary equipment easily accessible to staff ensures that all items are available when needed. It is important for the operative team to be aware of the case so they can plan appropriate safeguards; scheduling the latex-sensitive patient for the first case means that latex dust from the previous day was removed overnight before the latex-sensitive patient's operation. c. Communicating with the operating room (OR) team as soon as 24 to 48 hours in advance of the surgery when a latex-sensitive patient is identified Identifying patients with potential cross-reactivity is important since they may be unaware of their latex sensitivity. Having all necessary equipment easily accessible to staff ensures that all items are available when needed. It is important for the operative team to be aware of the case so they can plan appropriate safeguards; scheduling the latex-sensitive patient for the first case means that latex dust from the previous day was removed overnight before the latex-sensitive patient's operation.

Following a head injury, the client has thin drainage coming from the left ear. The nurse describes this drainage as: a. Serous b. Purulent c. Cerebrospinal fluid d. Serosanguineous

a. Serous Serous drainage is clear, watery plasma. Purulent drainage is thick, yellow, green, tan, or brown. Drainage must be tested to determine if it is cerebrospinal fluid. The nurse should describe the drainage by its appearance (i.e., serous). Serosanguineous drainage is pale, red, and watery, a mixture of clear and red fluid.

Which type of pressure ulcer consists of an observable pressure-related alteration of intact skin that may show changes in skin temperature (warmth or coolness), tissue consistency (firm or beefy feel), and/or sensation (pain, itching) compared with an adjacent or opposite area on the body? a. Stage I b. Stage II c. Stage III d. Stage IV

a. Stage I In stage I the ulcer appears as a defined area of persistent redness in lightly pigmented skin and as a darker red, blue, or purple area in darker pigmented skin, with no open skin areas. The skin will be warmer or cooler than other areas, with a change in consistency and sensation. A stage II ulcer is characterized by partial-thickness skin loss involving the epidermis and possibly the dermis. In stage III the ulcer appears as a full-thickness skin loss involving damage or necrosis of subcutaneous tissue that may extend down to, but not through, the underlying fascia. In stage IV the ulcer shows as a full-thickness loss with extensive destruction, tissue necrosis, or damage to muscle, bone, or supporting structures.

During the administration of a warm tap-water enema, the patient complains of cramping abdominal pain that he rates 6 out of 10. What is the first thing the nurse should do? a. Stop the instillation b. Ask the patient to take deep breaths to decrease the pain c. Add soapsuds to the enema d. Tell the patient to bear down as he would when having a bowel movement

a. Stop the instillation When a patient complains of pain during an enema, the instillation should be stopped, and an assessment done before discontinuing or resuming the procedure.

Which interventions are appropriate for the nursing diagnosis Impaired tissue integrity related to poor wound healing secondary to diabetes? (Select all that apply.) a. Teach the patient about signs and symptoms of infection. b. Help the patient cope with changes in body image that result from the wound. c. Perform dressing changes twice a day as ordered. d. Administer medications to control the patient's blood sugar as ordered. e. Teach the family how to perform dressing changes.

a. Teach the patient about signs and symptoms of infection. c. Perform dressing changes twice a day as ordered. d. Administer medications to control the patient's blood sugar as ordered. e. Teach the family how to perform dressing changes.

The following statements are on a patient's nursing care plan. Which of the following statements is written as an outcome? a. The patient will verbalize a decreased pain level to a tolerable level by the end of this shift. b. The patient will demonstrate increased mobility by the end of this shift. c. The patient will demonstrate increased tolerance to activity over the next month. d. The patient will understand needed dietary changes by discharge.

a. The patient will verbalize a decreased pain level to a tolerable level by the end of this shift.

Of the following nursing goals, which is the most appropriate for a patient who has had a total hip replacement? a. The patient will walk 1000 feet using her walker by the time of discharge. b. The patient will ambulate by the time of discharge. c. The patient will ambulate briskly on the treadmill by the time of discharge. d. The nurse will assist the patient to ambulate in the hall.

a. The patient will walk 1000 feet using her walker by the time of discharge.

The infection control nurse has asked the staff to work on reducing the number of iatrogenic infections on the unit. Which of the following actions on your part would contribute to reducing health care-acquired infections? a. Teaching correct handwashing to assigned patients b. Using correct procedures in starting and caring for an intravenous infusion c. Providing perineal care to a patient with an indwelling urinary catheter d. Isolating a patient who has just been diagnosed as having tuberculosis e. Decreasing a patient's environmental stimuli to decrease nausea

a. Teaching correct handwashing to assigned patients Iatrogenic infections are infections associated with a procedure or therapy. The patient with tuberculosis was probably infected outside of the health care environment, and preventing nausea is not associated directly with infection prevention. b. Using correct procedures in starting and caring for an intravenous infusion Iatrogenic infections are infections associated with a procedure or therapy. The patient with tuberculosis was probably infected outside of the health care environment, and preventing nausea is not associated directly with infection prevention. c. Providing perineal care to a patient with an indwelling urinary catheter Iatrogenic infections are infections associated with a procedure or therapy. The patient with tuberculosis was probably infected outside of the health care environment, and preventing nausea is not associated directly with infection prevention.

The nurse is intervening for an identified nursing diagnosis of Risk for infection. Which direct care nursing intervention is most appropriate? a. Teaching the family proper handwashing technique b. Leaving side rails up at all times c. Teaching the patient how to use crutches d. Counseling the family on stress reduction techniques

a. Teaching the family proper handwashing technique

A patient with chest pain is having an emergency cardiac catheterization. Which teaching approach does the nurse use in this situation? a. Telling approach b. Selling approach c. Entrusting approach d. Participating approach

a. Telling approach The telling approach is most appropriate when preparing a patient for an emergency procedure.

The nurse is completing the preoperative checklist for an adult client who is scheduled for an operative procedure later in the morning. Which of the following preoperative assessment findings for this client indicates a need to contact the anesthesiologist? a. Temperature is 100° F. b. Pulse is 90 beats per minute. c. Respiratory rate is 20 breaths per minute. d. Blood pressure is 130/74 mm Hg.

a. Temperature is 100° F. An elevated temperature before surgery is a cause for concern. If the client has an underlying infection, the surgeon may choose to postpone surgery until the infection has been treated. An elevated body temperature increases the risk for fluid and electrolyte imbalance after surgery. Anxiety and fear commonly cause elevations in heart rate and blood pressure. A pulse rate of 90 beats per minute is not a concern. A respiratory rate of 20 breaths per minute is normal for an adult. A blood pressure of 130/74 mm Hg is not excessively elevated.

Which of the following factors does a nurse consider for a patient with the nursing diagnosis of Disturbed Sleep Pattern related to noisy home environment in choosing an intervention for enhancing the patient's sleep? a. The intervention should be directed at reducing noise. b. The intervention should be one shown to be effective in promoting sleep on the basis of research. c. The intervention should be one commonly used by the patient's sleep partner. d. The intervention should be one acceptable to the patient. e. The intervention should be one you used with other patients in the past.

a. The intervention should be directed at reducing noise. Select interventions that alter the etiological factor, in this case noise. Choose interventions that have a research base and are acceptable to patients. b. The intervention should be one shown to be effective in promoting sleep on the basis of research. Select interventions that alter the etiological factor, in this case noise. Choose interventions that have a research base and are acceptable to patients. d. The intervention should be one acceptable to the patient. Select interventions that alter the etiological factor, in this case noise. Choose interventions that have a research base and are acceptable to patients.

Which situation will cause the nurse to intervene and follow up on the nurse aide's behavior? a. The nurse aide is calling the older adult patient "honey." b. The nurse aide is facing the older adult patient when talking. c. The nurse aide cleans the older adult patient's glasses. d. The nurse aide allows time for the older adult patient to respond.

a. The nurse aide is calling the older adult patient "honey."

A nurse makes the following statement during a change-of-shift report to another nurse. "I assessed Mr. Diaz, my 61-year-old patient from Chile. He fell at home and hurt his back 3 days ago. He has some difficulty turning in bed, and he says that he has pain that radiates down his leg. He rates his pain at a 6, and he moves slowly as he transfers to a chair." What can the nurse who is beginning a shift do to validate the previous nurse's assessment findings when she rounds on the patient? (Select all that apply) a. The nurse asks the patient to rate his pain on a scale of 0 to 10. b. The nurse asks the patient what caused his fall. c. The nurse asks the patient if he has had pain in his back in the past. d. The nurse assesses the patient's lower-limb strength. e. The nurse asks the patient what pain medication is most effective in managing his pain.

a. The nurse asks the patient to rate his pain on a scale of 0 to 10. Validation of assessment data is the comparison of data with another source to determine its accuracy. The nurse compares data reported by the previous nurse with data collected directly with the patient, including assessing pain on the rating scale and assessing the patient's lower limb strength. Asking the patient what caused his fall and about past back pain and experience with pain medications would offer the nurse new information about the patient. d. The nurse assesses the patient's lower-limb strength. Validation of assessment data is the comparison of data with another source to determine its accuracy. The nurse compares data reported by the previous nurse with data collected directly with the patient, including assessing pain on the rating scale and assessing the patient's lower limb strength. Asking the patient what caused his fall and about past back pain and experience with pain medications would offer the nurse new information about the patient.

A nurse has been gathering physical assessment data on a patient and is now listening to the patient's concerns. The nurse sets a goal of care that incorporates the patient's desire to make treatment decisions. This is an example of the nurse engaged in which phase of the nurse-patient relationship? a. Working phase b. Preinteraction phase c. Termination phase d. Orientation phase

a. Working phase The nurse helps the patient identify goals and express feelings during the working phase of the helping relationship.

( A nurse working on a surgery floor is assigned five patients and has a patient care technician assisting her. Which of the following shows the nurse's understanding and ability to safely delegate to the patient care tech? (Select all that apply) a. The nurse considers the time available to gather routine vital signs on one patient before checking on a second patient arriving from a diagnostic test. b. Determining what is the patient care technician's current workload. c. The nurse chooses to delegate the measurement of a stable patient's vital signs and not the assessment of the patient arriving from a diagnostic test. d. The nurse reviews with the NAP, newly hired to the floor, her experience in measuring a blood pressure. e. The nurse confers with another registered nurse about organizing priorities.

a. The nurse considers the time available to gather routine vital signs on one patient before checking on a second patient arriving from a diagnostic test. A nurse must consider priorities of all assigned patients in deciding which activities should be delegated to NAP. When the decision is between vital signs versus a patient arriving from a diagnostic test, delegation of routine vital signs is appropriate. Ensuring that a NAP is competent to perform an activity is also important. Conferring with another RN about organizing and checking the tech's personal workload are not factors that will assist the RN's own priority setting. c. The nurse chooses to delegate the measurement of a stable patient's vital signs and not the assessment of the patient arriving from a diagnostic test. A nurse must consider priorities of all assigned patients in deciding which activities should be delegated to NAP. When the decision is between vital signs versus a patient arriving from a diagnostic test, delegation of routine vital signs is appropriate. Ensuring that a NAP is competent to perform an activity is also important. Conferring with another RN about organizing and checking the tech's personal workload are not factors that will assist the RN's own priority setting. d. The nurse reviews with the NAP, newly hired to the floor, her experience in measuring a blood pressure. A nurse must consider priorities of all assigned patients in deciding which activities should be delegated to NAP. When the decision is between vital signs versus a patient arriving from a diagnostic test, delegation of routine vital signs is appropriate. Ensuring that a NAP is competent to perform an activity is also important. Conferring with another RN about organizing and checking the tech's personal workload are not factors that will assist the RN's own priority setting.

The nurse manager from the surgical unit was awarded the nursing leadership award for practice of transformational leadership. Which of the following are characteristics or traits of transformational leadership displayed by award winner? a. The nurse manager regularly rounds on staff to gather input on unit decisions. b. The nurse manager sends thank-you notes to staff in recognition of a job well done. c. The nurse manager sends memos to staff about decisions that the manager has made regarding unit policies. d. The nurse manager has an "innovation idea box" to which staff are encouraged to submit ideas for unit improvements. e. The nurse develops a philosophy of care for the staff.

a. The nurse manager regularly rounds on staff to gather input on unit decisions. b. The nurse manager sends thank-you notes to staff in recognition of a job well done. d. The nurse manager has an "innovation idea box" to which staff are encouraged to submit ideas for unit improvements. Nurse managers who practice transformational leadership are focused on change and innovation. They motivate and empower their staff with the focus on team development. The manager will spend time on the unit with the staff, sharing ideas and listening to staff input. The manager is enthusiastic about opportunities to enhance the team and shows appreciation and recognizes team members for good work. The manager holds the team accountable and provides support for the team members in the stressful health care environment.

In which of the following examples is a nurse applying critical thinking skills in practice? a. The nurse thinks back about a personal experience before administering a medication subcutaneously. b. The nurse uses a pain-rating scale to measure a patient's pain. c. The nurse explains a procedure step by step for giving an enema to a patient care technician. d. The nurse gathers data on a patient with a mobility limitation to identify a nursing diagnosis. e. A nurse offers support to a colleague who has witnessed a stressful event.

a. The nurse thinks back about a personal experience before administering a medication subcutaneously. Reflection, using a pain-rating scale to be precise and specific, and nursing assessment (the first step of the nursing process) are examples of critical thinking skills. Explaining a procedure based on policy is not critical thinking. However, performing a procedure following policy is basic critical thinking. Offering support to a colleague is an important way to help another manage stress but is not a critical thinking skill. b. The nurse uses a pain-rating scale to measure a patient's pain. Reflection, using a pain-rating scale to be precise and specific, and nursing assessment (the first step of the nursing process) are examples of critical thinking skills. Explaining a procedure based on policy is not critical thinking. However, performing a procedure following policy is basic critical thinking. Offering support to a colleague is an important way to help another manage stress but is not a critical thinking skill. d. The nurse gathers data on a patient with a mobility limitation to identify a nursing diagnosis. Reflection, using a pain-rating scale to be precise and specific, and nursing assessment (the first step of the nursing process) are examples of critical thinking skills. Explaining a procedure based on policy is not critical thinking. However, performing a procedure following policy is basic critical thinking. Offering support to a colleague is an important way to help another manage stress but is not a critical thinking skill.

In which of the following examples are nurses making diagnostic errors? (Select all that apply) a. The nurse who observes a patient wincing and holding his left side and gathers no additional assessment data b. The nurse who measures joint range of motion after the patient reports pain in the left elbow c. The nurse who considers conflicting cues in deciding which diagnostic label to choose d. The nurse who identifies a diagnosis on the basis of a patient reporting difficulty sleeping e. The nurse who makes a diagnosis of Ineffective Airway Clearance related to pneumonia.

a. The nurse who observes a patient wincing and holding his left side and gathers no additional assessment data When the nurse observes the patient wincing and holding his left side but does not gather additional assessment data, he or she makes a data collection error by omitting important data (i.e., pain severity). A nursing diagnosis cannot be made on basis of a single defining characteristic, as seen when the nurse identifies a diagnosis on the basis of a patient reporting difficulty sleeping. A nursing diagnosis needs to be related to a patient's response, not a medical diagnosis such as pneumonia. The nurse who measures joint range of motion after the patient reports pain is correctly validating findings. Considering conflicting clues ensures that the nurse does not make an interpretation error. d. The nurse who identifies a diagnosis on the basis of a patient reporting difficulty sleeping When the nurse observes the patient wincing and holding his left side but does not gather additional assessment data, he or she makes a data collection error by omitting important data (i.e., pain severity). A nursing diagnosis cannot be made on basis of a single defining characteristic, as seen when the nurse identifies a diagnosis on the basis of a patient reporting difficulty sleeping. A nursing diagnosis needs to be related to a patient's response, not a medical diagnosis such as pneumonia. The nurse who measures joint range of motion after the patient reports pain is correctly validating findings. Considering conflicting clues ensures that the nurse does not make an interpretation error. e. The nurse who makes a diagnosis of Ineffective Airway Clearance related to pneumonia. When the nurse observes the patient wincing and holding his left side but does not gather additional assessment data, he or she makes a data collection error by omitting important data (i.e., pain severity). A nursing diagnosis cannot be made on basis of a single defining characteristic, as seen when the nurse identifies a diagnosis on the basis of a patient reporting difficulty sleeping. A nursing diagnosis needs to be related to a patient's response, not a medical diagnosis such as pneumonia. The nurse who measures joint range of motion after the patient reports pain is correctly validating findings. Considering conflicting clues ensures that the nurse does not make an interpretation error.

A patient who has been isolated for Clostridium difficile (C. difficile) asks you to explain what he should know about this organism. What is the most appropriate information to include in patient teaching? a. The organism is usually transmitted through the fecal-oral route. b. Hands should always be cleaned with soap and water versus alcohol-based hand sanitizer. c. Everyone coming into the room must be wearing a gown and gloves. d. While the patient is in contact precautions, he cannot leave the room. e. C. difficile dies quickly once outside the body.

a. The organism is usually transmitted through the fecal-oral route. C. difficile enters a person's body via ingestion of the spores that are spread via the fecal-oral route. Alcohol-based hand sanitizers have proved ineffective with C. difficile because of the spore that surrounds the organism, thus thorough handwashing is recommended. The most common way C. difficile is spread in a health care environment is through workers' contaminated hands; therefore barriers such as gloves and gowns are an important part of preventing transmission between patients. b. Hands should always be cleaned with soap and water versus alcohol-based hand sanitizer. C. difficile enters a person's body via ingestion of the spores that are spread via the fecal-oral route. Alcohol-based hand sanitizers have proved ineffective with C. difficile because of the spore that surrounds the organism, thus thorough handwashing is recommended. The most common way C. difficile is spread in a health care environment is through workers' contaminated hands; therefore barriers such as gloves and gowns are an important part of preventing transmission between patients. c. Everyone coming into the room must be wearing a gown and gloves. C. difficile enters a person's body via ingestion of the spores that are spread via the fecal-oral route. Alcohol-based hand sanitizers have proved ineffective with C. difficile because of the spore that surrounds the organism, thus thorough handwashing is recommended. The most common way C. difficile is spread in a health care environment is through workers' contaminated hands; therefore barriers such as gloves and gowns are an important part of preventing transmission between patients.

The nurse is evaluating whether patient goals and outcomes have been met. Which option below is an expected outcome for a patient with Impaired physical mobility? a. The patient is able to ambulate in the hallway with crutches for 15 minutes with no assistance. b. The patient's level of mobility will improve. The nurse provides assistance while the patient is walking in the hallways. The patient will deny pain while walking in the hallway.

a. The patient is able to ambulate in the hallway with crutches for 15 minutes with no assistance.

The nurse is transferring a patient to a long-term, skilled care facility and has just given a telephone report to a registered nurse (RN) who works at that facility and who will be receiving the patient. In documenting this call, the nurse begins by writing the date and time the report was given and the name of the RN taking the report. Which of the following pieces of information does the nurse include in the documentation of this telephone call? a. The patient's name, age, and admitting diagnoses b. The discussion of any allergies to food and medications that the patient has c. That the nurse receiving the report was advised that the patient is "needy" and "on the call light all the time" d. That the patient's pain rating went from 8 to 2 on a scale of 1 to 10 after receiving 650 mg of Tylenol e. Description of any unresolved problems and current interventions in place

a. The patient's name, age, and admitting diagnoses b. The discussion of any allergies to food and medications that the patient has d. That the patient's pain rating went from 8 to 2 on a scale of 1 to 10 after receiving 650 mg of Tylenol e. Description of any unresolved problems and current interventions in place During transfer to another institution, include essential background information such as the patient's name, age, diagnosis, and allergies. Also include response to treatments such as response to pain-relieving measures. Information about how much the patient ate for breakfast is not necessary. This information is in the chart if the nurse really needs to know. Do not include critical comments about the patients.

A nurse gathers the following assessment data. Which of the following cues together form(s) a pattern suggesting a problem? (Select all that apply) a. The skin around the wound is tender to touch. b. Fluid intake for 8 hours is 800 mL. c. Patient has a heart rate of 78 beats/min and regular. d. Patient has drainage from surgical wound. e. Body temperature is 38.3° C (101° F). f. Patient states, "I'm worried that I won't be able to return to work when I planned."

a. The skin around the wound is tender to touch. Tender skin around the wound, drainage from the surgical wound, and a temperature of 38.3° C (101° F) indicate a wound infection. Fluid intake of 800 mL over 8 hours and a heart rate of 78 beats/min and regular are normal assessment findings. A patient's expressed concern about returning to work is his or her subjective response about a separate issue and is insufficient to form a pattern. d. Patient has drainage from surgical wound. Tender skin around the wound, drainage from the surgical wound, and a temperature of 38.3° C (101° F) indicate a wound infection. Fluid intake of 800 mL over 8 hours and a heart rate of 78 beats/min and regular are normal assessment findings. A patient's expressed concern about returning to work is his or her subjective response about a separate issue and is insufficient to form a pattern. e. Body temperature is 38.3° C (101° F). Tender skin around the wound, drainage from the surgical wound, and a temperature of 38.3° C (101° F) indicate a wound infection. Fluid intake of 800 mL over 8 hours and a heart rate of 78 beats/min and regular are normal assessment findings. A patient's expressed concern about returning to work is his or her subjective response about a separate issue and is insufficient to form a pattern.

When a wound specimen is obtained for culture to determine whether infection is present, the specimen should to be taken from: a. The wound after it has first been cleansed with normal saline b. Drainage on the dressing c. Wound drainage d. Necrotic tissue

a. The wound after it has first been cleansed with normal saline The wound should be cleaned with saline, then a culture specimen should be obtained from the wound. Necrotic tissue, drainage on the dressing, and old wound drainage can harbor old bacteria that may not necessarily be infecting the wound.

The effects of immobility on the cardiac system include which of the following? a. Thrombus formation b. Increased cardiac workload c. Weak peripheral pulses d. Irregular heartbeat e. Orthostatic hypotension

a. Thrombus formation The three major changes are orthostatic hypotension, increased cardiac workload, and thrombus formation. b. Increased cardiac workload The three major changes are orthostatic hypotension, increased cardiac workload, and thrombus formation. e. Orthostatic hypotension The three major changes are orthostatic hypotension, increased cardiac workload, and thrombus formation.

Purposes of the Nursing Outcomes Classification (NOC) include which of the following? a. To identify and label nurse-sensitive patient outcomes b. To test the classification in clinical settings c. To establish health care reimbursement guidelines d. To identify nursing interventions for linked nursing diagnoses e. To define measurement procedures for outcomes

a. To identify and label nurse-sensitive patient outcomes The NOC classification offers a language for the evaluation step of the nursing process. The purposes of NOC are to (1) identify, label, validate, and classify nurse-sensitive patient outcomes; (2) field test and validate the classification; and (3) define and test measurement procedures for the outcomes and indicators using clinical data. b. To test the classification in clinical settings The NOC classification offers a language for the evaluation step of the nursing process. The purposes of NOC are to (1) identify, label, validate, and classify nurse-sensitive patient outcomes; (2) field test and validate the classification; and (3) define and test measurement procedures for the outcomes and indicators using clinical data. e. To define measurement procedures for outcomes The NOC classification offers a language for the evaluation step of the nursing process. The purposes of NOC are to (1) identify, label, validate, and classify nurse-sensitive patient outcomes; (2) field test and validate the classification; and (3) define and test measurement procedures for the outcomes and indicators using clinical data.

When is an application of a warm compress to an ankle muscle sprain indicated? a. To relieve edema b. To reduce shivering c. To improve blood flow to an injured part d. To protect bony prominences from pressure ulcers e. To immobilize area

a. To relieve edema Warm compresses can improve circulation by dilating blood vessels, and they reduce edema. The moisture of the compress conducts heat. c. To improve blood flow to an injured part Warm compresses can improve circulation by dilating blood vessels, and they reduce edema. The moisture of the compress conducts heat.

A nurse needs to begin discharge planning for a patient admitted with pneumonia and a congested cough. When is the best time the nurse should start discharge planning for this patient? a. Upon admission b. Right before discharge c. After the congestion is treated d. When the primary care provider writes the order

a. Upon admission

A patient is aphasic, and the nurse notices that the patient's hands shake intermittently. Which nursing action is most appropriate to facilitate communication? a. Use a picture board. b. Use pen and paper. c. Use an interpreter. d. Use a hearing aid.

a. Use a picture board.

Which of the following are measures to reduce tissue damage from shear? a. Use a transfer device, e.g. transfer board b. Have head of bed elevated when transferring patient c. Have head of bed flat when re positioning patients d. Raise head of bed 60 degrees when patient positioned supine e. Raise head of bed 30 degrees when patient positioned supine

a. Use a transfer device, e.g. transfer board A transfer device can pick up a patient and prevent his or her skin from sticking to the bed sheet as he is repositioned. Positioning the patient flat when repositioning reduces shear. Positioning the patient with the head of the bed to be elevated at 30 degrees prevents him or her from sliding. The head of bed in higher position will cause patient to slide down, causing shear. c. Have head of bed flat when re positioning patients A transfer device can pick up a patient and prevent his or her skin from sticking to the bed sheet as he is repositioned. Positioning the patient flat when repositioning reduces shear. Positioning the patient with the head of the bed to be elevated at 30 degrees prevents him or her from sliding. The head of bed in higher position will cause patient to slide down, causing shear. e. Raise head of bed 30 degrees when patient positioned supine A transfer device can pick up a patient and prevent his or her skin from sticking to the bed sheet as he is repositioned. Positioning the patient flat when repositioning reduces shear. Positioning the patient with the head of the bed to be elevated at 30 degrees prevents him or her from sliding. The head of bed in higher position will cause patient to slide down, causing shear.

The American Dental Association suggests that patients who are at risk for poor hygiene use the following interventions for oral care: a. Use antimicrobial toothpaste. b. Brush teeth 4 times a day. c. Use 0.12% chlorhexidine gluconate (CHG) oral rinses. d. Use a soft toothbrush for oral care. e. Avoid cleaning the gums and tongue.

a. Use antimicrobial toothpaste. The American Dental Association guidelines (2014) for effective oral hygiene include brushing the teeth at least twice a day with an American Dental Association approved fluoride toothpaste. Use antimicrobial toothpastes and 0.12% CHG oral rinses for patients at increased risk for poor oral hygiene (e.g., older adults and patients with cognitive impairments and who are immunocompromised). Rounded soft bristles stimulate the gums without causing abrasion and bleeding. Patients should clean gum and the surface of the tongue. c. Use 0.12% chlorhexidine gluconate (CHG) oral rinses. The American Dental Association guidelines (2014) for effective oral hygiene include brushing the teeth at least twice a day with an American Dental Association approved fluoride toothpaste. Use antimicrobial toothpastes and 0.12% CHG oral rinses for patients at increased risk for poor oral hygiene (e.g., older adults and patients with cognitive impairments and who are immunocompromised). Rounded soft bristles stimulate the gums without causing abrasion and bleeding. Patients should clean gum and the surface of the tongue. d. Use a soft toothbrush for oral care. The American Dental Association guidelines (2014) for effective oral hygiene include brushing the teeth at least twice a day with an American Dental Association approved fluoride toothpaste. Use antimicrobial toothpastes and 0.12% CHG oral rinses for patients at increased risk for poor oral hygiene (e.g., older adults and patients with cognitive impairments and who are immunocompromised). Rounded soft bristles stimulate the gums without causing abrasion and bleeding. Patients should clean gum and the surface of the tongue.

A nurse is listening to a student provide instruction to a patient who is having difficulty with activities needed to care for soft contact lenses. Which of the following statements by the nursing student might require some correction by the nurse? a. Use tap water to clean soft lenses. b. Follow recommendations of lens manufacturer when inserting the lenses. c. Keep lenses moist or wet when not worn. d. Use fresh solution daily when storing and disinfecting lenses.

a. Use tap water to clean soft lenses. The patient should not use water to clean soft contact lenses.

A patient has a diagnosis of pneumonia. Which entry should the nurse chart to help with financial reimbursement? a. Used incentive spirometer to encourage coughing and deep breathing. Lung congested upon auscultation in lower lobes bilaterally. Pulse ox 86%. Oxygen per nasal cannula applied at 2 L/min per standing order. b. Cooperative, patient coughed and deep breathed using a pillow as a splint. Stated, "felt better." Finally, patient had no complaints. c. Breathing without difficulty. Sitting up in bed watching TV. Had a good day. d. Status unchanged. Remains stable with no abnormal findings. Checked every 2 hours.

a. Used incentive spirometer to encourage coughing and deep breathing. Lung congested upon auscultation in lower lobes bilaterally. Pulse ox 86%. Oxygen per nasal cannula applied at 2 L/min per standing order.

It is time for a nurse hand-off between the night nurse and nurse starting the day shift. The night nurse checks the most recent laboratory results for the patient and then begins to discuss the patient's plan of care to the day nurse using the standard checklist for reporting essential information. The patient has been seriously ill, and his wife is at the bedside. The nurse asks the wife to leave the room for just a few minutes. The night nurse completes the summary of care before the day nurse is able to ask a question. Which of the following activities are strategies for an effective hand-off? (Select all that apply) a. Using a standardized checklist for essential information b. Asking the wife to briefly leave the room c. Completing the hand-off without inviting questions d. Doing prework such as checking laboratory results before giving a report e. Including the wife in the hand-off discussion

a. Using a standardized checklist for essential information Using standardized forms or checklists and doing thorough prework enhance the nurse's ability to communicate the plan of care effectively during a hand-off. It is also important to include patient and family when possible. The other two options are barriers to an effective hand-off. d. Doing prework such as checking laboratory results before giving a report Using standardized forms or checklists and doing thorough prework enhance the nurse's ability to communicate the plan of care effectively during a hand-off. It is also important to include patient and family when possible. The other two options are barriers to an effective hand-off. e. Including the wife in the hand-off discussion Using standardized forms or checklists and doing thorough prework enhance the nurse's ability to communicate the plan of care effectively during a hand-off. It is also important to include patient and family when possible. The other two options are barriers to an effective hand-off.

Hand-off communications that occur between the postanesthesia care unit (PACU) nurse and the nurse on the postoperative nursing unit should be done when a patient returns to the nursing unit. Which are appropriate components of a safe and effective hand-off? a. Vital signs, type of anesthesia provided, blood loss, and level of consciousness b. Uninterrupted time to review the recent pertinent events and ask questions c. Verification of the patient using one identifier and the type of surgery performed d. Review of pertinent events occurring in the operating room (OR) while at the nurses' station e. Location of patient's family members

a. Vital signs, type of anesthesia provided, blood loss, and level of consciousness A standardized approach or tool for hand-off communication helps providers provide accurate information about the care received in the operating room and the PACU before coming to the postoperative nursing unit. Proper identification of the patient requires using a standard of two identifiers and explaining the surgery performed and information about the type of anesthesia provided, blood loss, and level of consciousness. Allowing appropriate time for questions and communication free of distraction improves the quality of the hand-off. It must occur at the patient's bedside. Informing the nurse of the family's location ensures prompt notification. b. Uninterrupted time to review the recent pertinent events and ask questions A standardized approach or tool for hand-off communication helps providers provide accurate information about the care received in the operating room and the PACU before coming to the postoperative nursing unit. Proper identification of the patient requires using a standard of two identifiers and explaining the surgery performed and information about the type of anesthesia provided, blood loss, and level of consciousness. Allowing appropriate time for questions and communication free of distraction improves the quality of the hand-off. It must occur at the patient's bedside. Informing the nurse of the family's location ensures prompt notification. e. Location of patient's family members A standardized approach or tool for hand-off communication helps providers provide accurate information about the care received in the operating room and the PACU before coming to the postoperative nursing unit. Proper identification of the patient requires using a standard of two identifiers and explaining the surgery performed and information about the type of anesthesia provided, blood loss, and level of consciousness. Allowing appropriate time for questions and communication free of distraction improves the quality of the hand-off. It must occur at the patient's bedside. Informing the nurse of the family's location ensures prompt notification.

When receiving a report at the beginning of your shift, you learn that your assigned client has a surgical incision that is healing by primary intention. You know that your client's incision is: a. Well approximated, with minimal or no drainage. b. Going to take a little longer than usual to heal. c. Going to have more scarring than most incisions. d. Draining some serosanguineous drainage.

a. Well approximated, with minimal or no drainage. Well approximated, with minimal or no drainage; Primary intention means that the wound edges are well approximated, with minimal or no tissue loss as well as formation of minimal granulation tissue and scarring.

An 82-year-old patient who resides in a nursing home has the following three nursing diagnoses: Risk for Fall, Impaired Physical Mobility related to pain, and Imbalanced Nutrition: Less Than Body Requirements related to reduced ability to feed self. The nursing staff identified several goals of care. Match the goals on the left with the appropriate outcome statements on the right. Goals Outcomes (Patient will ambulate independently in 3 days.) a. patient walks 20 feet using a walker in 24 hrs. b. patient identifies barriers to remove in the home within 1 week c. patient expresses fewer nonverbal signs of discomfort within 24 hrs d. patient increases calorie intake to 2500 daily

a. patient walks 20 feet using a walker in 24 hrs.

The most effective way to prevent the spread of Mr. Johnson's pneumonia is: a. practicing good hand hygiene. b. wearing gloves. c. placing clients in isolation. d. providing private rooms for clients.

a. practicing good hand hygiene.

"The pain increases every time I try to turn on my left side." a. subjective b. objective c. assessment d. plan

a. subjective

Since pneumonia can be transmitted directly from Mr. Johnson to another person, it is considered a communicable disease. a. true b. false

a. true

Which of the following symptoms are warning signs of possible colorectal cancer according to the American Cancer Society guidelines? (Select all that apply) a.Change in bowel habits b. Blood in the stool c. A larger-than-normal bowel movement d. Fecal impaction e. Muscle aches f. Incomplete emptying of the colon g. Food particles in the stool h. Unexplained abdominal or back pain

a.Change in bowel habits According to the American Cancer Society current guidelines, anyone with these symptoms should seek medical evaluation because they may have colon cancer. Other conditions may also cause these symptoms; but, if colon cancer is present, early diagnosis is important. b. Blood in the stool According to the American Cancer Society current guidelines, anyone with these symptoms should seek medical evaluation because they may have colon cancer. Other conditions may also cause these symptoms; but, if colon cancer is present, early diagnosis is important. f. Incomplete emptying of the colon According to the American Cancer Society current guidelines, anyone with these symptoms should seek medical evaluation because they may have colon cancer. Other conditions may also cause these symptoms; but, if colon cancer is present, early diagnosis is important. h. Unexplained abdominal or back pain According to the American Cancer Society current guidelines, anyone with these symptoms should seek medical evaluation because they may have colon cancer. Other conditions may also cause these symptoms; but, if colon cancer is present, early diagnosis is important.

Which of these questions would be most appropriate for a nurse to ask a patient to assist in establishing a nursing diagnosis of Diarrhea? a. "What types of foods do you think caused your upset stomach?" b. "How many bowel movements a day have you had?" c. "Are you able to get to the bathroom in time?" d. "What medications are you currently taking?"

b. "How many bowel movements a day have you had?"

A patient says, "You are the worst nurse I have ever had." Which response by the nurse is the most assertive? a. "If I were you, I'd feel grateful for a nurse like me." b. "I feel uncomfortable hearing that statement." c. "How can you say that when I have been checking on you regularly?" d. "You shouldn't say things like that, it is not right."

b. "I feel uncomfortable hearing that statement."

A patient asks a nurse what the patient-centered care model for the hospital means. What is the nurse's best answer? a. "This model ensures that all patients have private rooms when they are admitted to the hospital." b. "In this model you and the health care team are full partners in decisions related to your health care." c. "This model focuses on making the patient experience a good one by providing amenities such as restaurant-style food service." d. "Patients and families sign a document providing them full access to their medical charts."

b. "In this model you and the health care team are full partners in decisions related to your health care." Patient- and family-centered care is based on the development of mutual partnerships among the patient, family, and health care team to plan, implement, and evaluate the patient's health care. The patient and the family are at the center of the care and are full partners in decision making.

A 24-year-old male client has been scheduled to undergo surgery for an ACL repair of his right knee. The client states that he is confused about what the surgeon will be doing. The best response from the nurse is: a. "The surgeon went over this procedure with you in his office" b. "Let me get the surgeon to talk with you before we proceed so that you fully understand what will be happening" c. To share with the client what he can expect in regard to the procedure d. "This is just a simple procedure—you should feel much better afterwards"

b. "Let me get the surgeon to talk with you before we proceed so that you fully understand what will be happening" The surgeon is responsible for making sure that the client completely understands the procedure before the client gives informed consent. The client may not remember the conversation that the surgeon had with him regarding the procedure due to anxiety. The nurse should not discount the client's concerns.

A patient states, "I would like to see what is written in my medical record." What is the nurse's best response? a. "Only your family can read your medical record." b. "You have the right to read your record." c. "Patients are not allowed to read their records." d. "Only health care workers have access to patient records."

b. "You have the right to read your record." Patients have the right to read their medical records, but the nurse should always know the facility policy regarding personal access to medical records because some require a nurse manager or other official to be present to answer questions about what is in the record. Families may read the records only when the patient has given permission.

During the implementation step of the nursing process, a nurse reviews and revises a patient's plan of care. What is the correct order of steps for this?1. Modify care plan as needed. 2. Decide if the nursing interventions remain appropriate. 3. Reassess the patient. 4. Compare assessment findings to validate existing nursing diagnoses. a. 2, 1, 3, 4 b. 3, 4, 2, 1 c. 4, 3, 2, 1 d. 3, 4, 1, 2

b. 3, 4, 2, 1 Reassessment allows you to review a patient's care plan by validating the nursing diagnoses and determine whether the nursing interventions remain the most appropriate for the clinical situation. When changes are needed, you modify the plan of care.

Which of the following clients is most at risk for developing a pressure ulcer? a. 3-year-old in Buck's traction b. 33-year-old comatose client c. 76-year-old client who has had a mild stroke d. 38-week-old infant in an oxygen hood Clients in a coma cannot perceive pressure and are unable to move voluntarily to relieve pressure.

b. 33-year-old comatose client Clients in a coma cannot perceive pressure and are unable to move voluntarily to relieve pressure.

The nurse defines a clinical guideline or protocol as a a. Guideline to follow that replaces the nursing care plan. b. Document that assists the clinician in making decisions and choosing interventions for specific health care problems or conditions. c. Hospital policy designating each nurse's duty according to standards of care and a code of ethics. d. Prescriptive order form that individualizes the plan of care.

b. Document that assists the clinician in making decisions and choosing interventions for specific health care problems or conditions.

A nurse prepares to contact a patient's physician about a change in the patient's condition. Using SBAR (Situation, Background, Assessment, and Recommendation) communication, which of the following is the correct order? 1."She is a 53-year-old female who was admitted 2 days ago with pneumonia and was started on Levaquin at 5 pm yesterday. She complains of a poor appetite." 2. "The patient reported feeling very nauseated after her dose of Levaquin an hour ago." 3. "Would you like to make a change in antibiotics, or could we give her a nutritional supplement before her medication?" 4. "The patient started complaining of nausea yesterday evening and has vomited several times during the night." a. 1, 3, 4, 2 b. 4, 1, 2, 3 c. 2, 1, 3, 4 d. 4, 2, 1, 3

b. 4, 1, 2, 3 The nurse describes the patient's complaint of nausea and vomiting to the physician (Situation). Specific patient demographic information and reason for admission with current symptomology are provided (Background). The physician is informed of the patient's complaint of nausea after receiving Levaquin (Assessment). Physician is asked if he or she would like to make a change in the antibiotic or provide a nutritional supplement before medication administration (Recommendation).

A patient's surgical wound has become swollen, red, and tender. You note that the patient has a new fever, purulent wound drainage, and leukocytosis. Which interventions would be appropriate and in what order? 1. Notify the health care provider of the patient's status. 2. Reassure the patient and recheck the wound later. 3. Support the patient's fluid and nutritional needs. 4. Use aseptic technique to change the dressing. a. 4, 1, 2, 3 b. 4, 2, 1, 3 c. 4, 2, 3, 1 d. 2, 4, 1, 3

b. 4, 2, 1, 3 The first three interventions require more immediate attention to the signs of suspected infection; supporting the patient's wound healing and hydration is important but can be addressed after action is taken to address early signs of infection.

Which of the following is the correct order for insertion of an indwelling catheter in a female patient?1. Insert and advance catheter.2. Lubricate catheter. 3. Inflate catheter balloon. 4. Cleanse urethral meatus. 5. Drape the patient with the sterile square and fenestrated drapes. 6. When urine appears advance another 2.5 to 5 cm. 7. Prepare sterile field and supplies. 8. Gently pull catheter until resistance is felt. 9. Attach drainage tubing. a. 7, 5, 2, 1, 4, 6, 3, 8, 9 b. 5, 7, 2, 4, 1, 6, 3, 8, 9 c. 5, 7, 1, 2, 4, 6, 3, 9, 8 d. 5, 7, 2, 1, 4, 3, 6, 8, 9

b. 5, 7, 2, 4, 1, 6, 3, 8, 9 This is the correct order for insertion of an indwelling catheter in a female patient.

The patient at greatest risk for developing multiple adverse effects of immobility is a: a. 1-year-old child with a hernia repair. b. 80-year-old woman who has suffered a hemorrhagic cerebrovascular accident (CVA). c. 51-year-old woman following a thyroidectomy. d. 38-year-old woman undergoing a hysterectomy.

b. 80-year-old woman who has suffered a hemorrhagic cerebrovascular accident (CVA).

You have been given the following postoperative patients for care on your medical-surgical unit. On the basis of the information provided, which patient should you see first? a. A 75-year-old following hip replacement surgery who is complaining of moderate pain in the surgical site, with a heart rate of 92 b. A 57-year-old following hip replacement 6 hours earlier who is receiving intravenous patient-controlled analgesia (PCA) with a history of obstructive sleep apnea (OSA) (The pulse oximeter has been alarming and reading 85%.) c. A 36-year-old following bladder neck suspension who is 30 minutes late to receive her postoperative dose of antibiotic d. A 48-year-old following total knee replacement who needs help repositioning in bed

b. A 57-year-old following hip replacement 6 hours earlier who is receiving intravenous patient-controlled analgesia (PCA) with a history of obstructive sleep apnea (OSA) (The pulse oximeter has been alarming and reading 85%.) The patient with OSA has a risk of airway obstruction, which takes immediate precedence. She is symptomatic of oxygen desaturation.

A nurse is planning a teaching session about healthy nutrition with a group of children who are in first grade. The nurse determines that after the teaching session the children will be able to name three examples of foods that are fruits. This is an example of: a. A teaching plan. b. A learning objective. c. Reinforcement of content. d. Enhancing the children's self-efficacy.

b. A learning objective. A learning objective describes the behavior(s) the learner will exhibit as a result of successful instruction.

As a nurse, you are assigned to four patients. Which patient do you need to see first? a. The patient who had abdominal surgery 2 days ago who is requesting pain medication b. A patient admitted yesterday with atrial fibrillation with decreased level of consciousness c. A patient with a wound drain who needs teaching before discharge in the early afternoon d. A patient going to surgery for a mastectomy in 3 hours who has a question about the surgery

b. A patient admitted yesterday with atrial fibrillation with decreased level of consciousness This patient is of high priority. The patient is experiencing the physiological problem of decreased level of consciousness, which is an immediate threat to his or her survival and safety. The nurse must intervene promptly and notify the health care provider of the life-threatening problem.

The nurse is evaluating whether a patient's turning schedule was effective in preventing the formation of pressure ulcers. Which finding indicates success of the turning schedule? a. Staff documentation of turning the patient every 2 hours. b. Absence of skin breakdown. c. Presence of redness only on the heels of the patient. d. Patient's eating 100% of all meals.

b. Absence of skin breakdown.

The nurse should do which of the following when placing a bedpan under an immobilized patient? a. Lift the patient's hips off the bed and slide the bedpan under the patient b. After positioning the patient on the bedpan, elevate the head of the bed to a 45-degree angle c. Adjust the head of the bed so it is lower than the feet and use gentle but firm pressure to push the bedpan under the patient d. Have the patient stand beside the bed and then have him or her sit on the bedpan on the edge of the bed

b. After positioning the patient on the bedpan, elevate the head of the bed to a 45-degree angle Elevating the head of the bed allows the patient the most normal and comfortable position for defecation on a bedpan.

Of the following developmental changes, which are most commonly associated with the elderly? (Select all that apply.) a. Increased eccrine and apocrine gland function b. Fungal nail infections c. Less resilient skin and bruising d. Increased skin lubrication e. Dry, itchy skin

b. Fungal nail infections c. Less resilient skin and bruising e. Dry, itchy skin

Which nursing interventions should the nurse implement when removing an indwelling urinary catheter in an adult patient? (Select all that apply) a. Attach a 3 mL syringe to the inflation port b. Allow the balloon to drain into the syringe by gravity. c. Initiate a voiding record/bladder diary d. Pull catheter quickly e. Clamp the catheter prior to removal.

b. Allow the balloon to drain into the syringe by gravity. By allowing the balloon to drain by gravity the development of creases or ridges in the balloon may be avoided and thus minimize trauma to the urethra during withdrawal. All patients who have a catheter removed should have their voiding monitored. The best way to do this is with a voiding record or bladder diary. The size syringe used to deflate the balloon is dictated by the size of the balloon. In the adult patient balloon sizes are either 10 mLs or 30 mLs. Catheters should be pulled out slowly and smoothly. There is no evidence to support clamping catheters prior to removal. c. Initiate a voiding record/bladder diary By allowing the balloon to drain by gravity the development of creases or ridges in the balloon may be avoided and thus minimize trauma to the urethra during withdrawal. All patients who have a catheter removed should have their voiding monitored. The best way to do this is with a voiding record or bladder diary. The size syringe used to deflate the balloon is dictated by the size of the balloon. In the adult patient balloon sizes are either 10 mLs or 30 mLs. Catheters should be pulled out slowly and smoothly. There is no evidence to support clamping catheters prior to removal.

A confused older adult patient is wearing thick glasses and a hearing aid. Which intervention is priority to facilitate communication? a. Focus on tasks to be completed. b. Allow time for the patient to respond. c. Limit conversations with the patient. d. Use gestures and other nonverbal cues.

b. Allow time for the patient to respond.

To prevent complications of immobility, what would be the most effective activity on the first postoperative day for a patient who has had abdominal surgery? a. Turn, cough, and deep breathe every 30 minutes while awake b. Ambulate patient to chair in the hall c. Passive range of motion 4 times a day d. Immobility is not a concern the first postoperative day

b. Ambulate patient to chair in the hall Prevention of complications of immobility begins when the patient becomes immobilized. Every 30 minutes is not necessary and disruptive to the healing process. Active patient participation in exercises is more beneficial to preventing venous stasis.

When a nurse is teaching a patient about how to administer an epinephrine injection in case of a severe allergic reaction, the nurse tells the patient to hold the injection like a dart. Which of the following instructional methods did the nurse use? a. Telling b. Analogy c. Demonstration d. Simulation

b. Analogy Analogies use familiar images when teaching to help explain complex information.

A patient who is receiving chemotherapy has inflamed gums and oral mucosa and painful sores in the mouth. Which of the following oral care actions are appropriate? (Select all that apply.) a. Decreasing frequency of oral hygiene b. Applying water-soluble moisturizing gel on the oral mucosa c. Encouraging intake of soft foods d. Using commercial mouthwash

b. Applying water-soluble moisturizing gel on the oral mucosa c. Encouraging intake of soft foods Oral care should be increased however alcohol based mouth wash can burn the patients mouth sores and cause pain. Soft foods should be encouraged.

A faculty member is reviewing a nursing student's plan of care, including the interventions the student provided for a patient with dementia. The student reviewed clinical guidelines on a professional website to identify interventions successful in reducing wandering in patients with dementia. The faculty member should evaluate which of the following? a. Number of interventions b. Appropriateness of the intervention for the patient c. The prior use of interventions by other nursing staff d. Correct application of the intervention for the patient care setting e. The time it takes to provide interventions

b. Appropriateness of the intervention for the patient d. Correct application of the intervention for the patient care setting In this situation the faculty member reviews the plan for the appropriateness of the intervention and its correct application. Because the nursing student selected proven interventions from a professional website, it is likely the interventions represent an accepted standard of care and meet the criteria of appropriateness. The number of interventions is not important. Whether an intervention has been used by other nurses is not important in the context of evaluating this nurse's plan of care. However, if other interventions are known to be effective for this patient, the student might choose to revise the plan later and add such interventions.

You are caring for a 65-year-old patient 2 days after surgery and helping him ambulate down the hallway. The surgeon has ordered exercise as tolerated. Your assessment indicates that the patient's heart rate at baseline is 88. After walking approximately 30 yards down the hallway, the heart rate is 110. What should be your next action? a. Stop exercise immediately and have him sit in a nearby chair. b. Ask him how he feels; determine if there is any discomfort or shortness of breath; and, if not, continue exercise. c. Tell him that he needs to walk further to reach a heart rate of 120. d. Have him walk slower; he has reached his maximum.

b. Ask him how he feels; determine if there is any discomfort or shortness of breath; and, if not, continue exercise. The patient's maximum heart rate with exercise should be 220 - 65 = 155. He is still in a safe range. An assessment of how the patient feels is good practice. The patient can safely continue to walk.

While completing an admission database, the nurse is interviewing a patient who states that he is allergic to latex. The most appropriate nursing action is to first a. Leave the room and place the patient in isolation. b. Ask the patient to describe the type of reaction. c. Proceed to the termination phase of the interview. d. Document the latex allergy on the medication administration record.

b. Ask the patient to describe the type of reaction.

Which of these patient scenarios is most indicative of critical thinking? a. Administering pain relief medication according to what was given last shift b. Asking a patient what pain relief methods, pharmacological and nonpharmacological, have worked in the past. c. Offering pain relief medication based on physician orders d. Explaining to the patient that his reports of severe pain are not consistent with the minor procedure that was performed

b. Asking a patient what pain relief methods, pharmacological and nonpharmacological, have worked in the past.

A nurse caring for a patient with heart failure instructs the patient on foods to eat for a low-sodium diet. The nurse will perform which of the following evaluation measures to determine success of her instruction? a. Patient weight b. Asking patient to identify three low-sodium foods to eat for lunch c. A calorie count of food d. Patient description of how food selections are made

b. Asking patient to identify three low-sodium foods to eat for lunch If the nurse is instructing patient on foods to eat, the goal and expected outcome would be knowledge based. Asking the patient to identify three low-sodium foods to eat for lunch is an evaluative measure for knowledge application.

The student nurse is teaching a family member the importance of foot care for his or her mother, who has diabetes. Which safety precautions are important for the family member to know to prevent infection? a. Cut nails frequently. b. Assess skin for redness, abrasions, and open areas daily. c. Soak feet in water at least 10 minutes before nail care. d. Apply lotion to feet daily. e. Clean between toes after bathing.

b. Assess skin for redness, abrasions, and open areas daily. Because of a patient's risk for infection, it is important to assess skin for redness, abrasions, and open areas daily. Apply lotion to feet daily to keep the skin hydrated, but do not leave excess lotion on the skin. Clean between toes carefully after bathing to avoid maceration. Do not cut nails or soak the feet of a patient with diabetes because this may create skin breakdown and open sores, leading to skin breakdown or infection.

The NAP reports to the nurse that a patient's catheter drainage bag has been empty for 4 hours. What is a priority nursing intervention? a. Implement the "as needed" order to irrigate the catheter. b. Assess the catheter and drainage tubing for obvious occlusion. c. Notify the health care provider immediately. d. Assess the vital signs and intake and output record.

b. Assess the catheter and drainage tubing for obvious occlusion. The priority nursing intervention is to ensure that there is not an occlusion in the catheter or drainage tubing.

A post-operative patient with a three-way indwelling urinary catheter and continuous bladder irrigation (CBI) complains of lower abdominal pain and distention. What should be the nurse's initial intervention? a. Increase the rate of the CBI b. Assess the intake and output c. Decrease the rate of the CBI d. Assess vital signs

b. Assess the intake and output An appropriate first action would be to assess the patency of the drainage system. Urine output in the drainage bag should be more than the volume of the irritant solution infused. If the system is not draining urine and irritant, the irritant should be stopped immediately, the catheter may be occluded and the bladder distended.

Which situation will require the nurse to obtain a telephone order? a. As the nurse and primary care provider leave a patient's room, the primary care provider gives the nurse an order. b. At 0100, a patient's blood pressure drops from 120/80 to 90/50 and the incision dressing is saturated with blood. c. At 0800, the nurse and primary care provider make rounds and the primary care provider tells the nurse a diet order. d. A nurse reads an order correctly as written by the primary care provider in the patient's medical record.

b. At 0100, a patient's blood pressure drops from 120/80 to 90/50 and the incision dressing is saturated with blood.

The patient has new-onset restlessness and confusion. His pulse rate is elevated, as is his respiratory rate. His oxygen saturation, however, is 94% according to the portable pulse oximeter. The nurse ignores the oximeter reading and calls the physician to obtain an order for an arterial blood gas (ABG). The nurse does this because many things can cause inaccurate pulse oximetry readings, including which of the following? (Select all that apply.) a. O2 saturations (SaO2) >70% b. Carbon monoxide inhalation c. Nail polish d. Hypothermia at the assessment site e. Intravascular dyes

b. Carbon monoxide inhalation c. Nail polish d. Hypothermia at the assessment site e. Intravascular dyes

Which of the following goals is most appropriate for a preoperative client with a nursing diagnosis of deficient knowledge regarding preoperative requirements related to lack of exposure to information? a. Client will understand the need for scheduled surgery before leaving the provider's office. b. Client will understand the preoperative routines of surgical care before leaving provider's office. c. Client will present for drawing of preoperative laboratory blood at least 48 hours before scheduled surgery. d. Client will be able to successfully accomplish the preoperative bowel preparation by morning of scheduled surgery.

b. Client will understand the preoperative routines of surgical care before leaving provider's office. Understanding the need for the surgery is not as directly related to preoperative requirements as is the understanding of preoperative routines. The remaining options are client outcomes.

If an infectious disease can be transmitted directly from one person to another, it is a: a. Port of entry to a host. b. Communicable disease. c. Susceptible host. d. Port of exit from the reservoir.

b. Communicable disease.

Which of the following may cause Clostridium difficile infection? a. Chronic laxative use b. Contact with C. difficile bacteria c. Overuse of antibiotics d. Frequent episodes of diarrhea caused by food intolerance e. Inflammation of the bowel

b. Contact with C. difficile bacteria These are the two main causes of C. difficile infection. c. Overuse of antibiotics These are the two main causes of C. difficile infection.

Which is the most likely means of transmitting infection between patients? a. Exposure to another patient's cough b. Contact with a health care worker's hands c. Sharing equipment among patients d. Disposing of soiled linen in a shared linen bag

b. Contact with a health care worker's hands

The patient is diagnosed with athlete's foot (tinea pedis). The patient says that she is relieved because it is only athlete's foot, and it can be treated easily. The nurse explains that athlete's foot is a. Generally isolated to the feet and never recurs. b. Contagious and frequently recurs. c. Caused by the papillomavirus. d. Treated with salicylic acid or electrodesiccation.

b. Contagious and frequently recurs.

A patient of Middle Eastern descent has lost 5 lbs during hospitalization and states that the food offered is not allowed in his diet owing to religious preferences. Based on this information, an appropriate nursing diagnostic statement is Imbalanced nutrition: less than body requirements related to a. Religious preferences. b. Decreased oral intake. c. Weight loss. d. Race and ethnicity.

b. Decreased oral intake.

A patient is diagnosed with methicillin-resistant Staphylococcus aureus (MRSA) pneumonia. Which type of isolation precaution is most appropriate for this patient? (Select all that apply) a. Reverse isolation b. Droplet precautions c. Standard precautions d. Contact precautions

b. Droplet precautions The patient has a multidrug resistant organism within his respiratory tract that has become pathogenic. The route of transmission for this type of condition is respiratory; thus whenever the patient coughs or sneezes, organisms are sprayed into the air and then drop onto surfaces in the room. In addition to gown and gloves, a mask must also be worn. d. Contact precautions The patient has a multidrug resistant organism within his respiratory tract that has become pathogenic. The route of transmission for this type of condition is respiratory; thus whenever the patient coughs or sneezes, organisms are sprayed into the air and then drop onto surfaces in the room. In addition to gown and gloves, a mask must also be worn.

The nurse observes all wounds closely. At what time is the risk of hemorrhage the greatest for surgical wounds? a. 7 days after surgery, when the client is more active. b. During the first 24 to 48 hours after surgery. c. Between 60 and 72 hours after surgery. d. Between 48 and 60 hours after surgery.

b. During the first 24 to 48 hours after surgery.

The nurse is caring for a school-aged child who has injured his leg after a bicycle accident. To determine whether the child is experiencing a localized inflammatory response, the nurse should assess for which of these signs and symptoms? a. Dizziness and disorientation to time, date, and place b. Edema, redness, tenderness, and loss of function c. Chest pain, shortness of breath, and nausea and vomiting d. Fever, malaise, anorexia, and nausea and vomiting

b. Edema, redness, tenderness, and loss of function

A home health nurse visits a 42-year-old woman with diabetes who has a recurrent foot ulcer. The ulcer has prevented the woman from working for over 2 weeks. The patient has had diabetes for 10 years. The ulcer has not been healing; it has drainage with a foul-smelling odor. As the nurse examines the patient, she learns that the patient is not following the ordered diabetic diet. Which of the following is considered a low-priority goal for this patient? a. Achieving wound healing of the foot ulcer b. Enhancing patient knowledge about the effects of diabetes c. Providing a dietitian consultation for diet retraining d. Improving patient adherence to diabetic diet

b. Enhancing patient knowledge about the effects of diabetes The high priority for this patient is wound healing. If the ulcer is left untreated, it will cause more serious harm; an infection is likely, and it could spread. Providing a diet consultation is an intervention. Improving patient adherence to her diet is an intermediate outcome. Adherence to the diet is important but not life threatening when unmet. Since the patient has had diabetes for 10 years, enhancing knowledge is important because of her poor adherence but a lower priority than the others.

The charge nurse is reviewing a patient's plan of care, which includes the nursing diagnostic statement, Impaired physical mobility related to tibial fracture as evidenced by patient's inability to ambulate to bathroom. The nurse needs to revise which part of the diagnostic statement? a. Nursing diagnosis b. Etiology c. Patient chief complaint d. Defining characteristic

b. Etiology

A nurse is getting ready to discharge to home a patient who has a nursing diagnosis of Impaired physical mobility. Before discontinuing the patient's plan of care, what does the nurse need to do? a. Determine whether the patient has transportation to get home. b. Evaluate whether patient goals and outcomes have been met. c. Establish whether the patient has a follow-up appointment scheduled. d. Ensure that the patient's prescriptions have been filled.

b. Evaluate whether patient goals and outcomes have been met.

A patient is being discharged after treatment for colitis (inflammation of the colon). The patient has had no episodes of diarrhea or abdominal pain for 24 hours. Following instruction, the patient identified correctly the need to follow a low-residue diet and the types of food to include if a bout of diarrhea develops at home. These behaviors are examples of: a. Evaluative measures. b. Expected outcomes. c. Reassessments. d. Standards of care.

b. Expected outcomes. The absence of diarrhea and abdominal pain and the ability to identify the correct diet are expected outcomes. If outcomes had not been met, the nurse would reassess. The low-residue diet is a standard of care, but the patient's ability to describe it is an outcome. An evaluative measure is the nurse questioning the patient about symptoms.

A patient continues to report postsurgical incision pain at a level of 9 out of 10 after pain medicine is given. It is 11 am and the next dose of pain medicine is not due for another hour. What should the critically thinking nurse do first? a. Explain to the patient that nothing else has been ordered. b. Explore other options for pain relief. c. Offer to notify the health care provider when he comes for evening follow up. d. Discuss the surgical procedure and reason for the pain.

b. Explore other options for pain relief.

A patient needs to learn how to administer a subcutaneous injection. Which of the following reflects that the patient is ready to learn? a. Describing difficulties a family member has had in taking insulin b. Expressing the importance of learning the skill correctly c. Being able to see and understand the markings on the syringe d. Having the dexterity needed to prepare and inject the medication

b. Expressing the importance of learning the skill correctly Patients are ready to learn when they understand the importance of learning and are motivated to learn.

At 1200 the registered nurse (RN) says to the nursing assistive personnel (NAP), "You did a good job walking Mrs. Taylor by 0930. I saw that you recorded her pulse before and after the walk. I saw that Mrs. Taylor walked in the hallway barefoot. For safety, the next time you walk a patient, you need to make sure that the patient wears slippers or shoes. Please walk Mrs. Taylor again by 1500." Which characteristics of positive feedback did the RN use when talking to the nursing assistant? a. Feedback is given immediately. b. Feedback focuses on one issue. c. Feedback offers concrete details. d. Feedback identifies ways to improve. e. Feedback focuses on changeable things. f. Feedback is specific about what is done incorrectly only.

b. Feedback focuses on one issue. c. Feedback offers concrete details. d. Feedback identifies ways to improve. e. Feedback focuses on changeable things. These are characteristics of positive feedback. The other options (1 and 6) are not appropriate because the RN did not provide feedback immediately (the NAP performed the task in the morning, but the feedback was not given until the afternoon) and you should give both positive feedback and feedback to improve the incorrectly done tasks.

What nursing intervention decreases the risk for catheter associated urinary tract infection (CAUTI)? a. Cleanse the urinary meatus 3-4 times daily with antiseptic solution. b. Hang the urinary drainage bag below the level with the bladder. c. Empty the urinary drainage bag daily. d. Irrigate the urinary catheter with sterile water.

b. Hang the urinary drainage bag below the level with the bladder. Evidenced based interventions shown to decrease the risk for CAUTI include ensuring that there is a free flow of urine from the catheter to the drainage bag.

The nurse is caring for a patient who has head lice (pediculosis capitis). The nurse knows that in treating this condition, one must understand that a. Regular shampoo products are most effective treatment. b. Head lice may spread to furniture and other people. c. Treatment must be repeated every 7 to 10 days. d. Manual removal is not a realistic option as treatment.

b. Head lice may spread to furniture and other people.

For a patient who has a muscle sprain, localized hemorrhage, or hematoma, which wound care product helps prevent edema formation, control bleeding, and anesthetize the body part? a. Binder b. Ice bag c. Elastic bandage d. Absorptive dressing

b. Ice bag An ice bag helps to constrict excess fluid in tissues, which prevents edema. The blood vessels become constricted, help to control bleeding, and can decrease pain where the ice bag is placed.

A patient has trouble speaking words, and the patient's speech is garbled. Which nursing diagnosis is most appropriate for this patient? a. Hopelessness b. Impaired verbal communication c. Hearing loss d. Self-care deficit

b. Impaired verbal communication

The nurse evaluates that the NAP has applied a patient's sequential compression device (SCD) appropriately when which of the following is observed? a. Initial patient measurement is made around the calves b. Inflation pressure averages 40 mm Hg c. Patient's leg placed in SCD sleeve with back of knee aligned with popliteal opening on the sleeve. d. Stockings are removed every 2 hours during application. e. Yellow light indicates SCD device is functioning (SELECT ALL THAT APPLY)

b. Inflation pressure averages 40 mm Hg The most effective way to prevent deep vein thrombosis is through an aggressive program of prophylaxis. A properly functioning SCD inflates with a pressure around 40 mm Hg. Inflation pressure averages 40 mm Hg, and the patient's leg should be placed in the SCD sleeve with the back of knee aligned with the popliteal opening on the sleeve. Measurement involves length of leg, not calf. A green light indicates the SCD device is functioning. c. Patient's leg placed in SCD sleeve with back of knee aligned with popliteal opening on the sleeve. The most effective way to prevent deep vein thrombosis is through an aggressive program of prophylaxis. A properly functioning SCD inflates with a pressure around 40 mm Hg. Inflation pressure averages 40 mm Hg, and the patient's leg should be placed in the SCD sleeve with the back of knee aligned with the popliteal opening on the sleeve. Measurement involves length of leg, not calf. A green light indicates the SCD device is functioning.

The nurse is organizing a disease prevention program for a specific cultural group. To effectively meet the needs of this group the nurse will: a. Assess the needs of the community in general. b. Involve those affected by the problem in the planning process. c. Develop generalized goals and objectives for the program. d. Use educational materials that are simplistic and have many pictures. e. Assess commonly held health beliefs among the cultural group. f. Educate the specific cultural group about Western concepts of health and illness. g. Include cultural practices that are relevant to the specific community.

b. Involve those affected by the problem in the planning process. e. Assess commonly held health beliefs among the cultural group. g. Include cultural practices that are relevant to the specific community. When planning a program, it is important to involve those affected by the problem to better serve the group's needs. Learning about traditional health practices/beliefs within the community enhances the nurse's understanding of a specific cultural group. Effective educational strategies require the nurse to use culturally tailored, interactive instructional methods that involve the family and community in the learning process.

Which of the following actions by the nurse comply with core principles of surgical asepsis? a. Set up sterile field before patient and other staff come to the operating suite. b. Keep the sterile field in view at all times. c. Consider the outer 2.5 cm (1 inch) of the sterile field as contaminated. d. Only health care personnel within the sterile field must wear personal protective equipment. e. The sterile gown must be put on before the surgical scrub is performed.

b. Keep the sterile field in view at all times. Keeping the sterile field in view at all times confirms that no contamination has occurred. The outer 2.5-cm (1-inch) of the sterile field is the most likely place for accidental contamination. The sterile table should be set up after the patient and staff are in the room to prevent a higher risk of contamination of the sterile field by air current. All surgical personnel will be wearing protective personal equipment in the surgical suite, not just those within the sterile field. The sterile gown and gloves are donned after the surgical scrub. c. Consider the outer 2.5 cm (1 inch) of the sterile field as contaminated. Keeping the sterile field in view at all times confirms that no contamination has occurred. The outer 2.5-cm (1-inch) of the sterile field is the most likely place for accidental contamination. The sterile table should be set up after the patient and staff are in the room to prevent a higher risk of contamination of the sterile field by air current. All surgical personnel will be wearing protective personal equipment in the surgical suite, not just those within the sterile field. The sterile gown and gloves are donned after the surgical scrub.

There is no urine when a catheter is inserted into a female's urethra. What should the nurse do next? a. Remove the catheter and start all over with a new kit and catheter. b. Leave the catheter there and start over with a new catheter. c. Pull the catheter back and re-insert at a different angle. d. Ask the patient to bear down and insert the catheter further.

b. Leave the catheter there and start over with a new catheter. The catheter may be in the vagina, leave the catheter in the vagina as landmark indicating where not to insert, and insert another sterile catheter. Pulling the catheter back and re-inserting is poor technique increasing the risk for CAUTI.

The nurse is caring for a patient with leukemia and is preparing to provide fluids through a vascular access device. Which nursing intervention is priority in this procedure? a. Position the patient comfortably. b. Maintain aseptic technique. c. Gather available supplies. d. Review the procedure with the patient.

b. Maintain aseptic technique. Patients with leukemia have low immunity, when administering medication through an IV the nurse must be sure not to introduce infection through the proper use of aseptic technique.

A patient has the nursing diagnosis of Nausea. The nurse develops a care plan with the following interventions. Which are examples of collaborative interventions? a. Providing mouth care every 4 hours b. Maintaining intravenous (IV) infusion at 100 mL/hr c. Administering prochlorperazine (Compazine) via rectal suppository d. Consulting with dietitian on initial foods to offer patient e. Controlling aversive odors or unpleasant visual stimulation that triggers nausea

b. Maintaining intravenous (IV) infusion at 100 mL/hr Providing mouth care every 4 hours and controlling aversive odors or unpleasant visual stimuli that triggers nausea are both independent nursing interventions. Administering prochlorperazine via suppository is a dependent intervention. d. Consulting with dietitian on initial foods to offer patient Providing mouth care every 4 hours and controlling aversive odors or unpleasant visual stimuli that triggers nausea are both independent nursing interventions. Administering prochlorperazine via suppository is a dependent intervention.

A nurse is visiting a patient in the home and is assessing the patient's adherence to medications. While talking with the family caregiver, the nurse learns that the patient has been missing doses. The nurse wants to perform interventions to improve the patient's adherence. Which of the following will affect how this nurse will make clinical decisions about how to implement care for this patient? (Select all that apply) a. Reviewing the family caregiver's availability during medication administration times b. Making a judgment of the value of improved adherence for the patient c. Reviewing the number of medications and time each is to be taken d. Determining all consequences associated with the patient missing specific medicines e. Reviewing the therapeutic actions of the medications

b. Making a judgment of the value of improved adherence for the patient Tips for making good clinical decisions during implementation include making a judgment of the value of the consequence to the patient, reviewing all possible consequences associated with each nursing action, determining the probability of all possible consequences, and reviewing the set of all possible nursing interventions for a patient's problems. d. Determining all consequences associated with the patient missing specific medicines Tips for making good clinical decisions during implementation include making a judgment of the value of the consequence to the patient, reviewing all possible consequences associated with each nursing action, determining the probability of all possible consequences, and reviewing the set of all possible nursing interventions for a patient's problems.

A client with a history of sleep apnea has had a same-day surgery procedure that will require the administration of morphine postoperatively to manage pain. This client will be assessed most appropriately by the perioperative nurse for the risk for respiratory complications by frequently: a. Listening to breath sounds b. Monitoring pulse oximetry c. Evaluating spirometer use d. Counting respirations per minute

b. Monitoring pulse oximetry Administration of opioids increases risk for airway obstruction postoperatively. Clients will desaturate as revealed by a drop in oxygen saturation by pulse oximetry. The remaining options are not as specific for this particular client's risk.

While the nurse is assessing the patient's respirations, it is important for the patient to a. Be aware of the procedure being done. b. Not know that respirations are being assessed. c. Understand that respirations are estimated to save time. d. Not be touched until the entire process is finished.

b. Not know that respirations are being assessed. If a patient knows respirations are being assessed they are more likely to alter their breathing patterns.

Which of the following factors does a nurse consider in setting priorities for a patient's nursing diagnoses? a. Numbered order of diagnosis on the basis of severity b. Notion of urgency for nursing action c. Symptom pattern recognition suggesting a problem d. Mutually agreed on priorities set with patient e. Time when a specific diagnosis was identified

b. Notion of urgency for nursing action These three factors are considered in setting priorities for a patient's nursing diagnoses or collaborative problems. The other options are inappropriate because a numbering system and time of identification hold little meaning when a patient's condition changes. c. Symptom pattern recognition suggesting a problem These three factors are considered in setting priorities for a patient's nursing diagnoses or collaborative problems. The other options are inappropriate because a numbering system and time of identification hold little meaning when a patient's condition changes. d. Mutually agreed on priorities set with patient These three factors are considered in setting priorities for a patient's nursing diagnoses or collaborative problems. The other options are inappropriate because a numbering system and time of identification hold little meaning when a patient's condition changes.

Which of the following are components of interprofessional collaboration? (Select all that apply) a. Interprofessional education does not impact the collaboration among interprofessional team members. b. Nurses are often viewed as the team leader because of their coordination of patient care. c. Effective interprofessional collaboration requires mutual respect and trust from all team members. d. Open communication improves the collaboration among the interprofessional team members. e. The goal of interprofessional collaboration is to improve the quality of patient care.

b. Nurses are often viewed as the team leader because of their coordination of patient care. c. Effective interprofessional collaboration requires mutual respect and trust from all team members. d. Open communication improves the collaboration among the interprofessional team members. e. The goal of interprofessional collaboration is to improve the quality of patient care. The nurse plays a critical role within the team and is often viewed as the team leader through coordination of communication and patient care. Open communication, cooperation, trust, mutual respect, and understanding of team member roles and responsibilities are critical for successful interprofessional collaboration. The development of these competencies comes through interprofessional education. A change in education and team training of health care practitioners is needed to build effective teams to improve interprofessional collaboration.

Which of the following is an indication for a binder to be placed around a surgical patient with a new abdominal wound? a. Collection of wound drainage b. Provides support to abdominal tissues when coughing or walking c. Reduction of abdominal swelling d. Reduction of stress on the abdominal incision e. Stimulation of peristalsis (return of bowel function) from direct pressure

b. Provides support to abdominal tissues when coughing or walking A binder placed over the abdomen can provide protection to the abdominal incision by offering support and decreasing stress from coughing and movement. d. Reduction of stress on the abdominal incision A binder placed over the abdomen can provide protection to the abdominal incision by offering support and decreasing stress from coughing and movement.

A nurse is talking with a young-adult patient about the purpose of a new medication. The nurse says, "I want to be clear. Can you tell me in your words the purpose of this medicine?" This exchange is an example of which element of the transactional communication process? a. Message b. Obtaining feedback c. Channel d. Referent

b. Obtaining feedback In this example the nurse's question is a way to obtain feedback. Feedback is the message a receiver receives from the sender. It indicates whether the receiver, in this case the patient, understood the meaning of the sender's message.

Which type of interview question does the nurse first use when assessing the reason for a patient seeking health care? a. Probing b. Open-ended c. Problem-oriented d. Confirmation

b. Open-ended The best interview question for initially determining why a patient is seeking health care is by asking an open-ended question that allows the patient to tell his or her story. This is also a more patient-centered approach. Probing questions are asked after data are gathered to seek more in-depth information. Problem-oriented and confirmation are not types of interview questions.

During the initial home visit, a home health nurse lets the patient know that the visits are expected to end in about a month. The nurse is in which phase of the helping relationship? a. Pre-interaction b. Orientation c. Working d. Termination

b. Orientation

Two patient deaths have occurred on a medical unit in the last month. The staff notices that everyone feels pressured and team members are getting into more arguments. As a nurse on the unit, what will best help you manage this stress? a. Keep a journal b. Participate in a unit meeting to discuss feelings about the patient deaths c. Ask the nurse manager to assign you to less difficult patients d. Review the policy and procedure manual on proper care of patients after death

b. Participate in a unit meeting to discuss feelings about the patient deaths By connecting and meeting with staff colleagues, you can talk about the experiences of caring for dying patients and learn that your feelings are likely shared by others. A journal is helpful but not the best way to relieve stress. A policy and procedure manual will not help you examine and understand the nature of the stress. Asking for a different assignment is no guarantee that another stressful experience will not develop.

A family member is providing care to a loved one who has an infected leg wound. What would you instruct the family member to do after providing care and handling contaminated equipment or organic material? a. Have the family member check with the doctor about need for immunization. b. Perform hand hygiene after care and/or handling contaminated equipment or material. c. Wear gloves before eating or handling food. d. Place any soiled materials into a bag and double bag it.

b. Perform hand hygiene after care and/or handling contaminated equipment or material.

After completing a thorough database and analyzing the data to identify any problems, the nurse should proceed to what step of the nursing process? a. Assessment b. Planning c. Implementation d. Evaluation

b. Planning

A client scheduled for an ambulatory surgery procedure requiring anesthetics arrives with a low-grade fever and a productive sough. The postponement of the procedure is most likely a result of the: a. Client's increased risk for a respiratory tract infection b. Possibility of a respiratory complication during anesthesia c. Increased risk for the client's infecting staff and other clients d. Client's impaired resistance as a result of a respiratory tract infection

b. Possibility of a respiratory complication during anesthesia Cough and low-grade fever increases the risk for respiratory complications during anesthesia (e.g., pneumonia and spasm of laryngeal muscles). Although the other options are not incorrect, they do not represent the most likely risk factor that would result in the cancellation of the procedure.

The nursing diagnosis Impaired Parenting related to mother's developmental delay is an example of a(n): a. Risk nursing diagnosis. b. Problem-focused nursing diagnosis. c. Health promotion nursing diagnosis. d. Wellness nursing diagnosis.

b. Problem-focused nursing diagnosis. This is an example of a problem-focused nursing diagnosis with a related factor, based on NANDA-I diagnostic terminology. Most health promotion diagnoses do not have established related factors based on NANDA-I; their use is optional. Wellness diagnoses are not one of the types of NANDA-I diagnoses.

The nurse uses silence as a therapeutic communication technique. What is the purpose of the nurse's silence? a. Prevent the nurse from saying the wrong thing b. Prompt the patient to talk when he or she is ready c. Allow the patient time to think and gain insight d. Allow time for the patient to drift off to sleep e. Determine if the patient would prefer to talk with another staff member

b. Prompt the patient to talk when he or she is ready c. Allow the patient time to think and gain insight Silence can provide the patient an opportunity to think and gain insight. Often the patient feels compelled to break the silence and is prompted to talk.

A patient is admitted for dehydration caused by pneumonia and shortness of breath. He has a history of heart disease and cardiac dysrhythmias. The nursing assistant reports his admitting vital signs to the nurse. Which measurements should the nurse reassess? a. Right arm BP: 118/72 b. Radial pulse rate: 72 and irregular c. Temporal temperature: 37.4° C (99.3° F) d. Respiratory rate: 28 e. Oxygen saturation: 99%

b. Radial pulse rate: 72 and irregular Irregular pulse and elevated respiratory rate are outside of expected values and require further assessment by the nurse. Pneumonia and shortness of breath can cause low oxygen saturation; an assessment of 99% may be a false-high value. Blood pressure and temperature are within expected values for the patient history. d. Respiratory rate: 28 Irregular pulse and elevated respiratory rate are outside of expected values and require further assessment by the nurse. Pneumonia and shortness of breath can cause low oxygen saturation; an assessment of 99% may be a false-high value. Blood pressure and temperature are within expected values for the patient history. e. Oxygen saturation: 99% Irregular pulse and elevated respiratory rate are outside of expected values and require further assessment by the nurse. Pneumonia and shortness of breath can cause low oxygen saturation; an assessment of 99% may be a false-high value. Blood pressure and temperature are within expected values for the patient history.

Nurses must communicate effectively with the health care team for which of the following reasons? a. Improve the nurse's status with the health team members b. Reduce the risk of errors to the patient c. Provide optimum level of patient care d. Improve patient outcomes e. Prevent issues that need to be reported to outside agencies

b. Reduce the risk of errors to the patient c. Provide optimum level of patient care d. Improve patient outcomes Effective communication in health care has been linked to a decrease in medical errors and an improvement in quality of care and patient outcomes. The status of the nurse or reportable issues are not the focus of communication with patients.

A nurse is recovering a patient who received conscious sedation for cosmetic surgery. Which of the following is an advantage that conscious sedation has over general anesthesia? a. Loss of sensation at the surgical site b. Reduction of fear and anxiety c. Amnesia about procedure d. Monitoring in phase I recovery e. Close monitoring for airway patency

b. Reduction of fear and anxiety Conscious sedation offers adequate sedation, reduction of fear and anxiety, amnesia, and relief of pain while maintaining airway patency and ventilation independently along with stable vital signs and rapid recovery. Loss of sensation at the surgical site is an effect of local anesthesia. These patients usually only go through phase II recovery. c. Amnesia about procedure Conscious sedation offers adequate sedation, reduction of fear and anxiety, amnesia, and relief of pain while maintaining airway patency and ventilation independently along with stable vital signs and rapid recovery. Loss of sensation at the surgical site is an effect of local anesthesia. These patients usually only go through phase II recovery.

Immobilized patients frequently have hypercalcemia, placing them at risk for a. Osteoporosis b. Renal calculi c. Pressure ulcers d. Thrombus formation

b. Renal calculi

The nursing assistive personnel (NAP) reports to you that the blood pressure (BP) of the patient in Question 11 is 140/76 on the left arm and 128/72 on the right arm. What actions do you take on the basis of this information? a. Notify the health care provider immediately b. Repeat the measurements on both arms using a stethoscope c. Ask the patient if she has taken her blood pressure medications recently d. Obtain blood pressure measurements on lower extremities e. Verify that the correct cuff size was used during the measurements f. Review the patient's record for her baseline vital signs g. Compare right and left radial pulses for strength

b. Repeat the measurements on both arms using a stethoscope The systolic BP measurements are significantly different and may reflect the vascular and muscular changes caused by the stroke. However, unexpected findings require reassessment by the nurse with a comparison to previous values. It is premature to notify the provider; differences are not caused by medications; inappropriate cuff size would reflect similar systolic pressures; pulse strength would be similar for these BP measurements. f. Review the patient's record for her baseline vital signs The systolic BP measurements are significantly different and may reflect the vascular and muscular changes caused by the stroke. However, unexpected findings require reassessment by the nurse with a comparison to previous values. It is premature to notify the provider; differences are not caused by medications; inappropriate cuff size would reflect similar systolic pressures; pulse strength would be similar for these BP measurements.

An older adult has limited mobility as a result of a surgical repair of a fracture hip. During assessment you note that the patient cannot tolerate lying flat. Which of the following assessment data support a possible pulmonary problem related to impaired mobility? (Select all that apply.) a. B/P = 128/84 b. Respirations 26 per minute on room air c. HR 114 d. Crackles heard on auscultation e. Pain reported as 3 on scale of 0 to 10 after medication

b. Respirations 26 per minute on room air c. HR 114 d. Crackles heard on auscultation

An older adult has limited mobility as a result of a total knee replacement. During assessment you note that the patient has difficulty breathing while lying flat. Which of the following assessment data support a possible pulmonary problem related to impaired mobility? a. B/P = 128/84 b. Respirations 26/min on room air c. HR 114 d. Crackles over lower lobes heard on auscultation e. Pain reported as 3 on scale of 0 to 10 after medication

b. Respirations 26/min on room air Patients who are immobile are at high risk for developing pulmonary complications. The most common respiratory complications are atelectasis (collapse of alveoli) and hypostatic pneumonia (inflammation of the lung from stasis or pooling of secretions). Ultimately the distribution of mucus in the bronchi increases, particularly when the patient is in the supine, prone, or lateral position. c. HR 114 Patients who are immobile are at high risk for developing pulmonary complications. The most common respiratory complications are atelectasis (collapse of alveoli) and hypostatic pneumonia (inflammation of the lung from stasis or pooling of secretions). Ultimately the distribution of mucus in the bronchi increases, particularly when the patient is in the supine, prone, or lateral position. d. Crackles over lower lobes heard on auscultation Patients who are immobile are at high risk for developing pulmonary complications. The most common respiratory complications are atelectasis (collapse of alveoli) and hypostatic pneumonia (inflammation of the lung from stasis or pooling of secretions). Ultimately the distribution of mucus in the bronchi increases, particularly when the patient is in the supine, prone, or lateral position.

A nurse is caring for a complicated patient 3 days in a row. The nurse attends an interdisciplinary conference to discuss the patient's plan of care. In which ways can the nurse develop trust with members of the conference team? a. Is willing to challenge other members' ideas because the nurse disagrees with their rationale b. Shows competence in how to monitor patients' clinical status and inform the physician of critical changes c. Asks a more experienced nurse to attend the conference d. Listens to opinions of members of interdisciplinary team and expresses recommendations for care clearly e. During the meeting focus on similar problems the nurse has had in delivering care to other patients.

b. Shows competence in how to monitor patients' clinical status and inform the physician of critical changes Showing competence and exercising effective communication are important for developing trust with interdisciplinary team members. Having another nurse attend the conference who might be less familiar with the patient would not promote trust. Challenging other ideas just because of disagreement does not foster trust. Changing the focus from the patient to the problems of the nurse will not foster trust. d. Listens to opinions of members of interdisciplinary team and expresses recommendations for care clearly Showing competence and exercising effective communication are important for developing trust with interdisciplinary team members. Having another nurse attend the conference who might be less familiar with the patient would not promote trust. Challenging other ideas just because of disagreement does not foster trust. Changing the focus from the patient to the problems of the nurse will not foster trust.

What is the proper position to use for an unresponsive patient during oral care to prevent aspiration? a. Prone position b. Sims position c. Semi-fowler's position with head to side d. Trendelenburg position e. Supine position

b. Sims position Place the unconscious patient in semi-Fowler's position with head to the side or use the Sim's position to help avoid aspiration while performing oral care. The supine and Trendelenburg positions would make it easier for a patient to aspirate. The prone position would not be suitable for accessing the oral cavity. c. Semi-fowler's position with head to side Place the unconscious patient in semi-Fowler's position with head to the side or use the Sim's position to help avoid aspiration while performing oral care. The supine and Trendelenburg positions would make it easier for a patient to aspirate. The prone position would not be suitable for accessing the oral cavity.

Which of the following nursing diagnoses is stated correctly? a. Fluid Volume Excess related to heart failure b. Sleep Deprivation related to sustained noisy environment c. Impaired Bed Mobility related to postcardiac catheterization d. Ineffective Protection related to inadequate nutrition e. Diarrhea related to frequent, small, watery stools.

b. Sleep Deprivation related to sustained noisy environment The correct diagnoses of Sleep Deprivation and Ineffective Protection are worded with related factors that will respond to nursing interventions. Nursing interventions do not change a medical diagnosis or diagnostic test. Instead nurses direct nursing interventions at behaviors or conditions that they are able to treat or manage. The first two incorrect diagnoses use a medical diagnosis and diagnostic procedure respectively as related or etiological factors. These are not conditions that nursing interventions can treat. The last diagnosis is incorrect because it is related to an assessment finding of a symptom or a defining characteristic. d. Ineffective Protection related to inadequate nutrition The correct diagnoses of Sleep Deprivation and Ineffective Protection are worded with related factors that will respond to nursing interventions. Nursing interventions do not change a medical diagnosis or diagnostic test. Instead nurses direct nursing interventions at behaviors or conditions that they are able to treat or manage. The first two incorrect diagnoses use a medical diagnosis and diagnostic procedure respectively as related or etiological factors. These are not conditions that nursing interventions can treat. The last diagnosis is incorrect because it is related to an assessment finding of a symptom or a defining characteristic.

The nurse notes a client's skin is reddened with a small abrasion and serous fluid present. The nurse should classify this stage of ulcer formation as: a. Stage I b. Stage II c. Stage III d. Stage IV

b. Stage II

Prolonged, intense pressure affects cellular metabolism by decreasing or obliterating blood flow, which results in tissue ischemia and ultimately tissue death. There are four stages of pressure ulcer formation. The nurse observes partial-thickness skin loss involving the epidermis and possibly the dermis. What stage of ulcer will the nurse document? a. Stage I b. Stage II c. Stage III d. Stage IV

b. Stage II Partial-thickness skin loss involving the epidermis and possibly the dermis is classified as a stage II ulcer. In stage I the ulcer appears as a defined area of persistent redness in lightly pigmented skin or a darker red, blue, or purple area in darker pigmented skin, with no open skin areas. In stage III the ulcer appears as a full-thickness skin loss involving damage or necrosis of subcutaneous tissue that may extend down to, but not through, the underlying fascia. In stage IV the ulcer appears as a full-thickness loss with extensive destruction, tissue necrosis, or damage to muscle, bone, or supporting structures.

An ambulatory elderly woman with dementia is incontinent of urine. She has poor short term memory and has not been seen toileting independently. What is the best nursing intervention for this patient? a. Recommend she be evaluated for an OAB medication. b. Start a scheduled toileting program. c. Recommend she be evaluated for an indwelling catheter. d. Start a bladder retraining program

b. Start a scheduled toileting program. The first nursing intervention for any patient with incontinence, who is able to toilet, is to assist them with toilet access. This patient is not cognitively intact so a bladder retraining program is not appropriate for her. It is not clear in this case that she has OAB and a catheter is never a good solution for incontinence.

While reviewing the pulmonary assessment entered by a nurse in a patient's electronic medical record (EMR), a physician notices that the only information documented in that section is "WDL" (within defined limits). The physician also is not able to find a narrative description of the patient's respiratory status in the nurse's progress notes. What is the most likely reason for this? a. The nurse caring for the patient forgot to document on the pulmonary system. b. The EMR uses a charting-by-exception format. c. The computer shut down unexpectedly when the nurse was documenting the assessment. d. Because of HIPAA regulations, physicians are not authorized to view the nursing assessment.

b. The EMR uses a charting-by-exception format. Given that the initial assessment indicated that the pulmonary system was within normal limits, the facility is most likely documenting by exception. There is no need for further documentation unless the pulmonary assessment changes and is no longer within normal limits.

What does it mean when a patient is diagnosed with a multidrug-resistant organism in his or her surgical wound? a. There is more than one organism in the wound that is causing the infection. b. The antibiotics the patient has received are not strong enough to kill the organism. c. The patient will need more than one type of antibiotic to kill the organism. d. The organism has developed a resistance to one or more broad-spectrum antibiotics, indicating that the organism will be hard to treat effectively. e. There are no longer any antibiotic options available to treat the patient's infection.

b. The antibiotics the patient has received are not strong enough to kill the organism. Multidrug-resistant organisms are bacteria that have become resistant to certain antibiotics, and these antibiotics can no longer be used to control or kill the bacteria. d. The organism has developed a resistance to one or more broad-spectrum antibiotics, indicating that the organism will be hard to treat effectively. Multidrug-resistant organisms are bacteria that have become resistant to certain antibiotics, and these antibiotics can no longer be used to control or kill the bacteria.

The nurse enters a patient's room and finds that the patient was incontinent of liquid stool. Because the patient has recurrent redness in the perineal area, the nurse worries about the risk of the patient developing a pressure ulcer. The nurse cleanses the patient, inspects the skin, and applies a skin barrier ointment to the perineal area. The nurse consults the ostomy and wound care nurse specialist for recommended skin care measures. Which of the following correctly describe the nurse's actions? a. The application of the skin barrier is a dependent care measure. b. The call to the ostomy and wound care specialist is an indirect care measure. c. The cleansing of the skin is a direct care measure. d. The application of the skin barrier is an instrumental activity of daily living. e. Inspecting the skin in a direct care activity.

b. The call to the ostomy and wound care specialist is an indirect care measure. The call to the specialist is a referral and an indirect care measure on the patient's behalf. Cleansing of the skin is an example of direct care. Application of a skin barrier is an independent measure and it is not an instrumental activity of daily living. Inspecting the skin is assessment, not direct care. c. The cleansing of the skin is a direct care measure. The call to the specialist is a referral and an indirect care measure on the patient's behalf. Cleansing of the skin is an example of direct care. Application of a skin barrier is an independent measure and it is not an instrumental activity of daily living. Inspecting the skin is assessment, not direct care.

A 43-year-old client is scheduled to have a gastrectomy. Which of the following is a major preoperative concern? a. The client's brother had a tonsillectomy at age 11. b. The client smokes a pack of cigarettes a day. c. The client has an intravenous (IV) infusion. d. The client has a history of employment as a computer programmer.

b. The client smokes a pack of cigarettes a day. The client who smokes is at greater risk for postoperative pulmonary complications than a client who does not. An IV should be in place for surgery so access is available to administer medications, fluids, or blood products if necessary. Keeping the client well hydrated will help prevent postoperative thrombophlebitis.

A patient of any age can develop a contracture of a joint when: a. The adductors muscles are weakened as a result of immobility. b. The muscle fibers become shortened because of disuse. c. The calcium-to-phosphorus ratio becomes disrupted. d. There is a deficiency in vitamin D.

b. The muscle fibers become shortened because of disuse.

A preceptor is working with a new nurse on documentation. Which situation will cause the preceptor to intervene? a. The new nurse uses a black ink pen to chart. b. The new nurse charts consecutively on every other line. c. The new nurse ends each entry with signature and title. d. The new nurse keeps the password secure.

b. The new nurse charts consecutively on every other line.

When should a nurse wear a mask? (Select all that apply). a. The patient's dental hygiene is poor. b. The nurse is assisting with an aerosolizing respiratory procedure such as suctioning. c. The patient has acquired immunodeficiency syndrome (AIDS) and a congested cough. d. The patient is in droplet precautions. e. The nurse is assisting a health care provider in the insertion of a central line catheter.

b. The nurse is assisting with an aerosolizing respiratory procedure such as suctioning. Masks are used for three primary purposes in health care settings: (1) placed on health care personnel to protect them from contact with infectious material from patients (e.g., respiratory secretions); (2) placed on health care personnel when engaged in procedures requiring sterile technique to protect patients from exposure to infectious agents carried in a health care worker's mouth or nose; and (3) placed on coughing patients to limit potential dissemination of infectious respiratory secretions from the patient to others. d. The patient is in droplet precautions. Masks are used for three primary purposes in health care settings: (1) placed on health care personnel to protect them from contact with infectious material from patients (e.g., respiratory secretions); (2) placed on health care personnel when engaged in procedures requiring sterile technique to protect patients from exposure to infectious agents carried in a health care worker's mouth or nose; and (3) placed on coughing patients to limit potential dissemination of infectious respiratory secretions from the patient to others. e. The nurse is assisting a health care provider in the insertion of a central line catheter. Masks are used for three primary purposes in health care settings: (1) placed on health care personnel to protect them from contact with infectious material from patients (e.g., respiratory secretions); (2) placed on health care personnel when engaged in procedures requiring sterile technique to protect patients from exposure to infectious agents carried in a health care worker's mouth or nose; and (3) placed on coughing patients to limit potential dissemination of infectious respiratory secretions from the patient to others.

A nursing student is completing an assessment on an 80-year-old patient who is alert and oriented. The patient's daughter is present in the room. Which of the following actions made by the nursing student requires the nursing professor to intervene? a. The nursing student is making eye contact with the patient. b. The nursing student is speaking only to the patient's daughter. c. The nursing student nods periodically while the patient is speaking. d. The nursing student leans forward while talking with the patient.

b. The nursing student is speaking only to the patient's daughter.

A nurse is working in the preoperative holding area and is assigned to care for a patient who is having a prosthetic aortic valve placed. The nurse inserts an intravenous (IV) line and obtains vital signs. The patient has a temperature of 39° C (102° F), heart rate of 120, blood pressure (BP) of 84/50, and an elevated white blood cell (WBC) count. The nurse immediately notifies the surgeon of the patient's vital signs because: a. He or she needs to get the patient into the operating room (OR) quickly to start the surgery because of the low BP. b. The surgery may need to be delayed to recheck the patient's WBC count and investigate the source of fever before surgery. c. The nurse anticipates the need for a fluid bolus to increase the patient's BP. d. The nurse anticipates an order for a sedative to help calm the patient and decrease the heart rate.

b. The surgery may need to be delayed to recheck the patient's WBC count and investigate the source of fever before surgery. The patient has a fever, elevated WBC count, tachycardia, and hypotension, which are all signs of a potential infection. It may be necessary to delay the surgery until the source of the fever is treated.

A home care nurse is preparing the home for a patient who is discharged to home following a left-sided stroke. The patient is cooperative and can ambulate with a quad-cane. Which of the following must be corrected or removed for the patient's safety? (Select all that apply.) a. The rubber mat in the walk-in shower b. The three-legged stool on wheels in the kitchen c. The braided throw rugs in the entry hallway and between the bedroom and bathroom d. The night-lights in the hallways, bedroom, and bathroom e. The cordless phone next to the patient's bed

b. The three-legged stool on wheels in the kitchen c. The braided throw rugs in the entry hallway and between the bedroom and bathroom

Application of a warm compress is indicated: a. To protect bony prominences from pressure ulcers. b. To improve blood flow to an injured part. c. For a client who is shivering. d. To relieve edema.

b. To improve blood flow to an injured part. Warm compresses are used to improve blood flow to an affected part. Warm compresses are typically not used for edema relief. A warm compress will not necessarily help with shivering; extra blankets should be used instead. A warm compress will not protect from pressure ulcers.

An elderly patient is wearing a hearing aid. Which technique should the nurse use to facilitate communication? a. Speak clearly and loudly. b. Turn off the television. c. Chew gum. d. Use at least 14-point print.

b. Turn off the television.

When evaluating a plan of care, the nurse reviews the goals for the patient. Which goal statement is REALISTIC to assign to a patient with a pelvic fracture on bed rest? The patient will increase mobility by a. Ambulating in the hallway two times this shift. b. Turning side to back to side with assistance every 2 hours. c. Using the walker correctly to ambulate to the bathroom as needed. d. Using a sliding board correctly to transfer to the bedside commode as needed.

b. Turning side to back to side with assistance every 2 hours.

The patient requires routine temperature assessment but is confused and easily agitated and has a history of seizures. The nurse's best option would be to take his temperature a. Orally. b. Tympanically. c. Rectally. d. By the axillary method.

b. Tympanically. Avoid oral temperatures in patients with seizures. Avoid rectal temperatures unless necessary. The quickest way to take a patients temperature would be to use a tympanic temperature.

Obesity places patients at an increased surgical risk because of which of the following factors? a. Risk for bleeding is increased. b. Ventilatory capacity is reduced. c. Fatty tissue has a poor blood supply. d. Metabolic demands are increased. e. Physical mobility is often impaired.

b. Ventilatory capacity is reduced. A decreased blood supply in adipose tissue slows the delivery of essential nutrients, antibodies, and enzymes needed for wound healing. A decreased ventilatory capacity allows for alveolar collapse, which can lead to pneumonia. c. Fatty tissue has a poor blood supply. A decreased blood supply in adipose tissue slows the delivery of essential nutrients, antibodies, and enzymes needed for wound healing. A decreased ventilatory capacity allows for alveolar collapse, which can lead to pneumonia. e. Physical mobility is often impaired. A decreased blood supply in adipose tissue slows the delivery of essential nutrients, antibodies, and enzymes needed for wound healing. A decreased ventilatory capacity allows for alveolar collapse, which can lead to pneumonia.

The nurse is planning to teach a patient about the importance of exercise. When is the best time for teaching to occur? a. When there are visitors in the room b. When the patient states that he or she is pain free c. Just before lunch, when the patient is most awake and alert d. When the patient is talking about current stressors in his or her life e. When the patient is being transported for a procedure

b. When the patient states that he or she is pain free c. Just before lunch, when the patient is most awake and alert Plan teaching when the patient is most attentive, receptive, alert, and comfortable.

The nurse is caring for a patient who is having a seizure. Which of the following measures will protect the patient and the nurse from injury? a. If patient is standing, attempt to get him or her back in bed. b. With patient on floor, clear surrounding area of furniture or equipment. c. If possible, keep patient lying supine. d. Do not restrain patient; hold limbs loosely if they are flailing. e. Never force apart a patient's clenched teeth.

b. With patient on floor, clear surrounding area of furniture or equipment. d. Do not restrain patient; hold limbs loosely if they are flailing. e. Never force apart a patient's clenched teeth. During a seizure, if a patient is standing, guide to floor. Do not try to place in bed. Do not position the patient supine; instead turn patient onto one side with head tilted slightly. When patient is on the floor, remove any furniture or objects that he or she could strike during tonic and clonic activity. Never force apart a patient's clenched teeth; you might be bitten. Do not restrain patient; hold limbs loosely if they are flailing. A postictal phase follows the seizure, during which the patient has amnesia or confusion and falls into a deep sleep.

The nurse asks a patient, "Describe for me a typical night's sleep. What do you do to fall asleep? Do you have difficulty falling or staying asleep? This series of questions would likely occur during which phase of a patient-centered interview? a. Orientation b. Working phase c. Data validation d. Termination

b. Working phase The gathering of information is the working phase of a patient-centered interview.

A nursing student is working with a faculty member to identify a nursing diagnosis for an assigned patient. The student has assessed that the patient is undergoing radiation treatment and has had liquid stool and the skin is clean and intact; therefore she selects the nursing diagnosis Impaired Skin Integrity. The faculty member explains that the student has made a diagnostic error for which of the following reasons? a. Incorrect clustering b. Wrong diagnostic label c. Condition is a collaborative problem. d. Premature closure of clusters

b. Wrong diagnostic label The more appropriate nursing diagnosis for this patient would be Risk for Impaired Skin Integrity because the patient's skin is clean and intact. A risk nursing diagnosis is appropriate because the patient has two risk factors, radiation and secretions on the skin.

Anticipate how a patient might respond to a treatment. a. open-mindedness b. analyticity c. truth seeking d. systematicity

b. analyticity

If you use a child BP cuff, you anticipate that Mr. Johnson's BP would be: a. false low. b. false high. c. accurate. d. with high pulse pressure.

b. false high.

Left lower abdominal surgical incision, 3 inches in length, closed, sutures intact, no drainage. Pain noted on mild palpation. a. subjective b. objective c. assessment d. plan

b. objective

Category/Stage II pressure ulcer a. nonblanchable redness of intact skin. Discoloration, warmth, edema, or pain may also be present. b. partial thickness skin loss or intact blister with serosanginous fluid c. full thickness tissue loss, muscle and bone visible. may include undermining d. full thickness skin loss, subcutaneous fat may be visible. may include undermining

b. partial thickness skin loss or intact blister with serosanginous fluid

An 82-year-old patient who resides in a nursing home has the following three nursing diagnoses: Risk for Fall, Impaired Physical Mobility related to pain, and Imbalanced Nutrition: Less Than Body Requirements related to reduced ability to feed self. The nursing staff identified several goals of care. Match the goals on the left with the appropriate outcome statements on the right. Goals Outcomes (Patient will be injury free for 1 month.) a. patient walks 20 feet using a walker in 24 hrs. b. patient identifies barriers to remove in the home within 1 week c. patient expresses fewer nonverbal signs of discomfort within 24 hrs d. patient increases calorie intake to 2500 daily

b. patient identifies barriers to remove in the home within 1 week

Lifesaving measure _____ a. assisting patient with oral care b. protecting a violent patient from injury c. discussing a patient's options in choosing palliative care d. using safe patient handling during positioning of a patient

b. protecting a violent patient from injury

After you assess the vital signs of Mr. Johnson, which of his vital signs must you address and report to the nurse immediately: a. heart rate= 72 beats per minute b. respiration rate= 28 breaths per minute c. blood pressure= 135/87 mmHg d. Temperature= 37.2C (99F) Oral

b. respiration rate= 28 breaths per minute All VS normal except RR.

Knowing that Mr. Johnson has a history of heart failure and dysrhythmia, you expect his radial pulse to be: a. bounding, 4+, and regular. b. weak, 1+, and irregular. c. normal, 2+, and regular. d. Absent.

b. weak, 1+, and irregular.

The nurse is caring for a group of medical-surgical patients. The patient most at risk for developing an infection is the patient who a. Has been admitted for stabilization of atrial fibrillation. b. Is in observation for chest pain. c. Is recovering from a right total hip arthroplasty. d. Has been admitted with dehydration.

c. Is recovering from a right total hip arthroplasty.

A nursing student asks her nursing instructor to describe the primary purpose of evaluation. Which of the following statements made by the nursing instructor is most accurate? a. "During evaluation, you determine whether all nursing interventions were completed." b. "During evaluation, you determine when to downsize staffing on nursing units." c. "Nurses use evaluation to determine the effectiveness of nursing care." d. "Evaluation eliminates unnecessary paperwork and care planning."

c. "Nurses use evaluation to determine the effectiveness of nursing care."

A patient is evaluated in the emergency department after causing an automobile accident while being under the influence of alcohol. While assessing the patient, which statement would be the most therapeutic? a. "Why did you drive after you had been drinking?" b. "We have multiple patients to see tonight as a result of this accident." c. "Tell me what happened before, during, and after the automobile accident tonight." d. "It will be okay. No one was seriously hurt in the accident."

c. "Tell me what happened before, during, and after the automobile accident tonight." Focusing gives direction, which enables the nurse to obtain clearer information without probing. Asking "why" questions can convey judgment on the part of the nurse. Giving false reassurance is not a therapeutic communication technique.

When asked what the role of the skin is in maintaining homeostasis, the answer that reflects the greatest insight is: a. "Our body needs vitamin D, and without healthy skin we cannot utilize it into a form we can use." b. "Without skin we would not be able to enjoy the sense of touch that is so important to us as humans." c. "The skin is a barrier that is really quite good at keeping disease-causing pathogens from getting into our body." d. "It is the pain with its pain receptors that alert us to danger so that we can take appropriate action in order to be safe."

c. "The skin is a barrier that is really quite good at keeping disease-causing pathogens from getting into our body." Although it is a sensory organ for pain, temperature, and touch and synthesizes vitamin D, its primary role is that of a protective barrier against disease-causing organisms.

Which of the following statements made by the nurse shows the greatest insight into the need to manage the risk factors that contribute to the formation of a pressure ulcer? a. "Her diet needs to include more protein and less sugary foods." b. "She needs to be moved more gently and with attention to her skin." c. "We need to decrease the time she spends with the weight of her body resting on her hip" d. "The urinary incontinency makes the risk for developing a pressure ulcer so much greater for her."

c. "We need to decrease the time she spends with the weight of her body resting on her hip" Pressure is the major cause in pressure ulcer formation. Three pressure-related factors contribute to pressure ulcer development: (1) pressure intensity, (2) pressure duration, and (3) tissue tolerance. The remaining options, although related to contributing factors, do not address the primary factor, pressure.

A nurse is assigned to a 42-year-old mother of 4 who weighs 136.2 kg (300 lbs), has diabetes, and works part time in the kitchen of a restaurant. The patient is facing surgery for gallbladder disease. Which of the following approaches demonstrates the nurse's cultural competence in assessing the patient's health care problems? a. "I can tell that your eating habits have led to your diabetes. Is that right?" b. "It's been difficult for people to find jobs. Is that why you work part time?" c. "You have four children; do you have any concerns about going home and caring for them?" d. "I wish patients understood how overeating affects their health."

c. "You have four children; do you have any concerns about going home and caring for them?" This is the only assessment approach that is not biased or does not show judgment about the patient's weight or occupational status. With the other options, the nurse is reacting to the patient on the basis of personal stereotypes and biases.

What is the correct order in which elastic stockings should be applied?1. Identify patient using two identifiers. 2. Smooth any creases or wrinkles. 3. Slide the remainder of the stocking over the patient's heel and up the leg 4. Turn the stocking inside out until heel is reached. 5. Assess the condition of the patient's skin and circulation of the legs. 6. Place toes into foot of the stocking. 7. Use tape measure to measure patient's legs to determine proper stocking size. a. 1, 5, 7, 4, 6, 2, 3 b. 1, 7, 5, 4, 6, 2, 3 c. 1, 5, 7, 4, 6, 3, 2 d. 1, 5, 4, 7, 6, 3, 2

c. 1, 5, 7, 4, 6, 3, 2 This is the correct order in which elastic stockings should be applied.

A nurse obtained a telephone order from a primary care provider for a patient in pain. Which chart entry should the nurse document? a. 12/16/20XX 0915 Tylenol 3, 2 tablets, every 6 hours for incisional pain. VO Dr. Day/J. Winds, RN, read back. b. 12/16/20XX 0915 Tylenol 3, 2 tablets, every 6 hours for incisional pain. TO J. Winds, RN, read back. c. 12/16/20XX 0915 Tylenol 3, 2 tablets, every 6 hours for incisional pain. TO Dr. Day/J. Winds, RN, read back. d. 12/16/20XX 0915 Tylenol 3, 2 tablets, every 6 hours for incisional pain. TO J. Winds, RN.

c. 12/16/20XX 0915 Tylenol 3, 2 tablets, every 6 hours for incisional pain. TO Dr. Day/J. Winds, RN, read back.

The nurse observes a nursing student taking a blood pressure (BP) on a patient. The nurse notes that the student very slowly deflates the cuff in an attempt to hear the sounds. The patient's BP range over the past 24 hours is 132/64 to 126/72 mm Hg. Which of the following BP readings made by the student is most likely caused by an incorrect technique? a. 96/40 mm Hg b. 110/66 mm Hg c. 130/90 mm Hg d. 156/82 mm Hg

c. 130/90 mm Hg Deflating the cuff too slowly will result in a false-high diastolic blood pressure.

When calculating the daily intake and output, the nurse anticipates that the urinary output for an average adult should be: a. 800 to 1000 mL/day b. 1000 to 1200 mL/day c. 1500 to 1600 mL/day d. 2000 to 2300 mL/day

c. 1500 to 1600 mL/day Although output does depend on intake, the normal adult urine output is 1500 to 1600 mL/day.

The nurse is working in a postoperative care unit in an ambulatory surgery center. Of the following clients that have come to have surgery, the client at the greatest risk during surgery is a: a. 78-year-old taking an analgesic agent b. 43-year-old taking an antihypertensive agent c. 27-year-old taking an anticoagulant agent d. 10-year-old taking an antibiotic agent

c. 27-year-old taking an anticoagulant agent Anticoagulants alter normal clotting factors and thus increase the risk for hemorrhaging during surgery. Aminoglycosides (a type of antibiotic) may cause mild respiratory depression from depressed neuromuscular transmission; however, the client who has been taking anticoagulants is at greater risk during surgery.

You are caring for a patient who frequently tries to remove his intravenous catheter and feeding tube. You have an order from the health care provider to apply a wrist restraint. What is the correct order for applying a wrist restraint?1. Be sure that patient is comfortable with arm in anatomic alignment. 2. Wrap wrist with soft part of restraint toward skin and secure snugly.3. Identify patient using two identifiers.4. Introduce self and ask patient about his feelings of being restrained.5. Assess condition of skin where restraint will be placed. a. 4, 3, 5, 1, 2 b. 4, 3, 1, 5, 2 c. 3, 4, 1, 5, 2 d. 3, 4, 5, 1, 2

c. 3, 4, 1, 5, 2 These are the correct steps for applying a wrist restraint.

The nursing assistive personnel (NAP) informs you that the electronic blood pressure machine on the patient who has recently returned from surgery following removal of her gallbladder is flashing a blood pressure of 65/46 and alarming. What is the correct order for care activities? 1. Press the start button of the electronic blood pressure machine to obtain a new reading. 2. Obtain a manual blood pressure with a stethoscope. 3. Check the patient's pulse distal to the blood pressure cuff. 4. Assess the patient's mental status. 5. Remind the patient not to bend her arm with the blood pressure cuff. a. 5, 3, 1, 4, 2 b. 3, 2, 1, 4, 5 c. 4, 1, 3, 2, 5 d. 1, 2, 4, 3, 5 e. 2, 3, 1, 4, 5

c. 4, 1, 3, 2, 5 First priority is to verify that the patient's blood pressure is providing adequate blood flow to the brain and critical organs. Movement interferes with electronic blood pressure measurement; recycling the machine will obtain a blood pressure while you are assessing the patient. Check the distal pulse to verify circulation to the extremity and then obtain manual blood pressure if needed. Patient education can prevent false values and decrease patient anxiety with alarms.

When planning patient care, a goal can be described as a. A statement describing the patient's accomplishments without a time restriction. b. A realistic statement predicting any negative responses to treatments. c. A broad statement describing a desired change in patient behavior. d. An identified long-term nursing diagnosis.

c. A broad statement describing a desired change in patient behavior.

While planning morning care, which of the following patients would receive the highest priority to receive his or her bath first? a. A patient who just returned to the nursing unit from surgery and is experiencing pain at a level of 7 on a scale of 0 to 10 b. A patient who prefers a bath in the evening when his wife visits and can help him c. A patient who is experiencing frequent incontinent diarrheal stools d. A patient who has just returned from diagnostic testing and complains of being very fatigued

c. A patient who is experiencing frequent incontinent diarrheal stools Pain should be addressed before hygiene activities. Patients with incontinence are at risk for poor hygiene and skin breakdown and should be addressed first.

While planning morning care, which of the following patients would have the highest priority to receive his or her bath first? a. A patient who just returned to the nursing unit from a diagnostic test b. A patient who prefers a bath in the evening when his wife visits and can help him c. A patient who is experiencing frequent incontinent diarrheal stools and urine d. A patient who has been awake all night because of pain 8/10

c. A patient who is experiencing frequent incontinent diarrheal stools and urine A patient with urinary and bowel incontinence needs perineal cleaning with each episode of soiling, whereas patients who are normally inactive during the day and have skin that tends to be dry may need to bathe only twice a week.

Which patient is at highest risk for tachycardia? a. A healthy basketball player during warmup exercises b. A patient admitted with hypothermia c. A patient with a fever of 39.4° C (103° F) d. A 90-year-old male taking beta blockers

c. A patient with a fever of 39.4° C (103° F) Fever elevates metabolism by 10%, resulting in an increased heart rate to remove the heat produced. Hypothermia and beta blockers decrease heart rate. Healthy athletes have a lower heart rate as a result of conditioning.

Which of the following patients is most at risk for tachycardia? a. A healthy professional tennis player b. A patient admitted with hypothermia c. A patient with a fever of 39.4°C (103°F) d. A 90-year-old male taking beta blockers

c. A patient with a fever of 39.4°C (103°F) Elevated temperatures increase oxygen demand of the tissues and increase the heart rate to compensate.

Following abdominal surgery, the nurse suspects that the client may be having internal bleeding. Which of the following findings is indicative of this complication? a. Increased blood pressure b. Incisional pain c. Abdominal distention d. Increased urinary output

c. Abdominal distention Signs of internal bleeding following abdominal surgery may include abdominal distention; swelling or bruising around the incision; increased pain; a drop in blood pressure; elevated heart and respiratory rates; thready pulse; cool, clammy, pale skin; and restlessness. The client who is hemorrhaging will have a decreased blood pressure. Incisional pain may occur as a result of surgery. A continuous increase in pain in conjunction with other symptoms of bleeding may indicate internal hemorrhaging. A client who is bleeding will have a decreased urinary output.

While administering medications, a nurse realizes that a prescribed dose of a medication was not given. The nurse acts by completing an incident report and notifying the patient's health care provider. The nurse is exercising: a. Authority. b. Responsibility. c. Accountability. d. Decision making.

c. Accountability. Accountability is nurses being answerable for their actions. It means that nurses accept the commitment to provide excellent patient care and the responsibility for the outcomes of the actions in providing it. Following institutional policy for reporting medication errors demonstrates the nurse's commitment to safe patient care.

The standing orders for a patient include acetaminophen (Tylenol) 650 mg every 4 hours prn for headache. After assessing the patient, identifying the need for headache relief, and determining that the patient has not had Tylenol in the past 4 hours, the nurse a. Notifies the health care provider to obtain a verbal order. b. Directs the nursing assistant to give the Tylenol. c. Administers the Tylenol. d. Performs a pain assessment only after administering the Tylenol.

c. Administers the Tylenol.

What is a critical step when inserting an indwelling catheter into a male patient? a. Slowly inflate the catheter balloon with sterile saline. b. Secure the catheter drainage tubing to the bed sheets c. Advance the catheter to the bifurcation of the drainage and balloon ports. d. Advance the catheter until urine flows, then insert ¼ inch more.

c. Advance the catheter to the bifurcation of the drainage and balloon ports. Advancing the catheter to the bifurcation avoids inflating the catheter balloon in the prostatic urethra causing trauma and pain. Catheter balloons are never inflated with saline. Securing the catheter drainage tubing to the bed sheets increases the risk for accidental pulling or tension on the catheter. The advancement of the catheter until flows and then inserting ¼ inch more is not unique to the male patient.

Which example demonstrates a nurse performing the skill of evaluation? a. The nurse explains the side effects of the new blood pressure medication ordered for the patient. b. The nurse asks a patient to rate pain on a scale of 0 to 10 before administering the pain medication. c. After completing the teaching, the nurse observes a patient draw up and administer an insulin injection. d. The nurse changes a patient's leg ulcer dressing using aseptic technique.

c. After completing the teaching, the nurse observes a patient draw up and administer an insulin injection. Evaluation is one of the most important aspects of clinical care coordination, involving the determination of patient outcomes. Observing a patient do a return demonstration of teaching is evaluation to ensure that patient has understood teaching. Asking a patient to rate their pain on a scale is not evaluation but rather an assessment of the patient's pain because it occurs before administering a pain medication. The other options are interventions.

Which person is the best referral for a patient who speaks a foreign language? a. A family member b. A speech therapist c. An interpreter d. A mental health nurse specialist

c. An interpreter

The family of a patient who is confused and ambulatory insists that all four side rails be up when the patient is alone. What is the best action to take in this situation? a. Contact the nursing supervisor. b. Restrict the family's visiting privileges. c. Ask the family to stay with the patient if possible. d. Inform the family of the risks associated with side-rail use. e. Thank the family for being conscientious and put the four rails up. f. Discuss alternatives that are appropriate for this patient with the family.

c. Ask the family to stay with the patient if possible. d. Inform the family of the risks associated with side-rail use. f. Discuss alternatives that are appropriate for this patient with the family. The family is concerned about ensuring a safe environment for their loved one. The nurse should discuss their concerns, the risk of using restraints related to using four side rails, and safer alternatives such as the presence of a family member. If the family still insists on use of four side rails, you could contact the nursing supervisor to further discuss the situation with them. This is not a reason to restrict visitation; but, although you should appreciate their concern, the use of four side rails should be avoided.

After reviewing the database, the nurse discovers that the patient's vital signs have not been recorded by the student nurse. With this in mind, what clinical decision should the nurse make? a. Administer scheduled medications assuming she would have been informed if the vital signs were abnormal. b. Have the patient transported to the radiology department for a scheduled x-ray, and review vital signs upon return. c. Ask the student nurse to record the patient's vital signs before administering medications. d. Omit the vital signs because the patient is presently in no distress.

c. Ask the student nurse to record the patient's vital signs before administering medications.

A patient presents in the clinic with dizziness and fatigue. The nursing assistant reports a slow but regular radial pulse of 44. What is your priority intervention? a. Request that the nursing assistant repeat the pulse check b. Call for a stat electrocardiogram (ECG) c. Assess the patient's apical pulse and evidence of a pulse deficit d. Prepare to administer cardiac-stimulating medications

c. Assess the patient's apical pulse and evidence of a pulse deficit A radial pulse of 44 is a critical value and requires additional assessment by the nurse. Decreased peripheral pulse can indicate cardiac or vascular abnormality, which can be determined by apical pulse and pulse deficit assessment.

A patient presents in the clinic with dizziness and fatigue. The nursing assistant reports a very slow radial pulse of 44. What is your priority intervention? a. Request that the nursing assistant repeat the pulse check b. Call for a stat electrocardiogram (ECG) c. Assess the patient's apical pulse and evidence of a pulse deficit d. Prepare to administer cardiac-stimulating medications

c. Assess the patient's apical pulse and evidence of a pulse deficit Assessment should always come before interventions. The nurse should be sure she has enough information to act on an assessment before preceding. The nurse should check for a pulse deficit and assess the patients pulse herself and notify the doctor before initiating therapy.

A patient has been transferred to your unit from the respiratory intensive care unit, where he has been for the past 2 weeks recovering from pneumonia. He is receiving oxygen via 4 L nasal cannula. His respiratory rate is 26 breaths/min, and his oxygen saturation is 92%. In planning his care, which information is most helpful in determining your priority nursing interventions? a. Physicians orders b. Medication list c. Baseline assessment d. Patient preferences

c. Baseline assessment Baseline assessment is vitally important for the nurse to understand what is going on with the patient and should be preformed first before any interventions. Correct Answer is C.

Serous drainage from a wound is defined as: a. Fresh bleeding b. Thick and yellow drainage c. Clear, watery plasma d. Beige to brown and foul-smelling drainage

c. Clear, watery plasma Serous drainage is clear, watery plasma. Bleeding is not serous. A thick, yellow drainage or beige to brown drainage is indicative of an infection.

The nurse is caring for a patient with a nasogastric feeding tube who is receiving a continuous tube feeding at a rate of 45 mL per hour. The nurse enters the patient assessment data and information that the head of the patient's bed is elevated to 20 degrees. An alert appears on the computer screen warning that this patient is at a high risk for aspiration because the head of the bed is not elevated enough. This warning is known as which type of system? a. Electronic health record b. Clinical documentation c. Clinical decision support system d. Computerized physician order entry

c. Clinical decision support system A clinical decision support system is based on rules that are triggered by data entry. When certain rules are not met, alerts, warnings, or other information may be provided to the user.

The nurse observes a patient walking down the hall with a shuffling gait. When the patient returns to bed, the nurse checks the strength in both of the patient's legs. The nurse applies the information gained to suspect that the patient has a mobility problem. This conclusion is an example of: a. Cue. b. Reflection. c. Clinical inference. d. Probing.

c. Clinical inference. An inference is your judgment or interpretation of cues such as the shuffling gait and reduced leg strength. Any information gathered through your senses is a cue. Probing is a technique used in interviewing. Reflection is an internal process of thinking back about a situation.

When planning for instruction on cardiac diets to a patient with heart failure, which of the following instructional methods would be the most appropriate for someone identified as a visual/spatial learner? a. Printed pamphlets on cardiovascular disease and dietary recommendations from the American Heart Association b. A role-play activity requiring the patient to select proper foods from a wide selection c. Colored visual diagrams that categorize foods according to fat and sodium content d. A lecture-style discussion on heart healthy diet options

c. Colored visual diagrams that categorize foods according to fat and sodium content Patients who are visual-spatial learners enjoy learning through pictures, visual charts, or any exercise that allows them to visualize concepts.

A nurse changed a patient's surgical wound dressing the day before and now prepares for another dressing change. The nurse had difficulty removing the gauze from the wound bed yesterday, causing the patient discomfort. Today he gives the patient an analgesic 30 minutes before the dressing change. Then he adds some sterile saline to loosen the gauze for a few minutes before removing it. The patient reports that the procedure was much more comfortable. Which of the following describes the nurse's approach to the dressing change? a. Clinical inference b. Basic critical thinking c. Complex critical thinking d. Experience e. Reflection

c. Complex critical thinking The nurse relies on experience and the ability to adapt a procedure such as a dressing change (complex critical thinking) to make it successful. d. Experience The nurse relies on experience and the ability to adapt a procedure such as a dressing change (complex critical thinking) to make it successful.

Postoperatively a client with a closed abdominal wound reports a sudden "pop" after coughing. When the nurse examines the surgical wound site, the nurse sees that the sutures are open and that pieces of small bowel are visible at the bottom of the now opened wound. The correct intervention would be to: a. Allow the area to be exposed to air until all drainage has stopped. b. Place several cold packs over the area, with care taken to protect the skin around the wound. c. Cover the area with sterile saline-soaked towels and immediately notify the surgical team; this is likely to indicate a wound evisceration. d. Cover the area with sterile gauze; place a tight binder over the areas; ask the client to remain in bed for 30 minutes because this is a minor opening in the surgical wound and should reseal quickly.

c. Cover the area with sterile saline-soaked towels and immediately notify the surgical team; this is likely to indicate a wound evisceration. In wound evisceration, the bowel extrudes from the body. The nurse should cover the visible bowel with sterile saline-soaked towels and notify the surgical team. The area should not be allowed to be exposed or to dry out. Cold packs and binders are not acceptable options.

Which of the following actions by the nurse would indicate the need for remedial education in the removal of an indwelling catheter? a. Draping the female client between the thighs b. Obtaining a specimen before removal c. Cutting the catheter to deflate the balloon d. Checking the client's output for 24 hours after removal

c. Cutting the catheter to deflate the balloon The nurse should not cut the catheter to deflate the balloon. The nurse inserts an empty, sterile syringe into the injection port. The nurse slowly withdraws all of the solution to deflate the balloon totally. The nurse then pulls the catheter out smoothly and slowly. The nurse positions the client in the same position as during catheterization. The nurse places a towel between a female client's thighs or over a male client's thighs. Some institutions recommend collecting a sterile urine specimen before removal of the catheter or sending the catheter tip for culture and sensitivity tests. The nurse should assess the client's urinary function by noting the first voiding after catheter removal and documenting the time.

Which of the following are physiological outcomes of immobility? a. Increased metabolism b. Reduced cardiac workload c. Decreased lung expansion d. Decreased oxygen demand

c. Decreased lung expansion Physiologic outcomes of immobility include decreased metabolism, increased cardiac workload, decreased lung expansion, and increased oxygen demand.

A patient exhibits the following symptoms: tachycardia, increased thirst, headache, decreased urine output, and increased body temperature. After analyzing these data, the nurse assigns which of the following nursing diagnoses? a. Adult failure to thrive b. Hypothermia c. Deficient fluid volume d. Nausea

c. Deficient fluid volume

A nurse has taught the patient how to use crutches. The patient went up and down the stairs using crutches with no difficulties. Which information will the nurse use for the "I" in PIE charting? a. Patient went up and down stairs b. Deficient knowledge regarding crutches c. Demonstrated use of crutches d. Used crutches with no difficulties

c. Demonstrated use of crutches

By using known criteria in conducting an assessment such as reviewing with a patient the typical characteristics of pain, a nurse is demonstrating which critical thinking attitude? a. Curiosity b. Adequacy c. Discipline d. Thinking independently

c. Discipline Discipline is being thorough in whatever you do. Using known criteria for assessment and evaluation, as in the case of pain, is an example of discipline.

The perioperative nurse realizes that the most effective means of evaluating the client's understanding of previous teaching is to: a. Provide written material on the subject to be reviewed after discharge b. Reinforce the material with family as the procedure is being performed c. Discuss it with the client and family in the immediate preoperative period d. Offer to answer any questions that the client or family have just before discharge

c. Discuss it with the client and family in the immediate preoperative period In the immediate preoperative period, assess the client's understanding of previous teaching. The other options are not truly evaluations of the client's knowledge.

Which nursing assessment question would best indicate that an incontinent man with a history of prostate enlargement might not be emptying his bladder adequately? a. Do you leak urine when you cough or sneeze? b. Do you need help getting to the toilet? c. Do you dribble urine constantly? d. Does it burn when you pass your urine?

c. Do you dribble urine constantly? Incontinence characterized by constant dribbling of urine is associated with incontinence associated with urinary retention. . The other options point to stress incontinence, functional incontinence or a UTI.

The physician order reads "Lopressor (metoprolol) 50 mg PO daily. Do not give if blood pressure is less than 100 mm Hg systolic." The patient's blood pressure is 92/66. The nurse does not give the medication and a. Does not tell the patient what the blood pressure is. b. Documents only what the blood pressure was. c. Documents that the medication was not given owing to low blood pressure. d. Does not need to inform the health care provider that the medication was held.

c. Documents that the medication was not given owing to low blood pressure.

A patient is isolated for pulmonary tuberculosis. The nurse notes that the patient seems to be angry, but he knows that this is a normal response to isolation. Which is the best intervention? a. Provide a dark, quiet room to calm the patient. b. Reduce the level of precautions to keep the patient from becoming angry. c. Explain the reasons for isolation procedures and provide meaningful stimulation. d. Limit family and other caregiver visits to reduce the risk of spreading the infection.

c. Explain the reasons for isolation procedures and provide meaningful stimulation. Patients on isolation precautions may interpret the needed restrictions as a sign of rejection by the health care worker.

A nurse using the problem-oriented approach to data collection will first a. Complete an observational overview. b. Disregard cues and complete the database questions in chronological order. c. Focus on the patient's presenting situation. d. Make accurate interpretations of the data.

c. Focus on the patient's presenting situation.

During bathing your patient experiences shortness of breath and labored breathing with a respiratory rate of 30. The bed is in a flat position. You change the bed position to: a. Trendelenburg's. b. Reverse Trendelenburg's. c. Fowler's. d. Semi-Fowler's.

c. Fowler's. sitting up position can help increase a patients ability to facilitate lung expansion and improve oxygenation.

A preceptor observes a new graduate nurse discussing changes in a patient's condition with a physician over the phone. The new graduate nurse accepts telephone orders for a new medication and for some laboratory tests from the physician at the end of the conversation. During the conversation the new graduate writes the orders down on a piece of paper to enter them into the electronic medical record when a computer terminal is available. At this hospital new medication orders entered into the electronic medical record can be viewed immediately by hospital pharmacists, and hospital policy states that all new medications must be reviewed by a pharmacist before being administered to patients. Which of the following actions requires the preceptor to intervene? The new nurse: a. Reads the orders back to the health care provider to verify accuracy of transcribing the orders after receiving them over the phone. b. Documents the date and time of the phone conversation, the name of the physician, and the topics discussed in the electronic record. c. Gives a newly ordered medication before entering the order in the patient's medical record. d. Asks the preceptor to listen in on the phone conversation.

c. Gives a newly ordered medication before entering the order in the patient's medical record. When provider orders for new medication(s) are entered into an electronic medical record, the new orders are available to pharmacists using the same electronic system within the hospital. To improve patient safety, many hospitals have a policy that new medications are not to be administered (unless in an emergency) until a pharmacist reviews the new order(s) and verifies that there are no documented allergies to the medications, the ordered dose(s) are appropriate, and there are no potential medication interactions with medications already ordered for a patient. Nurses enter orders into the computer or write them on the order sheet as they are being given to allow the read-back process to occur.

The nurse is assessing the pressure ulcer of a 68-year-old female client. Which of the following would indicate to the nurse that healing is taking place? a. Eschar b. Slough c. Granulation tissue d. Exudate

c. Granulation tissue Granulation tissue is red moist tissue composed of new blood vessels, the presence of which indicates progression toward healing. Black or brown necrotic tissue is eschar which you will need to remove before healing can proceed. Soft, yellow or white tissue is characteristic of slough (stringy substance attached to wound bed), and you will need to remove this before the wound is able to heal. Wound exudate describes the amount, color, consistency, and odor of wound drainage and is part of the wound assessment. Excessive exudate indicates the presence of infection. The presence of exudate on the skin surrounding the wound is indicative of wound deterioration.

The nurse is reviewing the Health Insurance Portability and Accountability Act (HIPAA) regulations with the patient during the admission process. The patient states, "I'm not familiar with these HIPAA regulations. How will they affect my care?" Which of the following is the best response? a. HIPAA allows all hospital staff access to your medical record. b. HIPAA limits the information that is documented in your medical record. c. HIPAA provides you with greater protection of your personal health information. d. HIPAA enables health care institutions to release all of your personal information to improve continuity of care.

c. HIPAA provides you with greater protection of your personal health information. HIPAA provides patients with control over who receives and accesses their medical records. It does not allow uncontrolled access to the medical records. HIPAA also does not dictate what must be documented in the patient's medical record.

What is the most effective way to control transmission of infection? a. Isolation precautions b. Identifying the infectious agent c. Hand hygiene practices d. Vaccinations

c. Hand hygiene practices Hands contaminated with transient bacteria are a primary source for transmission of infection.

The nurse is taking a health history of a newly admitted patient with a diagnosis of possible fecal impaction. Which of the following is the priority question to ask the patient or caregiver? a. Have you eaten more high-fiber foods lately? b. Are your bowel movements soft and formed? c. Have you experienced frequent, small liquid stools recently? d. Have you taken antibiotics recently?

c. Have you experienced frequent, small liquid stools recently? Frequent or continuous oozing of liquid stools occurs when liquid fecal matter above the impacted stool seeps around the fecal impaction.

A patient was admitted 2 days ago with pneumonia and a history of angina. The patient is now having chest pain with a pulse rate of 108. Using the SBAR, which piece of data will the nurse use for B? a. Having chest pain b. Pulse rate of 108 c. History of angina d. Oxygen is needed.

c. History of angina

For a client who has a muscle sprain, localized hemorrhage, or hematoma, application of which of the following helps to prevent edema formation, control bleeding, and anesthetize the body part? a. Absorptive diaper b. Elastic bandage c. Ice bag d. Binder

c. Ice bag The application of cold will help constrict blood vessels, which will reduce swelling that occurs with bleeding and edema formation in a muscle sprain. It also provides a numbing effect. Binders and elastic bandages are not initial treatments for a sprain. A diaper would not be used for a muscle sprain.

Immobility is a major risk factor for pressure ulcers. In caring for the patient who is immobilized, the nurse needs to be aware that a. Breaks in skin integrity are easy to heal. b. Preventing a pressure ulcer is more expensive than treating one. c. Immobilized patients can develop skin breakdown within 3 hours. d. Pressure ulcers are caused by a sudden influx of oxygen to the tissue.

c. Immobilized patients can develop skin breakdown within 3 hours.

Which nursing diagnostic statement is accurately written for a patient with a medical diagnosis of pneumonia? a. Risk for infection related to lower lobe infiltrate b. Risk for deficient fluid volume related to dehydration c. Impaired gas exchange related to alveolar-capillary membrane changes d. Ineffective breathing pattern related to pneumonia

c. Impaired gas exchange related to alveolar-capillary membrane changes

A client who has type 2 diabetes is scheduled for the removal of a skin lesion on his right shoulder at an ambulatory surgery unit. The nursing diagnosis the client is at greatest risk for postoperatively is: a. Risk for injury b. Risk for infection c. Impaired wound healing d. Imbalanced nutrition: less than body requirements

c. Impaired wound healing Diabetes increases susceptibility to infection and impairs wound healing from altered glucose metabolism and associated circulatory impairment. The stress of surgery often causes increases in blood glucose levels. Although all the options present with possible nursing diagnoses, the remaining options are not of primary concern because steps can be taken (e.g., antibiotic, intravenous fluids) to minimize the risk. Impaired wound healing is not as easily managed.

In which step of the nursing process does the nurse provide nursing care interventions to patients? a. assessment b. Planning c. Implementation d. Evaluation

c. Implementation

Which principle is most important for a nurse to follow when using a clinical practice guideline for an assigned patient? a. Knowing the source of the guideline b. Reviewing the evidence used to develop the guideline c. Individualizing how to apply the clinical guideline for a patient d. Explaining to a patient the purpose of the guideline

c. Individualizing how to apply the clinical guideline for a patient Individualizing patient care is still the important principle for implementing care, even when a clinical guideline is used. Explaining any interventions in a guideline to the patient is important but not the most critical factor in implementing care. Reviewing the source of the guideline and applicable evidence do not directly benefit a patient.

When a patient has fecal incontinence as a result of cognitive impairment, it may be helpful to teach caregivers to do which of the following interventions? a. Cleanse the skin with antibacterial soap and apply talcum powder to the buttocks b. Use diapers and heavy padding on the bed c. Initiate bowel or habit training program to promote continence d. Help the patient to toilet once every hour

c. Initiate bowel or habit training program to promote continence A cognitively impaired patient may have forgotten how to respond to the urge to defecate and benefit from a structured program of bowel retraining.

A nursing student reports to a lead charge nurse that his assigned patient seems to be less alert and his blood pressure is lower, dropping from 140/80 to 110/60. The nursing student states, "I believe this is a nursing diagnosis of Deficient Fluid Volume." The lead charge nurse immediately goes to the patient's room with the student to assess the patient's orientation, heart rate, skin turgor, and urine output for last 8 hours. The lead charge nurse suspects that the student has made which type of diagnostic error? a. Insufficient cluster of cues b. Disorganization c. Insufficient number of cues d. Evidence that another diagnosis is more likely

c. Insufficient number of cues It is likely the charge nurse suspects that the student has not collected enough cues to support the diagnosis. A change in blood pressure and mental status changes are significant findings that can be attributed to fluid volume excess and other diagnoses. The recommendation of the symptom cluster by the registered nurse would allow the student to have sufficient data to confirm a deficient fluid volume.

A nurse is teaching a patient about wound care that will need to be done daily at home after the patient is discharged. This is which priority nursing need for this patient? a. Low priority b. High priority c. Intermediate priority d. Nonemergency priority

c. Intermediate priority Teaching patients wound care for discharge is an intermediate priority. Intermediate priorities are nonemergency, nonlife-threatening, actual or potential needs that the patient and family members are experiencing.

A patient has been hospitalized for the past 48 hours with a fever of unknown origin. His medical record indicates tympanic temperatures of 38.7° C (101.6° F) (0400), 36.6° C (97.9° F) (0800), 36.9° C (98.4° F) (1200), 37.6° C (99.6° F) (1600), and 38.3° C (100.9° F) (2000). How would you describe this pattern of temperature measurements? a. Usual range of circadian rhythm measurements b. Sustained fever pattern c. Intermittent fever pattern d. Resolving fever pattern

c. Intermittent fever pattern Temperature was elevated above acceptable range, returned to normal, and then elevated.

When assessing a patient's first voided urine of the day, which finding should be reported to the health care provider? a. Pale yellow urine b. Slightly cloudy urine c. Light pink urine d. Dark amber urine

c. Light pink urine Light pink urine indicates the presence of blood in the urine, which is never a normal finding. First voided urine can normally be slightly cloudy and darker in color. Pale yellow urine indicates normal finding.

What is the importance of the Hospital Consumer Assessment of Healthcare Providers and Systems (HCAHPS) survey? a. Measures a nurse's competency in interdisciplinary care b. Measures the number of adverse events in a hospital c. Measures quality of care within hospitals d. Measures referrals to a health care agency

c. Measures quality of care within hospitals HCAHPS is a survey that has become a standard for measuring and comparing quality of hospitals. It is a survey of patient perceptions.

A nurse in a mother-baby clinic learns that a 16-year-old has given birth to her first child and has not been to a well-baby class yet. The nurse's assessment reveals that the infant cries when breastfeeding and has difficulty latching on to the nipple. The infant has not gained weight over the last 2 weeks. The nurse identifies the patient's nursing diagnosis as Ineffective Breastfeeding. Which of the following is the best "related to" factor? a. Infant crying at breast b. Infant unable to latch on to breast correctly c. Mother's deficient knowledge d. Lack of infant weight gain

c. Mother's deficient knowledge In this scenario the related factor is the mother's deficient knowledge. A related factor is a condition, historical factor, or etiology that gives a context for the defining characteristics, in this case the infant crying, inability to latch on to breast, and absent weight gain.

As you are obtaining the oxygen saturation on a 19-year-old college student with severe asthma, you note that she has black nail polish on her nails. You remove the polish from one nail, and she asks you why her nail polish had to be removed. What is the best response? a. Nail polish attracts microorganisms and contaminates the finger sensor. b. Nail polish increases oxygen saturation. c. Nail polish interferes with sensor function. d. Nail polish creates excessive heat in sensor probe.

c. Nail polish interferes with sensor function. The pigment in black nail polish affects light absorption and reflection.

During voluntary movement, impulses descend from the motor strip to the spinal cord. Impulses stimulate muscles by way of a. Ligaments b. Tendons c. Neurotransmitters d. Cartilage

c. Neurotransmitters

A patient receiving chemotherapy experiences stomatitis. The nurse advises the patient to use: a. Community mouthwash. b. Alcohol-based mouth rinse. c. Normal saline rinses. d. Firm toothbrush.

c. Normal saline rinses. Normal saline is the safest solution to use in caring for a patient with stomatitis. Alcohol and community mouthwashes can be irritating and burning. A soft toothbrush should be used.

Which of these interventions would take priority and should be included in a plan of care for a patient who presents with pneumonia? a. Maintain the temperature at 65° F. b. Assume that the patient is in pain and treat accordingly. c. Observe the patient for decreased activity tolerance. d. Provide the patient ice chips as requested.

c. Observe the patient for decreased activity tolerance. Patients with pneumonia have impaired oxygenation which may limit their activity tolerance.

The nurse is teaching the patient to obtain a specimen for fecal occult blood testing using fecal immunochemical (FIT) testing at home. How does the nurse instruct the patient to collect the specimen? a. Three fecal smears from one bowel movement b. One fecal smear from an early-morning bowel movement c. One fecal smear from three separate bowel movements d. Three fecal smears when blood can be seen in the bowel movement

c. One fecal smear from three separate bowel movements Samples from three separate bowel movements decrease the risk of a false-negative or a false-positive result.

After administration of oxygen, which of the following signs indicates that the oxygen therapy has been successful? a. BP 130/82 mmHg. b. Temp 99.8 F. c. Oxygen saturation 96%. d. Respiration rate 24 bpm.

c. Oxygen saturation 96%

For the nursing diagnosis of Deficient Knowledge a nurse selects an outcome from the Nursing Outcome Classification (NOC) of patient knowledge of arthritis treatment. Which of the following are examples of an outcome indicator for this outcome? (Select all that apply) a. Nurse provides four teaching sessions before discharge. b. Patient denies joint pain following heat application. c. Patient describes correct schedule for taking antiarthritic medications. d. Patient explains situations for using heat application on inflamed joints. e. Patient explains role family caregiver plays in applying heat to inflamed joint.

c. Patient describes correct schedule for taking antiarthritic medications. d. Patient explains situations for using heat application on inflamed joints. The patient must exhibit behaviors that measure knowledge of arthritis treatment. This would include describing his medication schedule and explaining when to apply heat to inflamed joints. The nurse providing teaching sessions is not a patient outcome. The patient denying joint pain is not an evaluative indicator of knowledge. Explanation of the family caregiver's role is not a measure of the patient's knowledge of treatment.

Which of the following methods of data collection is utilized to establish a nursing assessment? a. Reviewing the current literature to determine evidence-based nursing actions. b. Orders for diagnostic and laboratory tests. c. Physical examination. d. Anticipated medications to be ordered.

c. Physical examination.

What is the appropriate way for a nurse to dispose of information printed out from a patient's electronic health record? a. Rip the papers up into small pieces and place the pieces into a standard trash can b. Place all papers in the flip-top binder designated for that patient that is located in the nurse's station on the patient care unit c. Place papers with patient information in a secure canister marked for shredding d. Burn documents with patient information in the steel sink located within the dirty supply room on the patient care unit

c. Place papers with patient information in a secure canister marked for shredding Confidential patient information should be shredded. It is generally collected in large secure containers and shredded at scheduled times.

A patient on prolonged bed rest is at an increased risk to develop this common complication of immobility if preventive measures are not taken: a. Myoclonus b. Pathological fractures c. Pressure ulcers d. Pruritus

c. Pressure ulcers Immobility is a major risk factor for pressure ulcers. Any break in the integrity of the skin is difficult to heal. Preventing a pressure ulcer is much less expensive than treating one; therefore preventive nursing interventions are imperative.

A client has a healing abdominal wound. The wound has minimal exudate and collagen formation. The wound is identified by the nurse as being in which phase of healing? a. Primary intention b. Inflammatory phase c. Proliferative phase d. Secondary intention

c. Proliferative phase During the proliferative phase, the wound fills with granulation tissue (including collagen formation), the wound contracts, and the wound is resurfaced by epithelialization. Primary intention is not a phase of wound healing. Wounds that heal by primary intention have minimal tissue loss, such as a surgical wound. The edges are approximated and the risk for infection is low. During the inflammatory phase, platelets gather to stop bleeding, a fibrin matrix forms, and white blood cells reach the wound, clearing it of debris. Secondary intention is not a phase of wound healing. Wounds that heal by secondary intention have loss of tissue, such as a pressure ulcer. The wound is left open until it becomes filled by scar tissue.

There are three phases of wound healing. The nurse observes granulation tissue in a client's pressure ulcer. What phase of wound healing is represented by granulation tissue? a. Maturation phase b. Hemostasis phase c. Proliferative phase d. Inflammatory phase

c. Proliferative phase Tissue granulation occurs in the proliferative phase. Maturation is the final stage of wound healing. Hemostasis occurs during the inflammatory phase.

The client tells the nurse that "blowing into this tube thing (incentive spirometer) is a ridiculous waste of time." The nurse explains that the specific purpose of the therapy is to: a. Directly remove excess secretions from the lungs b. Increase pulmonary circulation c. Promote lung expansion d. Stimulate the cough reflex

c. Promote lung expansion The primary purpose of using an incentive spirometer is to promote lung expansion. Coughing exercises are used to remove excess secretions from the lungs. Ambulation helps increase pulmonary circulation as the respiratory rate increases. The primary purpose of incentive spirometry is not to stimulate the cough reflex, but to promote lung expansion.

Which of the following is a nursing intervention? a. The patient will ambulate in the hallway twice this shift using crutches. b. Impaired physical mobility related to inability to bear weight on right leg c. Provide assistance while the patient walks in the hallway twice this shift with crutches. d. The patient is unable to bear weight on right lower extremity.

c. Provide assistance while the patient walks in the hallway twice this shift with crutches.

A patient newly diagnosed with cervical cancer is going home. The patient is avoiding discussion of her illness and postoperative orders. What is the nurse's best plan in teaching this patient? a. Teach the patient's spouse b. Focus on knowledge the patient will need in a few weeks c. Provide only the information that the patient needs to go home d. Convince the patient that learning about her health is necessary

c. Provide only the information that the patient needs to go home This patient is in denial; thus it is appropriate to give her only information that is needed immediately.

A 48-year-old male client with a history of chronic obstructive pulmonary disease (COPD) is scheduled for an inguinal hernia repair. The nurse instructs that client that he can expect the health care provider to order which of the following tests before surgery? a. Human immunodeficiency virus (HIV) antibody b. Prolactin level c. Pulmonary function test d. Glucose tolerance test

c. Pulmonary function test Pulmonary function testing and occasionally arterial blood gas analysis are often performed before surgery on clients with preexisting lung disease. An HIV-antibody test diagnoses HIV. It is not a test that is normally ordered before surgery. Prolactin levels are used to diagnose and monitor prolactin-secreting pituitary adenomas. A glucose tolerance test is used to assist in the diagnosis of diabetes mellitus and is also used in the evaluation of hypoglycemia. It is not a test that is normally ordered before surgery.

Integrity of the oral mucosa depends on salivary secretion. Which of the following factors impairs salivary secretion? a. Use of cough drops b. Immunosuppression c. Radiation therapy d. Dehydration e. Presence of oral airway

c. Radiation therapy Radiation therapy reduces salivary flow. Dehydration impairs salivary secretion in the mouth. Cough drops increase sugar or acid content in the mouth, causing caries. Immunosuppression causes inflammation and bleeding of the gums. An oral airway irritates oral mucosa. d. Dehydration Radiation therapy reduces salivary flow. Dehydration impairs salivary secretion in the mouth. Cough drops increase sugar or acid content in the mouth, causing caries. Immunosuppression causes inflammation and bleeding of the gums. An oral airway irritates oral mucosa.

After assessing the patient and identifying the need for headache relief, the nurse administers acetaminophen (Tylenol) for the patient's headache. What is the nurse's next priority action for this patient? a. Eliminate Acute pain from the nursing care plan. b. Direct the nursing assistant to ask if the patient's headache is relieved. c. Reassess the patient's pain level in 30 minutes. d. Revise the plan of care.

c. Reassess the patient's pain level in 30 minutes.

What is the first intervention included on any patient's plan of care? a. Determine patient outcomes and goals. b. Prioritize the patient's nursing diagnoses. c. Reassess the patient. d. Assess for a patent airway.

c. Reassess the patient.

Granulated tissue is best described as a. Soft, yellow, and stringy b. Black, hard, and necrotic c. Red, moist, and vascular-rich d. Yellow, spongy, and sinewy

c. Red, moist, and vascular-rich Granulation tissue is red, moist tissue composed of new blood vessels, the presence of which indicates progression toward healing. Soft, yellow or white tissue is characteristic of slough (stringy substance attached to wound bed), and you will need to remove this before the wound is able to heal. Black or brown necrotic tissue is eschar, which you will also need to remove before healing can proceed.

The female client on the surgical unit is being prepared for abdominal surgery with general anesthesia. In preparing this client for surgery, the nurse should: a. Leave all of her jewelry intact b. Provide her with sips of water for a dry mouth c. Remove her makeup and nail polish d. Remove her hearing aid before transport to the operating room

c. Remove her makeup and nail polish All makeup, including nail polish, should be removed to expose normal skin and nail color to determine the client's level of oxygenation and circulation during and after surgery. Jewelry and other valuables should be given to family members or secured for safekeeping. A wedding band can be taped in place unless there is a risk that the client will experience swelling of the hand or fingers. For safety, metal items, such as for pierced areas, should be removed. The client should be allowed nothing by mouth (NPO) before surgery to prevent vomiting and aspiration with general anesthesia. Clients may be allowed to keep personal items such as a hearing aid until they reach the preoperative area.

What instructions should the nurse give the NAP concerning a patient who has had an indwelling urinary catheter removed that day? a. Limit oral fluid intake to avoid possible urinary incontinence. b. Expect patient complaints of suprapubic fullness and discomfort. c. Report the time and amount of first voiding. d. Instruct patient to stay in bed and use a urinal or bedpan.

c. Report the time and amount of first voiding. In order to adequately assess bladder function after a catheter is removed; voiding frequency and amount should be monitored. Unless contraindicated, fluids should be encouraged. To promote normal micturition, patients should be placed in as normal a posture for voiding as possible. Suprapubic tenderness and pain are possible indicators of urinary retention and/or a UTI.

Patients with diabetes mellitus need special foot care to prevent the development of ulcers. Knowing this, the nurse a. Trims the patient's toenails daily. b. Has the patient soak his or her feet twice a day. c. Requests a consult with a nail care specialist. d. Assesses the brachial artery.

c. Requests a consult with a nail care specialist. nail care for diabetic patients should be done by a podiatrist to avoid injury or infection to the patients foot, as diabetic patients are at high risk for lower limb amputations related to injury and ulcers on the lower extremities.

When you are assigned to a patient who has a reduced level of consciousness and requires mouth care, which physical assessment techniques should you perform before the procedure? a. Oxygen saturation b. Heart rate c. Respirations d. Gag reflex e. Response to painful stimulus

c. Respirations Check a patient's respirations and whether there is a gag reflex present to determine risk for aspiration and to establish a baseline for the patient's condition. d. Gag reflex Check a patient's respirations and whether there is a gag reflex present to determine risk for aspiration and to establish a baseline for the patient's condition.

The nurse is caring for a patient with an ileostomy. Which intervention is most important? a. Cleansing the stoma with hot water b. Inserting a deodorant tablet in the stoma bag c. Selecting or cutting a pouch with an appropriate-size stoma opening d. Wearing sterile gloves while caring for the stoma

c. Selecting or cutting a pouch with an appropriate-size stoma opening A properly fitting pouch that does not leave skin exposed prevents peristomal skin breakdown.

Several instruments are available for assessing clients who are at high risk for developing a pressure ulcer. To identify the risk, what areas should you assess? a. Infection, hemorrhage, dehiscence, evisceration, and fistulas b. Physical condition, mental condition, activity, mobility, and incontinence c. Sensory perception, moisture, activity, mobility, nutrition, friction, and shear

c. Sensory perception, moisture, activity, mobility, nutrition, friction, and shear The Braden Scale measures the following risk factors: sensory perception, moisture, activity, mobility, nutrition, friction, and shear. The Norton Scale measures five risk factors: physical condition, mental condition, activity, mobility, and incontinence. Infection, hemorrhage, dehiscence, evisceration, and fistulas are the complications of wound healing. The factors that influence pressure ulcer formation and wound healing are nutrition, tissue perfusion, infection, age, shear force and friction, and moisture.

An aspect of clinical decision making is knowing the patient. Which of the following is the most critical aspect of developing the ability to know the patient? a. Working in multiple health care settings b. Learning good communication skills c. Spending time establishing relationships with patients d. Relying on evidence in practice

c. Spending time establishing relationships with patients Knowing the patient relates to a nurse's experience with caring for patients, time spent in a specific clinical area, and having a sense of closeness with them. However, a critical aspect to knowing the patient and thus being able to make timely and appropriate decisions is spending time establishing relationships with them.

When cleaning a wound, the nurse should: a. Wash over the wound twice and discard that swab b. Move from the outer region of the wound toward the center c. Start at the drainage site and move outward with circular motions d. Use an antiseptic solution followed by a normal saline rinse

c. Start at the drainage site and move outward with circular motions To cleanse the area of an isolated drain site, the nurse cleans around the drain, moving in circular rotations outward from a point closest to the drain. The nurse never uses the same piece of gauze or swab to cleanse across an incision or wound twice. The wound should be cleansed in a direction from the least contaminated area, such as from the wound to the surrounding skin. The wound is cleaned from the center region to the outer region. An antiseptic solution is not used to clean a wound, as it may be cytotoxic.

Which of the following nursing interventions should be implemented to maintain a patent airway in a patient on bed rest? a. Isometric exercises b. Administration of low-dose heparin c. Suctioning every 4 hours d. Use of incentive spirometer every 2 hours while awake

c. Suctioning every 4 hours Incentive spirometry opens the airway, preventing atelectasis.

A nurse preceptor is working with a student nurse. Which behavior by the student nurse will require the nurse preceptor to intervene? a. The student nurse reviews the patient's medical record. b. The student nurse reads the patient's plan of care. c. The student nurse shares patient information with a friend. d. The student nurse documents medication administered to the patient.

c. The student nurse shares patient information with a friend.

You are admitting Mr. Jones, a 64-year-old patient who had a right hemisphere stroke and a recent fall. His wife stated that he has a history of high blood pressure, which is controlled by an antihypertensive and a diuretic. Currently he exhibits left-sided neglect and problems with spatial and perceptual abilities and is impulsive. He has moderate left-sided weakness that requires the assistance of two and the use of a gait belt to transfer to a chair. He currently has an intravenous (IV) line and a urinary catheter in place. Which factors increase his fall risk at this time? a. Smokes a pack a day b. Used a cane to walk at home c. Takes antihypertensive and diuretics d. History of recent fall e. Neglect, spatial and perceptual abilities, impulsive f. Requires assistance with activity, unsteady gait g. IV line, urinary catheter

c. Takes antihypertensive and diuretics d. History of recent fall e. Neglect, spatial and perceptual abilities, impulsive f. Requires assistance with activity, unsteady gait g. IV line, urinary catheter Smoking is not a risk factor for falls. Use of the cane at home is not a current risk factor for falls. Risk is determined by his current status.

A new nurse is experiencing lateral violence at work. Which steps could the nurse take to address this problem? a. Challenge the nurses in a public forum to embarrass them and change their behavior b. Talk with the department secretary and ask if this has been a problem for other nurses c. Talk with the preceptor or manager and ask for assistance in handling this issue d. Say nothing and hope things get better

c. Talk with the preceptor or manager and ask for assistance in handling this issue Talking with a preceptor, manager, or mentor notifies others of the problem, provides support for the nurse, and helps the nurse learn skills in addressing lateral violence.

A nurse enters the room of a 32-year-old patient newly diagnosed with cancer at the beginning of the 0700 evening/night shift. The nurse noted in the patient's nursing history that this is her first hospitalization. She is scheduled for surgery in the morning to remove a tumor and has questions about what to expect after surgery. She is observed talking with her mother and is crying. The patient says, "This is so unfair." An order has been written for an enema to be given this evening in preparation for the surgery. The nurse establishes priorities for which of the following situations first? a. Giving the enema on time b. Talking with the patient about her past experiences with illness c. Talking with the patient about her concerns and acknowledging her sense of unfairness d. Beginning instruction on postoperative procedures

c. Talking with the patient about her concerns and acknowledging her sense of unfairness The patient is obviously emotionally upset. Her concerns, whether they are about surgery or cancer or both, need to be addressed first for her to be able to be instructed and be comfortable for the enema. Talking with the patient about her past experiences may be appropriate in the long term but is less important than the other three priorities.

During a visit to the clinic, a patient tells the nurse that he has been having headaches on and off for a week. The headaches sometimes make him feel nauseated. Which of the following responses by the nurse is an example of probing? a. So you've had headaches periodically in the last week and sometimes they cause you to feel nauseated—correct? b. Have you taken anything for your headaches? c. Tell me what makes your headaches begin. d. Uh huh, tell me more.

c. Tell me what makes your headaches begin. An open-ended question that probes such as "Tell me what makes your headaches begin" encourages a fuller description of a situation. The statement "So you've had headaches periodically in the last week, and sometimes they cause you to feel nauseated—correct?" is a summative statement. Asking whether the patient has taken anything for the headaches is a closed-ended question. Saying "Uh huh, tell me more" is an example of back channeling.

Mr. Johnson is a 63-year old gentleman who is hospitalized for pneumonia. He has history of heart failure and hypertension. He looks dyspneic and is mouth-breathing. To establish a baseline assessment for Mr. Johnson, the nurse asks you to measure his vital signs. Inspect Mr. Johnson in the image below and decide what temperature route should use to obtain his temperature: a. Orally b. Tympanically c. Temporally d. Rectally

c. Temporally Orally: not appropriate since he is a mouth-breathing with shortness of breath.Tympanic: not appropriate since he wearing hearing aids.Rectally: not indicated since a non-invasive route (temporal) is available.

A nurse enters a patient's room and begins a conversation. During this time the nurse evaluates how a patient is tolerating a new diet plan. The nurse decides to also evaluate the patient's expectations of care. Which statement is appropriate for evaluating a patient's expectations of care? a. On a scale of 0 to 10 rate your level of nausea. b. The nurse weighs the patient. c. The nurse asks, "Did you believe that you received the information you needed to follow your diet?" d. The nurse states, "Tell me four different foods included in your diet."

c. The nurse asks, "Did you believe that you received the information you needed to follow your diet?" Evaluating patient expectations of care involves measuring his or her perceptions of care (e.g., if this particular patient thinks that he or she had received sufficient information).

The primary reason an older adult client is more likely to develop a pressure ulcer on the elbow as compared to a middle-age adult is: a. A reduced skin elasticity is common in the older adult b. The attachment between the epidermis and dermis is weaker c. The older client has less subcutaneous padding on the elbows d. Older adults have a poor diet that increases risk for pressure ulcers

c. The older client has less subcutaneous padding on the elbows Although all the options are related to causes of skin injury in older adults, the hypodermis decreases in size with age, and so the older client has little subcutaneous padding over bony prominences; thus they are more prone to skin breakdown.

A nurse is giving a hand-off report to the oncoming nurse. Which information is critical for the nurse to report? a. The patient had a good day with no complaints. b. The family is demanding and argumentative. c. The patient has a new pain medication, Lortab. d. The family is poor and had to go on welfare.

c. The patient has a new pain medication, Lortab.

The nurse observes an adult Middle Eastern patient attempting to bathe himself with only his left hand. The nurse recognizes that this behavior likely relates to: a. Obsessive compulsive behavior. b. Personal preferences. c. The patient's cultural norm. d. Controlling behaviors.

c. The patient's cultural norm. Cultural beliefs often influence patients' hygiene practices. Middle Eastern practices encourage one hand to be kept clean at all times.

After a nurse receives a change-of-shift report on his assigned patients, he prioritizes the tasks that need to be completed. This is an example of a nurse displaying which practice? a. Organizational skills b. Use of resources c. Time management d. Evaluation

c. Time management Completing a priority to-do list is a useful time-management skill. Change-of-shift report can help you sequence activities on the basis of what you learn about the patients' conditions and the care the patient has received.

Pressure ulcers form primarily as a result of: a. Nitrogen buildup in the underlying tissues b. Prolonged illness or disease c. Tissue ischemia d. Poor nutrition

c. Tissue ischemia Pressure is the major cause of pressure ulcer formation. Prolonged, intense pressure affects cellular metabolism by decreasing or obliterating blood flow, resulting in tissue ischemia and ultimately tissue death. Prolonged illness or disease and poor nutrition may place a client at risk for pressure ulcer development.

The nurse is very busy and needs to delegate some tasks to the nursing assistive personnel (NAP). Which of the following would be the most appropriate task to delegate? a. Postoperative client teaching b. Demonstrating postoperative exercises c. Transporting the preoperative client from the unit to the holding area d. Reviewing the preoperative assessment to make sure that the client's vital signs have been documented

c. Transporting the preoperative client from the unit to the holding area In many hospitals a nursing orderly or transporter brings a stretcher for transporting the client. The transporter checks the client's identification bracelet for two identifiers against the client's chart to be sure that the right person is going to surgery.

On assessing your patient's sacral pressure ulcer, you note that the tissue over the sacrum is dark, hard, and adherent to the wound edge. What is the correct category/stage for this patient's pressure ulcer? a. Category/Stage II b. Category/Stage IV c. Unstageable d. Suspected deep tissue damage

c. Unstageable To determine the category/stage of a pressure ulcer you examine the depth of the tissue involvement. Since the pressure ulcer assessed was covered with necrotic tissue, the depth could not be determined. Thus this pressure ulcer cannot be staged.

A patient who is Spanish-speaking does not appear to understand the nurse's information on wound care. Which action should the nurse take? a. Arrange for a Spanish-speaking social worker to explain the procedure b. Ask a fellow Spanish-speaking patient to help explain the procedure c. Use a professional interpreter to provide wound care education in Spanish d. Ask the patient to write down questions that he or she has for the nurse

c. Use a professional interpreter to provide wound care education in Spanish Professional certified interpreters can help with simple or complex health care communications such as teaching instructions, test results, or education related to surgical consent. Other health care workers who are not certified interpreters cannot be relied on to provide clear and effective communication of health care information or teaching.

A nursing student is reporting during hand-off to the registered nurse (RN) assuming her patient's care. The student states, "Mr. Roarke had a good day, his intravenous (IV) fluid is infusing at 124 mL/hr with D5 ½ NS infusing in right forearm. The IV site is intact, and no complaints of tenderness. I ambulated him twice during the shift; he tolerated well walking to end of hall and back with no shortness of breath. He still uses his cane without difficulty. Mr. Roarke said he slept better last night after I closed his door and gave him a chance to be uninterrupted. If the nurse's goal for Mr. Roarke was to improve activity tolerance, which expected outcomes were shared in the hand-off? a. IV site not tender b. Uses cane to walk c. Walked to end of hall d. No shortness of breath e. Slept better during night

c. Walked to end of hall The goal for improving activity tolerance will require an outcome that is a measure of changes in activity tolerance such as no shortness of breath during exercise or walking a set distance. d. No shortness of breath The goal for improving activity tolerance will require an outcome that is a measure of changes in activity tolerance such as no shortness of breath during exercise or walking a set distance.

Which nursing intervention minimizes the risk for trauma and infection when applying an external/condom catheter? a. Leave a gap of 3-5 inches between the tip of the penis and drainage tube b. Shave the pubic area so that hair does not adhere c. Wash with soap and water prior to applying the condom type catheter. d. Apply tape to the condom sheath to keep it securely in place.

c. Wash with soap and water prior to applying the condom type catheter.

The autolytic, mechanical, chemical, and surgical methods that are often used during wound management are all methods of accomplishing what? a. Wound dressing b. Wound cleansing c. Wound débridement d. Stimulation of growth factors

c. Wound débridement Methods of débridement include mechanical, autolytic, chemical, and surgical methods. All of these methods share the common objective of removing nonviable, necrotic tissue. Dressing, cleansing and stimulation of growth factors are not part of débridement.

Knowing that Mr. Johnson has a history of heart failure, which has led atrial fibrillation dysrhythmia, to accurately measure his pulse, should assess his: a. radial pulse. b. brachial pulse. c. apical pulse. d. Dorsalis pedis pulse.

c. apical pulse

Due to his history of dysrhythmia, you suspect that Mr. Johnson will have pulse deficit. To measure his pulse deficit, you should: a. subtract 60 from his pulse rate. b. subtract his pulse rate from 100. c. assess his apical pulse and radial pulse for the same minute and subtract the difference. d. assess his apical pulse and 30 minutes later assessing his carotid pulse and subtract the difference.

c. assess his apical pulse and radial pulse for the same minute and subtract the difference.

Acute pain related to tissue injury from surgical incision. a. subjective b. objective c. assessment d. plan

c. assessment

Counseling _______ a. assisting patient with oral care b. protecting a violent patient from injury c. discussing a patient's options in choosing palliative care d. using safe patient handling during positioning of a patient

c. discussing a patient's options in choosing palliative care

Category/Stage IV pressure ulcer a. nonblanchable redness of intact skin. Discoloration, warmth, edema, or pain may also be present. b. partial thickness skin loss or intact blister with serosanginous fluid c. full thickness tissue loss, muscle and bone visible. may include undermining d. full thickness skin loss, subcutaneous fat may be visible. may include undermining

c. full thickness tissue loss, muscle and bone visible. may include undermining

An 82-year-old patient who resides in a nursing home has the following three nursing diagnoses: Risk for Fall, Impaired Physical Mobility related to pain, and Imbalanced Nutrition: Less Than Body Requirements related to reduced ability to feed self. The nursing staff identified several goals of care. Match the goals on the left with the appropriate outcome statements on the right. Goals Outcomes (Patient will achieve pain relief by discharge.) a. patient walks 20 feet using a walker in 24 hrs. b. patient identifies barriers to remove in the home within 1 week c. patient expresses fewer nonverbal signs of discomfort within 24 hrs d. patient increases calorie intake to 2500 daily

c. patient expresses fewer nonverbal signs of discomfort within 24 hrs

The microorganism that causes pneumonia can be detected in Mr. Johnson's: a. blood. b. urine. c. sputum. d. stool.

c. sputum

Based on Mr. Johnson's vital sign(Heart rate = 72 beats per minute Respiration rate = 28 breaths per minute Blood pressure = 135/87 mmHg. Temperature = 37.2° C (99° F) Oral), he is having: a. Hyperthermia. b. Hypotension. c. Tachypnea. d. Bradycardia.

c. tachypnea

Be objective in asking questions of a patient. a. open-mindedness b. analyticity c. truth seeking d. systematicity

c. truth seeking

The nurse is caring for seven patients this shift. After completing their assessments, the nurse states that he doesn't know where to begin in developing care plans for these patients. Which of the following is an appropriate suggestion by another nurse? a. "Choose all the interventions and perform them in order of time needed for each one." b. "Make sure you identify the scientific rationale for each intervention first." c. "Decide on goals and outcomes you have chosen for the patients." d. "Begin with the highest priority diagnoses, then select appropriate interventions."

d. "Begin with the highest priority diagnoses, then select appropriate interventions."

When making rounds, the nurse finds a patient who is not able to sleep because of surgery in the morning. Which therapeutic response is most appropriate? a. "It will be okay. Your surgeon will talk to you in the morning." b. "Why can't you sleep? You have the best surgeon in the hospital." c. "Don't worry. The surgeon ordered a sleeping pill to help you sleep." d. "It must be difficult not to know what the surgeon will find. What can I do to help?"

d. "It must be difficult not to know what the surgeon will find. What can I do to help?"

Which of the following documentation entries is most accurate? a. "Patient walked up and down hallway with assistance, tolerated well." b. "Patient up, out of bed, walked down hallway and back to room, tolerated well." c. "Patient up, walked 50 feet and back down hallway with assistance from nurse. Spouse also accompanied patient during the walk." d. "Patient walked 50 feet and back down hallway with assistance from nurse; HR 88 and regular before exercise, HR 94 and regular following exercise."

d. "Patient walked 50 feet and back down hallway with assistance from nurse; HR 88 and regular before exercise, HR 94 and regular following exercise." This provides the most accurate, objective information for the chart.

Which of the following statements made by the nurse shows the most informed understanding of the role of family in the client's postoperative recovery? a. "The family will be the ones you will be dealing with regarding postoperative needs." b. "When the family is more relaxed about caring for the client, the client is more relaxed." c. "The more the family understands what to expect during recovery, the more comfortable they are in caring for the client." d. "Teaching the family what they need to know before the surgery will maximize their effectiveness regarding the client's postoperative care."

d. "Teaching the family what they need to know before the surgery will maximize their effectiveness regarding the client's postoperative care." Often a family member is the caregiver when the client recovers from surgery. Perioperative preparation of family members before surgery helps to maximize effective caregiving while minimizing anxiety and misunderstanding.

As the nurse enters a patient's room, the nurse notices that the patient is anxious. The patient quickly states, "I don't know what's going on; I can't get an explanation from my doctor about my test results. I want something done about this." Which of the following is the most appropriate way for the nurse to document this observation of the patient? a. "The patient has a defiant attitude and is demanding test results." b. "The patient appears to be upset with the nurse because he wants his test results immediately." c. "The patient is demanding and is complaining about the doctor." d. "The patient stated feelings of frustration from the lack of information received regarding test results."

d. "The patient stated feelings of frustration from the lack of information received regarding test results." This is a nonjudgmental statement regarding the nurse's observations about the patient. Documenting that the patient has a defiant attitude or is demanding is judgmental, and information in the medical record should be factual and nonjudgmental. Noting that the patient appeared upset with the nurse needs to be more specific; it does not provide enough information regarding the reason for the patient's concern.

Which entry will require follow-up by the nurse manager? 0800 Patient states, "Fell out of bed." Patient found lying by bed on the floor. Legs equal in length bilaterally with no distortion, pedal pulses strong, leg strength equal and strong, no bruising or bleeding. Neuro checks within normal limits. States, "Did not pass out." Assisted back to bed. Call bell within reach. Bed monitor on. -------------------Jane More, RN 0810 Notified primary care provider of patient's status. New orders received. -------------------Jane More, RN 0815 Portable x-ray of L hip taken in room. States, "I feel fine." -------------------Jane More, RN 0830 Incident report completed and placed on chart. -------------------Jane More, RN a. 0800 b. 0810 c. 0815 d. 0830

d. 0830

Which of the following BP readings of Mr. Johnson indicates that he is experiencing orthostatic hypotension? a. 118/68 when standing and 110/72 when lying down. b. 140/80, HR 82 bpm when sitting and 136/76, HR 98 bpm when standing. c. 126/72 lying down and 133/80 when sitting. d. 146/88 when lying down and 130/78 when standing.

d. 146/88 when lying down and 130/78 when standing.

A nurse is conducting a patient-centered interview. Place the statements from the interview in the correct order, beginning with the first statement a nurse would ask. 1. "You say you've lost weight. Tell me how much weight you've lost in the last month." 2. "My name is Todd. I'll be the nurse taking care of you today. I'm going to ask you a series of questions to gather your health history." 3. "I have no further questions. Thank you for your patience." 4. "Tell me what brought you to the hospital." 5. "So, to summarize, you've lost about 6 lbs in the last month, and your appetite has been poor—correct?" a. 4, 2, 1, 3, 5 b. 2, 4, 3, 1, 5 c. 4, 2, 5, 1, 3 d. 2, 4, 1, 5, 3

d. 2, 4, 1, 5, 3 This follows the correct order for the interview.

What is the correct order for an ostomy pouch change? 1. Close the end of the pouch. 2. Measure the stoma. 3. Cut the hole in the wafer. 4. Press the pouch in place over the stoma. 5. Remove the old pouch. 6. Trace the correct measurement onto the back of the wafer. 7. Assess the stoma and the skin around it. 8. Cleanse and dry the peristomal skin. a. 5, 8, 2, 7, 3, 6, 4, 1 b. 8, 5, 6, 2, 7, 3, 4, 1 c. 8, 5, 7, 6, 2, 3, 4, 1 d. 5, 8, 7, 2, 6, 3, 4, 1

d. 5, 8, 7, 2, 6, 3, 4, 1 This order of tasks describes the correct way to change an ostomy pouch.

Match the following steps for administering a prepackaged enema with the correct order in which they occur. 1. Insert enema tip gently in the rectum. 2. Help patient to bathroom when he or she feels urge to defecate. 3. Position patient on side. 4. Perform hand hygiene and apply clean gloves. 5. Squeeze contents of container into rectum. 6. Explain procedure to the patient. a. 6, 3, 4, 1, 5, 2 b. 6, 4, 1, 3, 2, 5 c. 4, 6, 3, 1, 2, 5 d. 6, 4, 3, 1, 5, 2

d. 6, 4, 3, 1, 5, 2 These steps are used to administer a prepackaged enema.

A nurse is assigned to care for the following patients. Which of the patients is most at risk for developing skin problems and thus requiring thorough bathing and skin care? a. A 44-year-old female who has had removal of a breast lesion and is having her menstrual period. b. A 56-year-old male patient who is homeless and admitted to the emergency department with malnutrition and dehydration and who has an intravenous line. c. A 60-year-old female who experienced a stroke with right-sided paralysis and has an orthopedic brace applied to the left leg. d. A 70-year-old patient who has diabetes and dementia and has been incontinent of stool.

d. A 70-year-old patient who has diabetes and dementia and has been incontinent of stool. All of the patients require careful bathing. The 44-year-old female needs good perineal hygiene. The 56-year-old patient is at risk for drying and fragility of the skin. The 60-year-old patient has reduced sensation and mobility and thus is unaware of skin problems or pressure areas. However, the 70-year-old patient has reduced circulation, which increases risk for infection, and is likely unaware of skin problems because of dementia. The presence of stool will also irritate the skin.

Which of the following describes a hydrocolloid dressing? a. A seaweed derivative that is highly absorptive b. Premoistened gauze placed over a granulating wound c. A debriding enzyme that is used to remove necrotic tissue d. A dressing that forms a gel that interacts with the wound surface

d. A dressing that forms a gel that interacts with the wound surface

Which of the following nursing actions would most increase a patient's risk for developing a health care-associated infection? a. Use of a sterile bottled solution more than once within a 24-hour period b. Use of surgical aseptic technique to suction an airway c. Urinary catheter drainage bag placed below the level of the bladder d. Clean technique for inserting a urinary catheter

d. Clean technique for inserting a urinary catheter A foley catheter is inserted using sterile technique

A 72-year-old male patient comes to the health clinic for an annual follow-up. The nurse enters the patient's room and notices him to be diaphoretic, holding his chest and breathing with difficulty. The nurse immediately checks the patient's heart rate and blood pressure and asks him, "Tell me where your pain is." Which of the following assessment approaches does this scenario describe? a. Review of systems approach b. Use of a structured database format c. Back channeling d. A problem-oriented approach

d. A problem-oriented approach This is an example of a problem-focused approach. The nurse focuses on assessing one body system (cardiovascular) to determine the nature of the patient's pain and other presenting symptoms.

A patient presents to the emergency department following a motor vehicle crash and suffers from a right femur fracture. The leg is stabilized in a full leg cast. Otherwise, the patient has no other major injuries, is in good health, and complains only of moderate discomfort. What is the most pertinent nursing diagnosis to be included in the plan of care based on the assessment data provided? a. Posttrauma syndrome b. Constipation c. Urinary retention d. Acute pain

d. Acute pain

A patient recovering from a leg fracture after a fall states that he has dull pain in the affected leg and rates it as a 7 on a 0 to 10 scale. The patient is not able to walk around in the room with crutches because of leg discomfort. What is the priority nursing intervention for this patient? a. Assist the patient to walk in the room with crutches. b. Obtain a walker for the patient. c. Consult physical therapy. d. Administer pain medication.

d. Administer pain medication.

An older adult is being started on a new antihypertensive medication. In teaching the patient about the medication, the nurse: a. Speaks loudly. b. Presents the information once. c. Expects the patient to understand the information quickly. d. Allows the patient time to express himself or herself and ask questions.

d. Allows the patient time to express himself or herself and ask questions. When teaching older adults, it is important to establish rapport, involve them in their care, and allow them to progress at their own pace.

A nurse has seen many cancer patients struggle with pain management because they are afraid of becoming addicted to the medicine. Pain control is a priority for cancer care. By helping patients focus on their values and beliefs about pain control, a nurse can best make clinical decisions. This is an example of: a. Creativity. b. Fairness. c. Clinical reasoning. d. Applying ethical criteria.

d. Applying ethical criteria. The use of ethical criteria for nursing judgment allows a nurse to focus on a patient's values and beliefs. Clinical decisions are then just, faithful to the patient's choices, and beneficial to the patient's well-being.

A new graduate nurse is not sure what the heart sound is that she is listening to on a patient. To avoid diagnostic error, what should the nurse do? a. Assign the nursing diagnosis of Decreased cardiac output. b. Ask the patient if he has a history of cardiac problems before assigning the diagnosis of Decisional conflict. c. Check the previous shift's assessment and document what was noted on the last shift. d. Ask a more experienced nurse to listen also.

d. Ask a more experienced nurse to listen also.

A nurse prepares to insert a Foley catheter. The procedure manual calls for the patient to lie in the dorsal recumbent position. The patient complains of having back pain when lying on her back. Despite this, the nurse positions the patient supine with knees flexed as the manual recommends and begins to insert the catheter. This is an example of: a. Accuracy. b. Reflection. c. Risk taking. d. Basic critical thinking.

d. Basic critical thinking. Basic critical thinking is concrete and based on a set of rules or principles such as the guidelines in a hospital procedure manual. The nurse's approach is not accurate because accuracy requires use of all of the facts (e.g., the patient's discomfort). A critical thinker is willing to take risks to try different ways to solve problems; following one basic approach is not risk taking. This is also not an example of reflection.

When repositioning an immobile patient, the nurse notices redness over the hip bone. What is indicated when a reddened area blanches on fingertip touch? a. A local skin infection requiring antibiotics b. Sensitive skin that requires special bed linen c. A stage III pressure ulcer needing the appropriate dressing d. Blanching hyperemia, indicating the attempt by the body to overcome the ischemic episode.

d. Blanching hyperemia, indicating the attempt by the body to overcome the ischemic episode. When repositioning an immobile patient, it is important to assess all bony prominences for the presence of redness, which can be the first sign of impaired skin integrity. Pressing over the area compresses the blood vessels in the area; and, if the integrity of the vessels is good, the area turns lighter in color and then returns to the red color. However, if the area does not blanch when pressure is applied, tissue damage is likely.

A nurse is caring for a patient who smokes and drinks caffeine. Which point is important for the nurse to understand before she assesses the patient's blood pressure? a. Neither caffeine nor smoking affects blood pressure. b. She needs to insist that the patient stop smoking for at least 3 hours. c. The nurse should have the patient perform mild exercises. d. Caffeine and smoking can cause false BP elevations.

d. Caffeine and smoking can cause false BP elevations.

The patient is found to be unresponsive and not breathing. To determine the presence of central blood circulation and circulation of blood to the brain, the nurse checks the patient's _____ pulse. a. Radial b. Brachial c. Posterior tibial d. Carotid

d. Carotid

In caring for a patient who is immobile, it is important for the nurse to understand that a. The effects of immobility are the same for everyone. b. Immobility helps maintain sleep-wake patterns. c. Immobile patients are often eager to help in their own care. d. Changes in role and self-concept may lead to depression.

d. Changes in role and self-concept may lead to depression.

A nurse wants to reduce data entry errors on the computer system. Which behavior should the nurse implement? a. Use the same password all the time. b. Share password with only one other staff member. c. Print out and review computer nursing notes at home. d. Chart on the computer immediately after care is provided.

d. Chart on the computer immediately after care is provided.

When obtaining a wound culture to determine the presence of a wound infection, from where should the specimen be taken? a. Necrotic tissue b. Wound drainage c. Wound circumference d. Cleansed wound

d. Cleansed wound Drainage that has been present on the wound surface can contain bacteria from the skin, and the culture may not contain the true causative organisms of a wound infection. By cleaning the area before obtaining the culture, the skin flora is removed.

A nurse assesses patients and uses assessment findings to identify patient problems and develop an individualized plan of care. The nurse is displaying: a. Organizational skills. b. Use of resources. c. Priority setting. d. Clinical decision making.

d. Clinical decision making. Clinical decision making depends on the application of the nursing process. You first complete a patient assessment so you are able to make accurate judgment about the patient's nursing diagnoses and health problems. The next step is to complete a plan of care for the patient. You use critical thinking in the clinical decision process.

The unit manager is evaluating the care of a new nursing staff member. Which of the following is an appropriate technique for the nurse to implement in order to obtain a clean-voided urine specimen? a. Apply sterile gloves for the procedure b. Restrict fluids before the specimen collection c. Place the specimen in a clean urinalysis container. d. Collect the specimen after the initial stream of urine has passed.

d. Collect the specimen after the initial stream of urine has passed. To collect a clean-voided specimen, the nurse should collect the specimen (30 to 60 mL) after the initial stream of urine has passed. Nonsterile gloves are adequate. Fluids are encouraged so the client will be more likely to be able to void. The specimen should be collected in a sterile container and then placed into a plastic specimen bag.

A nurse interviewed and conducted a physical examination of a patient. Among the assessment data the nurse gathered were an increased respiratory rate, the patient reporting difficulty breathing while lying flat, and pursed-lip breathing. This data set is an example of: a. Collaborative data set. b. Diagnostic label. c. Related factors. d. Data cluster.

d. Data cluster. A data cluster is a set of cues (i.e., the signs or symptoms gathered during assessment).

An obese client is admitted for abdominal surgery. The nurse recognizes that this client is more susceptible to the postoperative complication of: a. Anemia b. Seizures c. Protein loss d. Dehiscence

d. Dehiscence An obese client is susceptible to poor wound healing and wound infection because of the structure of fatty tissue, which contains a poor blood supply. This increases the risk for dehiscence. A client who is malnourished is more susceptible to being anemic. A client with liver disease may have altered protein metabolism.

A registered nurse administers pain medication to a patient suffering from fractured ribs. What type of nursing intervention is this nurse implementing? a. Collaborative b. Independent c. Interdependent d. Dependent

d. Dependent

A nurse enters a 72-year-old patient's home and begins to observe her behaviors and examine her physical condition. The nurse learns that the patient lives alone and notices bruising on the patient's leg. When watching the patient walk, the nurse notes that she has an unsteady gait and leans to one side. The patient admits to having fallen in the past. The nurse identifies the patient as having the nursing diagnosis of Risk for Falls. This scenario is an example of: a. Inference. b. Basic critical thinking. c. Evaluation. d. Diagnostic reasoning.

d. Diagnostic reasoning. Diagnostic reasoning begins when you interact with a patient or make physical or behavioral observations. An expert nurse sees the context of a patient situation (e.g., patient lives alone, has fallen in past), observes patterns and themes, and makes a diagnostic decision.

A patient has been admitted for a cerebrovascular accident (stroke). She cannot move her right arm, and she has a right-sided facial droop. She is able to eat with her dentures in place and swallow safely. The nursing assistive personnel (NAP) reports to you that the patient will not keep the oral thermometer probe in her mouth. What direction do you provide to the NAP? a. Direct the NAP to hold the thermometer in place with her gloved hand b. Direct the NAP to switch the thermometer probe to the left sublingual pocket c. Direct the NAP to obtain a right tympanic temperature d. Direct the NAP to use a temporal artery thermometer from right to left

d. Direct the NAP to use a temporal artery thermometer from right to left A temporal artery temperature verifies the forehead temperature in back of the left ear, which is the side not affected by the altered blood flow related to the stroke. Holding the thermometer or switching locations will not help the patient close her mouth during temperature assessment. The patient's right side has vascular changes related to the stroke.

A nurse assesses a young woman who works part time but also cares for her mother at home. The nurse reviews clusters of data that include the patient's report of frequent awakenings at night, reduced ability to think clearly at work, and a sense of not feeling well rested. Which of the following diagnoses is in the correct PES format? a. Disturbed Sleep Pattern evidenced by frequent awakening b. Disturbed Sleep Pattern related to family caregiving responsibilities c. Disturbed Sleep Pattern related to need to improve sleep habits d. Disturbed Sleep Pattern related to caregiving responsibilities as evidenced by frequent awakening and not feeling rested

d. Disturbed Sleep Pattern related to caregiving responsibilities as evidenced by frequent awakening and not feeling rested A nursing diagnosis in a PES format includes the diagnostic label, related factor, and the defining characteristics by which the diagnosis is evidenced. The second nursing diagnosis is the correct format in the two-part format for writing a diagnosis. The first diagnosis has no related factor. The third diagnosis is an error, using a goal as a related factor.

The client had surgery in the morning that involved the right femoral artery. To assess the client's circulation status to the right leg, the nurse will make sure to check the pulse at the: a. Radial artery b. Ulnar artery c. Brachial artery d. Dorsalis pedis artery

d. Dorsalis pedis artery The nurse should assess peripheral pulses and capillary refill distal to the site of surgery. After surgery to the femoral artery, the nurse assesses posterior tibial and dorsalis pedis pulses. The nurse also compares pulses in the affected extremity with those in the nonaffected extremity.

The patient that will cause the greatest communication concerns for a nurse is the patient who is a. Alert, has strong self-esteem, and is hungry. b. Oriented, pain free, and blind. c. Cooperative, depressed, and hard of hearing. d. Dyspneic, has a tracheostomy, and is anxious.

d. Dyspneic, has a tracheostomy, and is anxious.

The client asks the nurse the purpose of having medications (Demerol and Vistaril) given before surgery. The nurse should inform the client that these particular medications: a. Reduce preoperative fear b. Promote emptying of the stomach c. Reduce body secretions d. Ease the induction of the anesthesia

d. Ease the induction of the anesthesia Preoperative medications such as Demerol and Vistaril help reduce the client's anxiety, the amount of general anesthesia required, the risk for nausea and vomiting and resulting aspiration, and the amount of respiratory secretions. They may also help the client feel drowsy and lessen his or her anxiety associated with fear. Vistaril (hydroxyzine pamoate) is often given to control nausea and vomiting by suppressing the central nervous system (CNS). Vistaril will have an anticholinergic effect, reducing body secretions. These medications given together will ease the induction of anesthesia.

Because an older adult is at increased risk for respiratory complications after surgery, the nurse should: a. Withhold pain medications and ambulate the patient every 2 hours. b. Monitor fluid and electrolyte status every shift and vital signs with temperature every 4 hours. c. Orient the patient to the surrounding environment frequently and ambulate him or her every 2 hours. d. Encourage the patient to turn, deep breathe, and cough frequently and ensure adequate pain control.

d. Encourage the patient to turn, deep breathe, and cough frequently and ensure adequate pain control. Adequate pain control is important to allow participation in postoperative exercises such as turning, deep coughing, and deep breathing to prevent respiratory complications.

A manager is reviewing the nursing documentation entered by a staff nurse in a patient's electronic medical record and finds the following entry, "Patient is difficult to care for, refuses suggestion for improving appetite." Which of the following statements is most appropriate for the manager to make to the staff nurse who entered this information? a. "Avoid rushing when documenting an entry in the medical record." b. "Use correction fluid to remove the entry." c. "Draw a single line through the statement and initial it." d. Enter only objective and factual information about a patient in the medical record.

d. Enter only objective and factual information about a patient in the medical record. Nurses should enter only objective and factual information about patients. Opinions have no place in the medical record. Because the information has already been entered and is not incorrect, it should be left on the record. Never use correction fluid in a written medical record.

A nurse assesses a 78-year-old patient who weighs 108.9 kg (240 lbs) and is partially immobilized because of a stroke. The nurse turns the patient and finds that the skin over the sacrum is very red and the patient does not feel sensation in the area. The patient has had fecal incontinence on and off for the last 2 days. The nurse identifies the nursing diagnosis of Risk for Impaired Skin Integrity. Which of the following outcomes is appropriate for the patient? a. Patient will be turned every 2 hours within 24 hours. b. Patient will have normal bowel function within 72 hours. c. Patient's skin integrity will remain intact through discharge. d. Erythema of skin will be mild to none within 48 hours.

d. Erythema of skin will be mild to none within 48 hours. Turning the patient every 2 hours in a 24-hour period is an intervention. Both "Patient will have normal bowel function within 72 hours" and "Patient's skin integrity will remain intact through discharge" are goals.

A nursing student is reviewing a process recording with the instructor. The student engaged the patient in a discussion about availability of family members to provide support at home once the patient is discharged. The student reviews with the instructor whether the comments used encouraged openness and allowed the patient to "tell his story." This is an example of which step of the nursing process? a. Planning b. Assessment c. Intervention d. Evaluation

d. Evaluation Reviewing a conversation with a patient and determining whether the student encouraged openness and allowed the patient to "tell his story," expressing both thoughts and feelings, involve evaluation.

Which of the following pressure ulcers has the worse prognosis? a. Skin is intact with an area of nonblanching erythema. This is usually over a bony prominence. b. Partial-thickness skin loss with loss of the epidermis and some of the dermis. It appears as a shallow ulcer with a red-pink color. No slough or necrotic tissue is present in the base. It may also appear as an enclosed or open serum-filled blister. c. Full-thickness loss of skin with the epidermis and dermis gone and damage to or necrosis of subcutaneous tissues. Damage extends down to but not through the underlying fascia. Subcutaneous fat may be visible, but muscle, tendon, or bone is not seen. Slough may be present but does not hinder estimation of the extent of tissue loss. Tunneling or undermining may be present. d. Full-thickness loss of skin with extensive destruction, tissue necrosis, and damage to bone, muscle, or other supporting. structures that are exposed.

d. Full-thickness loss of skin with extensive destruction, tissue necrosis, and damage to bone, muscle, or other supporting. structures that are exposed.

The nurse is assessing the way the patient walks. The manner of walking is known as the patient's a. Activity tolerance. b. Body alignment. c. Range of motion. d. Gait.

d. Gait.

A nurse assesses a patient who comes to the pulmonary clinic. "I see that it's been over 6 months since you've been here, but your appointment was for every 2 months. Tell me about that. Also I see from your last visit that the doctor recommended routine exercise. Can you tell me how successful you've been in following his plan?" The nurse's assessment covers which of Gordon's functional health patterns? a. Value-belief pattern b. Cognitive-perceptual pattern c. Coping-stress-tolerance pattern d. Health perception-health management pattern

d. Health perception-health management pattern The nurse's assessment covers the health perception- health management pattern, which is a patient's self-report of how he or she manages his or her health and his or her knowledge of preventive health practices. The coping-stress tolerance pattern includes questions focused on how a patient manages stress and sources of support. An assessment covering the value belief pattern leads a patient to describe patterns of values, beliefs, and life goals. An assessment of the cognitive-perceptual pattern includes questions that focus on the patient's language adequacy, memory, and decision-making ability.

A nurse is teaching a group of young college-age women the importance of using sunscreen when going out in the sun. Which type of content is the nurse providing? a. Simulation b. Restoring health c. Coping with impaired function d. Health promotion and illness prevention

d. Health promotion and illness prevention Health promotion and illness prevention are the focus when nurses provide information to help patients improve their health and avoid illness.

Which task is appropriate for a registered nurse (RN) to delegate to a nursing assistant? a. Explaining to the patient the preoperative preparation before the surgery in the morning b. Administering the ordered antibiotic to the patient before surgery c. Obtaining the patient's signature on the surgical informed consent d. Helping the patient to the bathroom before leaving for the operating room

d. Helping the patient to the bathroom before leaving for the operating room Assisting the patient with toileting activities is within the scope of nursing assistive duties. The other activities require the skill and knowledge of the RN.

A nurse is assigned to care for a patient for the first time and states, "I don't know a lot about your culture and want to learn how to better meet your health care needs." Which therapeutic communication technique did the nurse use in this situation? a. Validation b. Empathy c. Sarcasm d. Humility

d. Humility Humility is admitting to limitations in knowledge and skill. This enables the nurse to admit a knowledge deficit so guidance is sought from the patient. Humility helps improve the therapeutic relationship and enables a nurse to provide safe and effective care.

A nursing student is reporting during hand-off to the RN assuming her patient's care. She explains, "I ambulated him twice during the shift; he tolerated well walking to end of hall and back with no shortness of breath. Mr. Roarke said he slept better last night after I closed his door and gave him a chance to be uninterrupted. I changed the dressing over his intravenous (IV) site and started a new bag of D5 ½ NS. Which intervention is a dependent intervention? a. Reporting hand-off at change of shift b. Ambulating patient down hallway c. Sleep hygiene d. IV fluid administration

d. IV fluid administration Administering IV fluids required a health care provider's order. The other three interventions are independent nursing activities.

A nurse is reviewing a patient's list of nursing diagnoses in the medical record. The most recent nursing diagnosis is Diarrhea related to intestinal colitis. For which of the following reasons is this an incorrectly stated diagnostic statement? a. Identifying the clinical sign instead of an etiology b. Identifying a diagnosis on the basis of prejudicial judgment c. Identifying the diagnostic study rather than a problem caused by the diagnostic study d. Identifying the medical diagnosis instead of the patient's response to the diagnosis.

d. Identifying the medical diagnosis instead of the patient's response to the diagnosis. Intestinal colitis is a medical diagnosis. The related factor in a nursing diagnostic statement is always within the domain of nursing practice and a condition that responds to nursing interventions. Nursing interventions do not change a medical diagnosis.

Which nursing entry is most complete in describing a client's wound? a. Wound appears to be healing well. Dressing dry and intact. b. Wound well approximated with minimal drainage. c. Drainage size of quarter; wound pink, 4 × 4s applied. d. Incisional edges approximated without redness or drainage; two 4 × 4s applied.

d. Incisional edges approximated without redness or drainage; two 4 × 4s applied. This is the most complete description of the client's wound. It describes the wound according to characteristics observed and the dressing that covers it. Wounds should be measured using the metric system, not described as the size of objects.

The nurse is evaluating the outcome "Client describes surgical procedures and postoperative treatment" and determines that the client has not achieved this outcome. The nurse should: a. Obtain the consent, because this is expected with preoperative anxiety b. Teach the client all about the procedure c. Ask the unit manager to assist with a teaching plan d. Inform the surgeon so that information can be provided

d. Inform the surgeon so that information can be provided When the client has little or no understanding about the surgery, the health care provider will need to be notified to reinform the client. If the client does not understand the surgical procedure, the client would not be giving informed consent. It is the surgeon's responsibility to explain the procedure and obtain the informed consent. The nurse can augment the health care provider's explanations, but it is the health care provider's responsibility to teach the client about the procedure. This teaching includes the need for the procedure, steps involved, risks, expected results, and alternative treatments.

A patient's son decides to stay at the bedside while his father is confused. When developing the plan of care for this patient, what should the nurse do? a. Individualize the care plan only according to the patient's needs. b. Request that the son leave at bedtime, so the patient can rest. c. Suggest that a female member of the family stay with the patient. d. Involve the son in the plan of care as much as possible.

d. Involve the son in the plan of care as much as possible.

A patient has an indwelling urinary catheter. Why does an indwelling urinary catheter present a risk for urinary tract infection? a. It keeps an incontinent patient's skin dry. b. It allows the patient to remain hydrated without having to urinate. c. It can get caught in the linens or equipment. d. It obstructs the normal flushing action of urine flow.

d. It obstructs the normal flushing action of urine flow.

Without balance control, the center of gravity is displaced, thus creating risk for falls and subsequent injuries. Balance is enhanced by a. Maintaining a narrow base of support. b. Creating a high center of gravity. c. Disregarding body posture. d. Keeping a low center of gravity.

d. Keeping a low center of gravity.

During the nursing assessment a patient reveals that he has diarrhea and cramping every time he has ice cream. He attributes this to the cold nature of the food. However, the nurse begins to suspect that these symptoms are associated with what problem? a. Food allergy b. Irritable bowel c. Increased peristalsis d. Lactose intolerance

d. Lactose intolerance These symptoms are consistent with lactose intolerance, and they occur with ingestion of dairy products.

An older-adult patient has been bedridden for 2 weeks. Which of the following complaints by the patient indicates to the nurse that he or she is developing a complication of immobility? a. Loss of appetite b. Gum soreness c. Difficulty swallowing d. Left-ankle joint stiffness

d. Left-ankle joint stiffness Patients whose mobility is restricted require range-of-motion (ROM) exercises daily to reduce the hazards of immobility. Temporary immobilization results in some muscle atrophy, loss of muscle tone, and joint stiffness. Two weeks of joint immobilization without ROM can quickly result in contractures.

A patient was recently diagnosed with pneumonia. The nurse and the patient have established a goal that the patient will not experience shortness of breath with activity in 3 days with an expected outcome of having no secretions present in the lungs in 48 hours. Which of the following is an appropriate evaluative measure demonstrating progress toward this goal? a. Nonproductive cough present in 4 days. b. Scattered rhonchi throughout all lung fields in 2 days. c. Respirations 30/minute in 1 day. d. Lungs clear to auscultation following use of inhaler.

d. Lungs clear to auscultation following use of inhaler.

You are a nurse in the postanesthesia care unit (PACU), and you note that your patient has a heart rate of 130 beats/min and a respiratory rate of 32 breaths/min; you also assess jaw muscle rigidity and rigidity of limbs, abdomen, and chest. What do you suspect, and which intervention is indicated? a. Infection: Notify surgeon and anticipate administration of antibiotics. b. Pneumonia: Listen to breath sounds, notify surgeon, and anticipate order for chest radiography. c. Hypertension: Check blood pressure, notify surgeon, and anticipate administration of antihypertensives. d. Malignant hyperthermia: Notify surgeon/anesthesia provider immediately, prepare to administer dantrolene sodium (Dantrium), and monitor vital signs frequently.

d. Malignant hyperthermia: Notify surgeon/anesthesia provider immediately, prepare to administer dantrolene sodium (Dantrium), and monitor vital signs frequently. Malignant hyperthermia is a life-threatening complication of general anesthesia. It is a severe hypermetabolic condition that causes rigidity of skeletal muscles caused by an increase in intracellular calcium ion concentration and leads to hypercarbia, tachypnea, and tachycardia. An elevated temperature is a late sign, and an increase in the respiratory rate to eliminate carbon dioxide is one of the first signs. Dantrolene sodium (Dantrium) is a skeletal muscle relaxant used to treat this complication.

When assessing a patient's skin, the nurse needs to know that a. Restricted movement can increase blood circulation. b. Paralyzed patients have normal sensory function. c. Loss of subcutaneous tissue may increase the rate of wound healing. d. Moisture on the skin can lead to skin maceration.

d. Moisture on the skin can lead to skin maceration.

A patient's surgical wound has become swollen, red, and tender. You note that the patient has a new fever and leukocytosis. What is the best immediate intervention? a. Notify the health care provider and use surgical technique to change the dressing. b. Alert the patient and caregivers to the presence of an infection to ensure care after discharge. c. Reassure the patient and recheck the wound later. d. Notify the health care provider and support the patient's fluid and nutritional needs.

d. Notify the health care provider and support the patient's fluid and nutritional needs. Fever and leukocytosis are signs of systemic infection. Hydration is important for patients with fever. Nutrition can help promote wound healing. Surgical technique is not used for general wound dressing changes.

A 52-year-old woman is admitted with dyspnea and discomfort in her left chest with deep breaths. She has smoked for 35 years and recently lost over 10 lbs. Her vital signs on admission are: HR 112, BP 138/82, RR 22, tympanic temperature 36.8° C (98.2° F), and oxygen saturation 94%. She is receiving oxygen at 2 L via a nasal cannula. Which vital sign reflects a positive outcome of the oxygen therapy? a. Temperature: 37° C (98.6° F) b. Radial pulse: 112 c. Respiratory rate: 24 d. Oxygen saturation: 96% e. Blood pressure: 134/78

d. Oxygen saturation: 96% Oxygen saturation is an assessment of oxygen perfusion. Respiratory rate assesses ventilation, radial pulse and blood pressure assess the cardiovascular system, and temperature is an assessment of thermal regulation.

A nurse collects equipment needed to administer an enema to a patient. Previously the nurse reviewed the procedure in the policy manual. The nurse raises the patient's bed and adjusts the room lighting to illuminate the work area. A patient care technician comes into the room to assist. Which aspect of organizing resources and care delivery did the nurse omit? a. Environment b. Personnel c. Equipment d. Patient

d. Patient In preparing to administer the enema, the nurse did not prepare for the patient's physical and psychological comfort.

The nurse writes an expected outcome statement in measurable terms. An example is: a. Patient will have normal stool evacuation. b. Patient will have fewer bowel movements. c. Patient will take stool softener every 4 hours. d. Patient will report stool soft and formed with each defecation.

d. Patient will report stool soft and formed with each defecation. Stool that is soft and formed at each defecation is measurable upon observation. "Patient will have normal stool evacuation" is a goal. Indicating that the patient will have fewer bowel movements is not specific enough for measuring improvement, and having a patient take a stool softener every 4 hours is an intervention.

A family member is providing care to a loved one who has an infected leg wound. What would you instruct the family member to do after providing care and handling contaminated equipment or organic material? a. Wear gloves before eating or handling food. b. Place any soiled materials into a bag and double bag it. c. Have the family member check with the health care provider about need for immunization. d. Perform hand hygiene after care and/or handling contaminated equipment or material.

d. Perform hand hygiene after care and/or handling contaminated equipment or material. Clean hands interrupt the transmission of microorganisms from family members.

The client is scheduled for abdominal surgery and has just received the preoperative medications. The nurse should: a. Keep the client quiet b. Obtain the consent c. Prepare the skin at the surgical site d. Place the side rails up on the bed or stretcher

d. Place the side rails up on the bed or stretcher After administering preoperative medications, the nurse should raise the side rails on the bed or stretcher and keep the bed or stretcher in low position. Preanesthetic medications will help reduce the client's anxiety. Consent must be obtained before preoperative medications are administered or the consent is invalid. Preparing the skin at the surgical site is often done in the operating room.

The nurse is concerned that the client's midsternal wound is at risk for dehiscence. Which of the following is the best intervention to prevent this complication? a. Administering antibiotics to prevent infection b. Using appropriate sterile technique when changing the dressing c. Keeping sterile towels and extra dressing supplies near the client's bed d. Placing a pillow over the incision site when the client is deep breathing or coughing

d. Placing a pillow over the incision site when the client is deep breathing or coughing A strategy to prevent dehiscence is to use a folded thin blanket or pillow placed over an abdominal wound when the client is coughing. This provides a splint to the area, supporting the healing tissue when coughing increases the intraabdominal pressure. A client who has an infection is at risk for poor wound healing and dehiscence. However, prophylactic use of antibiotics is not the best intervention to prevent dehiscence. Using appropriate sterile technique is always important to prevent the development of infection but is not the best intervention to prevent dehiscence.

Urinary elimination may be altered with different pathophysiological conditions. For the client with diabetes mellitus, the nurse anticipates that an initial urinary sign or symptom will be: a. Urgency b. Dysuria c. Hematuria d. Polyuria

d. Polyuria An initial urinary symptom of diabetes mellitus is polyuria. Urgency is not a symptom of diabetes mellitus. Urgency may be caused by a full bladder, bladder irritation from infection, incompetent urethral sphincter, or psychological stress. Dysuria is not a symptom of diabetes mellitus. Dysuria may be caused by bladder inflammation, trauma, or inflammation of the urethral sphincter. Hematuria is not a symptom of diabetes mellitus. Hematuria may be a symptom of neoplasms of the bladder or kidney, glomerular disease, infection of the kidney or bladder, trauma to urinary structures, calculi, or bleeding disorders.

In the postanesthesia care unit (PACU) the nurse notes that the patient is having difficulty breathing and suspects an upper-airway obstruction. The nurse would first: a. Suction the pharynx and bronchial tree. b. Give oxygen through a mask at 4 L/min. c. Ask the patient to use an incentive spirometer. d. Position the patient on one side with the face down and the neck slightly extended so the tongue falls forward.

d. Position the patient on one side with the face down and the neck slightly extended so the tongue falls forward. Weak pharyngeal/laryngeal muscle tone from anesthetics can occur. Positional change helps to move the tongue forward to open the airway. The immediate intervention should be to open the airway. Suctioning the bronchial tree or providing oxygen does not alleviate an upper-airway obstruction.

The nurse is reviewing the report of a client's routine urinalysis. Which value should the nurse consider abnormal? a. Specific gravity of 1.03 b. Urine pH of 5.0 c. Absence of protein d. Presence of glucose

d. Presence of glucose Normal urine pH is 4.5 to 8; therefore, a urine pH of 3.0 is abnormal. Urine specific gravity normally ranges from 1.002 to 1.035, making this client's value normal. Normally, urine contains no protein, glucose, ketones, bilirubin, bacteria, casts, or crystals. Red blood cells should measure 0 to 3 per high-power field; white blood cells, 0 to 4 per high-power field. Urine should be clear, its color ranging from pale yellow to deep amber.

The nurse begins a shift assessment by examining a surgical dressing that is saturated with serosanguineous drainage on a patient who had open abdominal surgery yesterday (or 1 day ago). The nurse is performing what type of assessment approach in this situation? a. Comprehensive assessment. b. General to specific assessment. c. Activity-exercise pattern assessment d. Problem-oriented assessment.

d. Problem-oriented assessment.

The nurse puts elastic stockings on a patient following major abdominal surgery. The nurse teaches the patient that the stockings are used after a surgical procedure to: a. Prevent varicose veins. b. Prevent muscular atrophy. c. Ensure joint mobility and prevent contractures. d. Promote venous return to the heart.

d. Promote venous return to the heart.

A patient needs to learn to use a walker. Which domain is required for learning this skill? a. Affective domain b. Cognitive domain c. Attentional domain d. Psychomotor domain

d. Psychomotor domain Using a walker requires the integration of mental and muscular activity.

Which behaviors indicate that the student nurse has a good understanding of confidentiality and the Health Insurance Portability and Accountability Act (HIPAA)? (Select all that apply.) a. Writes the patient's room number and date of birth on a paper for school b. Prints/copies material from the patient's health record for a graded care plan c. Reviews assigned patient's record and another unassigned patient's record d. Reads the progress notes of assigned patient's record e. Gives a change-of-shift report to the oncoming nurse about the patient f. Discusses patient care with the hospital volunteer

d. Reads the progress notes of assigned patient's record e. Gives a change-of-shift report to the oncoming nurse about the patient

A 92-year-old client is scheduled for a colectomy. Which normal physiological change that accompanies the aging process increases this client's risk for surgery? a. An increased tactile sensation b. An increased metabolic rate c. A relaxation of arterial walls d. Reduced glomerular filtration rate

d. Reduced glomerular filtration rate An older adult is likely to have a reduced glomerular filtration rate. This limits the body's ability to eliminate drugs or toxic substances. An older adult has reduced tactile sense, which decreases the client's ability to respond to early warning signs of surgical complications, including sensing pressure over bony prominences. An older adult has a lower basal metabolic rate, reducing total oxygen consumption. The nurse should ensure the client obtains adequate nutritional intake when diet is resumed, but the client should avoid intake of excess calories.

The nurse is planning a program on wound healing and includes information that smoking influences healing by: a. Suppressing protein synthesis b. Creating increased tissue fragility c. Depressing bone marrow function d. Reducing functional hemoglobin in the blood

d. Reducing functional hemoglobin in the blood Smoking reduces the amount of functional hemoglobin in the blood, thus decreasing tissue oxygenation. Antiinflammatory drugs suppress protein synthesis. Radiation creates tissue fragility. Chemotherapeutic drugs can depress bone marrow function.

After caring for a young man newly diagnosed with diabetes, a nurse is reviewing what was completed in his plan of care following discharge. She considers how she related to the patient and whether she selected interventions best suited to his educational level. It was the nurse's first time caring for a new patient with diabetes. The nurse's behavior is an example of which of the following? a. Reflection-in-action b. Reassessment c. Reprioritizing d. Reflection-on-action

d. Reflection-on-action The nurse is performing reflection-on-action. This means that when you gather evaluative measures about a patient, reflection on the findings and the exploration about what the findings might mean improve your ability to problem solve. The other three measures occur during evaluation because the nurse is still actively intervening in the patient's care.

Your assigned patient has a leg ulcer that has a dressing on it. During your assessment you find that the dressing is saturated with purulent drainage. Which action would be best on your part? a. Reinforce dressing with a clean, dry dressing and call the health care provider. b. Remove wet dressing and apply new dressing using sterile procedure. c. Put on gloves before removing the old dressing; then obtain a wound culture. d. Remove saturated dressing with gloves, remove gloves, then perform hand hygiene and apply new gloves before putting on a clean dressing.

d. Remove saturated dressing with gloves, remove gloves, then perform hand hygiene and apply new gloves before putting on a clean dressing. Gloves need to be changed, and hand hygiene performed to prevent transfer of microorganisms from one source to another. Gloves may have microscopic holes that allow microorganisms to have contact with the caregiver's skin. Therefore gloves are removed, and hand hygiene is performed whenever the nurse moves from an activity requiring gloves to another nursing action or leaves the patient's room and whenever all patient tasks are completed.

The nurse is providing perineal care to an uncircumcised male patient. When providing such care, the nurse should a. Leave the foreskin alone because there is little chance of infection. b. Retract the foreskin for cleansing and allow it to return on its own. c. Retract the foreskin and keep retracted. d. Retract the foreskin and return it to its natural position when done.

d. Retract the foreskin and return it to its natural position when done. the foreskin should be pulled back and cleaned to prevent infection. The foreskin should be returned to original position when completed to prevent dryness, constriction or irritation of the head of the penis.

A newly admitted patient who is morbidly obese asks the nurse to assist her to the bathroom for the first time. What should the nurse do first? a. Ask for at least two other assistive personnel to come to the room. b. Medicate the patient to alleviate discomfort while ambulating. c. Offer the patient a walker. d. Review the patient's activity orders.

d. Review the patient's activity orders.

Which of the following best describes the primary nursing role regarding a client's consent to surgery immediately before surgery? a. Explaining the procedure to the client in a fashion that is easily understood b. Placing the signed consent in the client's medical record c. Ensuring that the client understands the possible risks of the procedure before signing the consent d. Reviewing the client's surgical consent as a part of the routine preoperative checklist

d. Reviewing the client's surgical consent as a part of the routine preoperative checklist It is the surgeon's responsibility to explain the procedure and obtain the informed consent. After the client completes the consent form, place it in the medical record. The record goes to the operating room with the client after the nurse confirms all required information has been included.

The nursing assessment of a 78-year-old woman reveals orthostatic hypotension, weakness on the left side, and fear of falling. On the basis of the patient's data, which one of the following nursing diagnoses indicates an understanding of the assessment findings? a. Activity Intolerance b. Impaired Bed Mobility c. Acute Pain d. Risk for Falls

d. Risk for Falls For adults age 65 and older, orthostatic hypotension, fear of falling, and weakness on one side are risks for the nursing diagnosis of Risk for Falls.

A patient who visits the surgery clinic 4 weeks after a traumatic amputation of his right leg tells the nurse practitioner that he is worried about his ability to continue to support his family. He tells the nurse he feels that he has let his family down after having an auto accident that led to the loss of his left leg. The nurse listens and then asks the patient, "How do you see yourself now?" On the basis of Gordon's functional health patterns, which pattern does the nurse assess? a. Health perception-health management pattern b. Value-belief pattern c. Cognitive-perceptual pattern d. Self-perception-self-concept pattern

d. Self-perception-self-concept pattern This is an example of assessment of a patient's feelings about his worth and body image, which is the self-perception- self-concept health pattern.

A nurse is preparing a change-of-shift report for a patient who had chest pain. Which information is critical for the nurse to include? a. Pupils equal and reactive to light b. The family is a "pain" c. Had poor results from the pain medication d. Sharp pain of 8 on a scale of 1 to 10

d. Sharp pain of 8 on a scale of 1 to 10

After providing care, a nurse charts in the patient's record. Which entry should the nurse document? a. Appears restless when sitting in the chair b. Drank adequate amounts of water c. Apparently is asleep with eyes closed d. Skin pale and cool

d. Skin pale and cool

The nurse is providing an education session to an adult community group about the effects of smoking. Which of the following is the most important point to be included in the educational session? a. Smoking can affect the color of the patient's fingernails. b. Smoking tobacco products can be very expensive. c. Smoke from tobacco products clings to your clothing and hair. d. Smoking affects the cilia lining the upper airways in the lungs.

d. Smoking affects the cilia lining the upper airways in the lungs.

A nurse uses SBAR during hand-offs. The purpose of SBAR is to a. Use common courtesy. b. Establish trustworthiness. c. Promote autonomy. d. Standardize communication

d. Standardize communication

A 64-year-old male client has been scheduled to undergo surgery for a total knee replacement. The client would like to be able to use his own blood for the surgery, if needed. The nurse explains that there are several advantages to the client's having an autologous infusion, but there are some drawbacks as well. Which of the following would be considered a drawback to an autologous infusion? a. The client has a decreased risk for contracting HIV. b. There is a decreased risk for infection. c. The client has less risk for a transfusion reaction. d. The client may have a decreased hemoglobin and hematocrit level on the day of surgery.

d. The client may have a decreased hemoglobin and hematocrit level on the day of surgery. The client must plan ahead in plenty of time in order to be able to donate his own blood. In addition, the client who does self-donation sometimes exhibits a lower hemoglobin and hematocrit level on the day of surgery. Autologous infusions are an option for some clients who choose to donate their own blood before surgery to reduce the risk for transfusion-related infections. The client is at less risk for a transfusion reaction because it is his own blood. There is a lowered risk for infection because the blood is from the client.

The nurse recognizes that a client recovering from anesthesia required for surgical repair of a fractured ulna is likely to experience difficulty urinating primarily because of: a. The impaired cognitive state the client will experience as the effects of the anesthesia wear off b. The decreased volume of orally ingested fluids before, during, and after the surgical procedure c. The length of time the client was under the effects of general anesthesia required for the surgical procedure d. The effects of the anesthetic on the nerves and muscles controlling the relaxation of the urinary bladder

d. The effects of the anesthetic on the nerves and muscles controlling the relaxation of the urinary bladder Medications, including anesthesia, interfere with both the production and the characteristics of urine and affect the act of urination. The remaining options may affect urination but not to the extent of the anesthetic effects.

When a nurse delegates hygiene care for a male patient to a nurse assistant, the assistant to must use an electric razor to shave the patient with the following diagnosis: a. Congestive heart failure b. Pneumonia c. Arthritis d. Thrombocytopenia

d. Thrombocytopenia Patients prone to bleeding (e.g., those receiving anticoagulants or high doses of aspirin or those with low platelet counts) need to use an electric razor.

A parent calls the pediatrician's office to ask about directions for using a car seat. Which of the following is the most correct set of instructions the nurse gives to this parent? a. Only infants and toddlers need to ride in the back seat. b. All toddlers can move to a forward facing car seat when they reach age 2. c. Toddlers must reach age 2 and the height/weight requirement before they ride forward facing. d. Toddlers must reach age 2 or the height or weight requirement before they ride forward facing.

d. Toddlers must reach age 2 or the height or weight requirement before they ride forward facing. The American Academy of Pediatrics (2011a) recommends that all infants and toddlers ride in the back seat with a rear-facing-only seat and rear-facing convertible seat until they are 2 years of age or they reach the highest weight or height allowed by the manufacturer of the car safety seat.

When obtaining a sterile urine specimen from an indwelling urinary catheter the nurse should: a. Disconnect the catheter from the drainage tubing b. Withdraw urine from a urinometer c. Open the drainage bag and removing urine d. Use a needle to withdraw urine from the catheter port

d. Use a needle to withdraw urine from the catheter port A sterile specimen can be obtained through the special port found on the side of the indwelling catheter. The nurse clamps the tubing below the port, allowing fresh, uncontaminated urine to collect in the tube. After the nurse wipes the port with an antimicrobial swab, a sterile syringe needle is inserted, and at least 3 to 5 mL of urine is withdrawn. Using sterile technique, the nurse transfers the urine to a sterile container. The catheter should not be disconnected from the drainage tubing. The system should remain a closed system to prevent infection. A urinometer is a device used to determine the specific gravity of urine. It is not a sterile device and should not be used for obtaining a sterile urine specimen. Urine should not be obtained from a drainage bag for a specimen, because the urine would not be fresh and would be contaminated from microorganisms in the drainage bag.

A new graduate nurse will make the best clinical decisions by applying the components of the nursing critical thinking model and which of the following? a. Drawing on past clinical experiences to formulate standardized care plans b. Relying on recall of information from past lectures and textbooks c. Depending on the charge nurse to determine priorities of care d. Using the nursing process

d. Using the nursing process

Category/Stage III pressure ulcer a. nonblanchable redness of intact skin. Discoloration, warmth, edema, or pain may also be present. b. partial thickness skin loss or intact blister with serosanginous fluid c. full thickness tissue loss, muscle and bone visible. may include undermining d. full thickness skin loss, subcutaneous fat may be visible. may include undermining

d. full thickness skin loss, subcutaneous fat may be visible. may include undermining

An 82-year-old patient who resides in a nursing home has the following three nursing diagnoses: Risk for Fall, Impaired Physical Mobility related to pain, and Imbalanced Nutrition: Less Than Body Requirements related to reduced ability to feed self. The nursing staff identified several goals of care. Match the goals on the left with the appropriate outcome statements on the right. Goals Outcomes (Patient will achieve 5-lb weight gain in 1 month.) a. patient walks 20 feet using a walker in 24 hrs. b. patient identifies barriers to remove in the home within 1 week c. patient expresses fewer nonverbal signs of discomfort within 24 hrs d. patient increases calorie intake to 2500 daily

d. patient increases calorie intake to 2500 daily

Organize assessment on the basis of patient priorities. a. open-mindedness b. analyticity c. truth seeking d. systematicity

d. systematicity


Conjuntos de estudio relacionados

CompTIA Security+ - Lesson 11: Implementing Secure Network Protocols (Practice Questions)

View Set

ROLE OF KIDNEYS IN ACID-BASE BALANCE

View Set

CH 18 Beta-Lactam Antibacterial Agents (E1)

View Set

Chapter 3 (Federalism) AP Gov. classroom

View Set